HPAS Fully Solved Prelim Papers 2010-2018  HAS General Studies Prelim Solved Papers from 2010-2018 [II, II ed.]

Table of contents :
HPPSC HPAS General Studies Solved Papers
2018 HPAS Solved Paper
Q1. Oil refinery at Bhatinda is being established by:
Q2. Tata Iron and Steel Company at Jamshedpur was established in:
Q3. The term Operation Flood is associated with :
Q4. B.R. Ambedkar was born at :
Q5. Montague-Chelmsford Reforms were introduced in India in the year :
Q6. The first industrial policy was announced in:
Q7. Shingo-La is the name of which pass?
Q8. ‘Madho Rai ki Jaleb’ is the main attraction of which popular Himachali fair?
Q9. What is the architectural style of the temple built on the edge of the Prashar Lake?
Q10. In which district Central Government approved Rs. 80 lakh in 2011 for setting up of Snow Leopard centre?
Q11. Who is the author of the book ‘Polyandry in the Himalayas’?
Q12. The term 'Outwash plain' is associated with :
Q13. The concept of Normal Cycle of Erosion was given by :
Q14. The term 'Ria' stands for :
Q15. Who became first female President of Singapore?
Q16. Why Jagmeet Singh is in news?
Q17. Who is appointed as New Chairman of ISRO?
Q18. Renewable source of energy is :
Q19. Photosynthesis occurs in :
Q20. In which part of our body RBC formed?
Q21. In which year 'Project Crocodile' was launched in India?
Q22. 'Rataundhi' is due to deficiency of which Vitamin?
Q23. 'Dohru' is a traditional woollen dress worn by the women of which district?
Q24. The town of Rishra is known for the industry of:
Q25. Koraput has rich deposits of :
Q26. Who won the Women’s Single Wimbledon tournament in 2017?
Q27. Name the sportsperson who has been honoured with 'Bharat Ratna'?
Q28. 'Appiko Movement' was leaded by :
Q29. Which of the following is a non-renewable energy source?
Q30. Which famous football tournament was started in 1888 from Annadale?
Q31. Which was the old capital of Kullu Rajas from where the twelve generations of Kullu Rajas ruled?
Q32. The liquid used in a clinical thermometer is:
Q33. Who was awarded Best Actress award in the 64^th National Film awards function?
Q34. Match List-I and List-II and select the correct answer from the codes given below :
Q35. How many sustainable developments are there in the U.N. Agenda 2030?
Sustainable Development Goals (SDGs)
Q36. Which one of the following is the largest industry in India?
Q37. Which one of the following states have the highest percentage of irrigated area?
Q38. The term 'deflation' is associated with :
Q39. Which one of the following has the largest number of active volcanoes?
Q40. Which one of the following planets has the shortest day?
Q41. Which of the following valleys lies between the Dhauladhar and Pir Panjal?
Q42. Which of the following Himalayan zones occupies the maximum percentage of the total geographical area of Himachal Pradesh?
Q43. Where does river Yamuna, after flowing through Uttrakhand, enter in Sirmaur district of Himachal Pradesh?
Q44. The movement started under the aegis of ‘Kisan Sabha’ in which Shri Lakshmi Singh and Vaid Surat Singh played a pivotal role was:
Q45. ‘Democratisation of the Administration’ was the main aim of:
Q46. When Himachal Pradesh got right to participate in the election for the first time under the new constitution?
Q47. Which of the following Monasteries was built by a Western Tibet ruler ‘Ye-Sashoad’ around the year 996 A.D.?
Q48. In 1804 A.D. Rulers of Bilaspur, Mandi, Chamba and other small rulers of Kangra area invited whom to invade Kangra?
Q49. In 1848, who suppressed the rebellion of Wazir of Nurpur and then Pramod Chand and Rajas of Jaswan and Datarpur?
Q50. What is the approximate capacity of ‘Shontong Karcham’ Hydro Electric Power Project?
Q51. To improve the Gross Enrolment Ratio (GER) in Higher Education, the government of H.P. has adopted ‘Rashtriya Uchtar Shikha Abhiyan’ the centrally scheme in the year:
Q52. The Mushroom Development Project in Palampur was launched in 1985 with the introduction of the technology of which country?
Q53. What was the approximate decadal population growth rate of Himachal Pradesh in census 2011?
Q54. What is India’s estimated GDP growth rate for 2018-19 in the Economic Survey?
Q55. What was the Index of Industrial Production in December 2017?
Q56. With what is 'Operation Green' related?
Q57. Who is the first woman advocate whose name has been recommended by the collegium to become the Judge of Supreme Court of India?
Q58. What is the bank account available to Dalits and Women under 'Stand-up India?
Q59. Immunization of all children by 2020 is related to:
Q60. According to the Global Nutrition Report 2017, how many females suffered from anaemia in India?
Q61. What is the estimated social sector expenditure in budget 2018-19?
Q62. How many poor families will be provided with how much medical insurance per family in 2018-19 under the Ayushmaan Bharat Yojana?
Q63. How many hours per year are the people expected to contribute for cleanliness under the Swachh Bharat Abhiyaan?
Q64. Match the items given in column I and II:
Q65. In context of environment, the term “dirty dozen” refers to:
Q66. Vast majority of biodiversity occur in :
Q67. Freshwater present on earth is :
Q68. Which industry labours are most prone to Cytosilicosis disease?
Q69. Which of the following is not a fundamental unit?
Q70. The image formed by a convex mirror is :
Q71. The global warming has resulted in:
Q72. Core of a transformer is made of:
Q73. Which of the following is in the deficiency in a body during 'Dengue' fever?
Q74. Who won the Nobel Prize for Chemistry in 2017?
Q75. Why Hambantota Port has been in news in 2017?
Q76. Who said that, “The Constituent Assembly was a one party body in The Assembly was the Congress and the Congress was India”?
Q77. Adi Ambedkar Samaj movement is related to which state?
Q78. When did Communist Party of India (Marxist) emerge a separate political party?
Q79. Who described the critical and creative role of politics in India as 'the Indian model of development'?
Q80. The National Forest Policy was launched in :
Q81. The rate of decrease of temperature in the troposphere is :
Q82. In which case did The Supreme Court give the doctrine of ‘Basic Structure’ of the constitution?
Q83. Which states/union territories are involved in the Cauvery river dispute?
Q84. Supreme Court’s judgment in Indira Sawhney Vs. Union of India case deals with:
Q85. Which of the following is not associated with electoral reforms in India?
Q86. Who described Indian Capitalism as 'Dharmshala Capitalism'?
Q87. Who said, "Indian federalism is a new kind of federation to meet India’s peculiar need’s"?
Q88. Which of the following leaders had worked for tribal upliftment?
Q89. Which of the following leaders was never associated with the Indian National Congress?
Q90. Find the correct poverty estimates for the given states in 2011-12, as per the Planning Commission?
Q91. Which one of the following is not correctly matched?
Q92. Which one of the following igneous rocks has the highest density?
Q93. Which of the following Bills was drafted by Dr. B.R. Ambedkar as the first Law Minister of India?
Q94. Which of the following was not a socialist leader?
Q95. Match List-I and List-II and identify the correct answer from the codes given below:
Q96. Regarding the Interim Government of 1946 which of the following is not correct?
Q97. Which of the following are statutory bodies?
Q98. Match List-I and List-II and select the correct answer from the codes given below:
Q99. 1. Match List-I and List-II and select the correct answer from the codes given below :
Q100. 18. Match the items given in column I and II :
2017 HPAS Solved Paper
Q1. One of the better achievements of Kanha National Park is :
Q2. In India, life expectancy is:
Q3. United Nations has declared 2017 as the International Year of :
Q4. AMRUT was launched on :
Q5. Which of the following instruments is used for measuring the depth of the ocean?
Q6. Where from is litmus dye extracted?
Q7. Which of the following are the primary colors?
Q8. Nail polish remover contains:
Q9. Which of the following is the most promising biodiesel crop?
Q10. Light year is a unit of :
Q11. The most abundant rare gas in the atmosphere:
Q12. From which place does the Spiti River originate?
Spiti River
Q13. Nana Saheb, who fought against the English during the Indian Mutiny of 1857, was:
Q14. Who presided over the first session of Indian National Congress in Bombay?
Q15. Which Socio-Religious Movement of 19th-20th century adopted the practices of occult mysticism?
Q16. Where the first session of Muslim League was held?
Q17. Of the following, who shot dead Curzon Wyllie in London?
Q18. Where was founded the Communist Party of India in 1920?
Q19. Which event earned for Vallabhbhai Patel the title of "Sardar"?
Q20. In which ocean ‘Mindanao Trench’ exists?
Q21. Who is the author of the book "The Unstable Earth"?
Q22. In order to prevent oxidation of potato chips during storage in sealed bags, chip manufacturers seal the packet in the presence of :
Nitrogen in Food Preservation
Q23. Which of the following has a pH of above 10?
Milk of Magnesia
pH Values
Q24. Which of the following processes does not take place during photosynthesis?
Photosynthesis
Q25. If someone is suffering form the problem of acidity after overeating, what is the remedy?
Baking Soda
Q26. Which one of the following is a correct sequencing of the features of the Preamble of India Constitution?
Q27. Which schedule of the Constitution of India contains provisions for the disqualification of a legislator on grounds of defection?
Q28. Who presides over the joint sitting of the Parliament?
Q29. Under which provision can the Speaker allow any member of the House to speak in his/her mother language?
Q30. Power of Judicial Review, provided in the constitution, is exercised on the basis of which principle?
Q31. Which one of the following policies of Government of India is related to the availability of cooking gas to the BPL Households?
Ujjawala Yojana
Q32. HDI is measured with reference to:
The Human Development Index (HDI)
Q33. Which one of the following cities is called 'Silicon Valley of India'?
Q34. Which country owns the Margarita Island which was the venue of 17th NAM Summit?
Q35. Tehmina Janjua is the first woman in Pakistan’s history to hold the office of :
Q36. Who among the following is the Chancellor of Nalanda University (2017)?
Q37. To which country does the UN Secretary General, Antonio Guterres, belong?
Q38. In which field was Juan Manuel Santos awarded the 2016 Nobel Prize?
Q39. Donald Trump is:
Q40. Teacher’s Day is celebrated on:
Q41. Which Raja of Chamba transferred the capital of his state from Bharmaur to Chamba?
Q42. During the reign of which raja of Nurpur princely state the forts of Maukot, Nurpur and Taragarh were captured by the Mughals?
Q43. Which raja of Bushahr princely state was conferred the title of Chhatarpati by the Mughal emperor Aurangzed?
Q44. By what name did the Greek scholars call the Satluj River?
Q45. What is the approximate capacity of Sawra-Kuddu Hydro-Electric Power Project?
Q46. At which place in Shimla District of H.P. is Government Sheep Breeding Farm?
Q47. At which place in Bilaspur District of H.P. a woman Industrial Training Institute (I.T.I) has been made functional during 2015-16?
Bharari Women ITI
Q48. The global agreement that deals with the control of transboundary movements of hazardous wastes and their disposal is called:
Q49. The biological oxygen demand of water is reduced mainly by:
Q50. The use of carbon tax has been very successful to reduce emissions of greenhouse gases in :
Q51. Dead zones in the world’s oceans and large lakes are characterized by:
Q52. Who among the following were conferred Padma Vibhushan Awards in 2017?
2017 Padma Vibhushan Awards
Q53. Which of the following represents the major loss to the human kind due to environmental degradation?
Q54. India ratified the Stockholm Convention for protecting human health and the environment from persistent organic pollutants in :
Q55. The number of wetlands in India listed under Ramsar Convention is :
Q56. In the Budget for 2017-18, the funds allocated for MGNREGA is:
Q57. What was the unemployment rate during 2015-16 based on Usual Principal Status in India?
Q58. Who conducts the periodical sample survey for estimating the poverty line in India?
Q59. For the Budget proposals 2017-18; which of the following is not correct?
Q60. Stalactites and Stalagmites are:
Q61. Coral reefs are found between:
Q62. Savanna region of Brazil is locally called as:
Q63. In "Concentric Zone Model" of Burgess, the fifth ring is occupied by:
Q64. Brahmaputra, after joining Tista river; is known as:
Q65. Match is following and choose the correct answer from the codes given below:
Q66. Which of the following States of India has the highest poverty ratio?
Q67. The growth rate of Himachal Pradesh during 12th Plan (2012-17) has been estimated at:
Q68. As per Census 2011, literacy rate in Himachal Pradesh for male and female respectively is:
Q69. Match the following:
Q70. Supreme Court’s judgement in Indira Sawhney Vs. Union of India is related to which issue?
Q71. Provision for Grant-in-Aid under Article 243 H for the Panchayats has been made out of which source?
Q72. Which of the following statements is correct in reference to the Finance Commission of India?
Q73. Which one of the following is the correct statement with regard to the Emergency Provisions of Indian Constitution?
Q74. Match List1 (Mines) and List2 (Minerals) using the codes given below:
Q75. Who of the following, was not the Moderate leader of Indian National Congress?
Q76. When was the Montague Declaration made about the introduction of responsible government in stages for India?
Q77. Green Revolution made the country self-sufficient in food production but had some fallouts. Consider the following statements:
Q78. Kol tribe is found in :
Q79. Which of the following rivers have been termed as "living entities" by the High Court?
Q80. The NOTA option was first used in the elections in :
Q81. According to the UN Human Development Report (2016), the world’s top three countries in terms of human development are?
Q82. Who among the following has been appointed Director of the Central Bureau of Investigation (2017)?
Q83. The district to which two Chief Ministers of Northern States (2017) belong, falls in the state of :
Q84. The Supreme Court issued a contempt notice to the sitting High Court Judge of:
Q85. The Chief Minister and Deputy Chief Minister(s) of the following states were not members of the State Legislative Assembly at the time of their appointments (2017):
Q86. Which month of 2017 marks the 100th anniversary of Gandhi’s arrival in Champaran?
Q87. Who among the following were awarded Rajiv Gandhi Khel Ratan Award for 2015-16?
Q88. Who among the following was conferred Jnanpith Award in 2016?
Q89. According to legend which God (Devta) killed Banasur who was the ruler of Sarahan area of upper Shimla?
Q90. Who was the first European to draw attention to the history of royal families of Jalandhar and Trigarta?
Q91. Around which year did the Gurkhas capture the Sirmaur princely state?
Q92. Which Himachali revolutionary edited Sudarshan Chakra magazine from Lahore during the pre-independence period?
Q93. What is the number of women members in the present H.P. Vidhan Sabha?
Q94. How much subsidy is given in H.P. to the women of eligible category for providing Gas connection under Mata Shabri Mahila Sashaktikaran Yojana?
Q95. What is the architectural style of Hateshwari Temple at Hatkoti (in Jubbal Tehsil of Shimla District)?
Q96. Which of the following subject matter is not found in the Kangra Kalam style Kangra paintings of eighteenth century?
Q97. In which two Districts of H.P. will the H.P. Forest Ecosystem Climate Proofing Project be implemented?
Q98. According to 2010-11 Agricultural census what is the percentage of small land holding in H.P.?
Q99. Which cement company has been given a prospecting license to carry out a detailed study for proving the quantity of lime-stone deposits in Sugrathi-Thangar-Kura Khera-paulikhera-Kandal–dedera of Tehsil Chopal in Shimla District, H.P.?
Q100. Who is the author of book Kullu and Lahaul?
2015 HPAS Solved Paper
Q1. Which of the following is not a freshwater ecosystem?
Q2. Which of these is not an ecosystem service?
Q3. Desert ecosystem are characterized by annual rainfall of less than
Q4. Which of these is not a factor threatening extinction of several species in India?
Q5. Which of these places is reputed to be world’s most biologically diverse?
Q6. India is to home to how many (approximately) mammalian species?
Q7. According to Paris agreement on climate change, the rise in average temperature of earth’s surface above pre-industrial levels should be limited to:
Q8. In which of the following states, ‘Hornbill’ festival has been observed to uphold Unity in Diversity?
Q9. Which of the following is not correct about the Harappa’s contacts with its contemporary civilizations?
Q10. Who built the Sudarshana lake in the Saurashtra region?
Q11. Which of the following Bhakati poets of Maharashtra conveyed his teachings through the numerous Abhangas?
Q12. Dara Shikoh was the disciple of which sufi saint?
Q13. Who established the East India association in 1866 in London?
Q14. How much time did Lord Mountbatten give to Sir C. Radcliff to delineate the international border of India in 1947?
Q15. India celebrated its first constitution Day on?
Q16. In which year was the Protection of Women from Domestic Violence Act passes?
Q17. Ashish Bose is well known for his concept
Q18. Bombyx mori is a
Q19. Which of the following Indian States will get the first ever solar-stellar observatory?
Q20. Which is not an indicator of Human Development Index?
Q21. Which soil is best for cotton cultivation?
Q22. Bermuda Triangles is located in?
Q23. Which is the longest river of the World?
Q24. When a body is taken from the earth to moon
Q25. Decibel is used to measure the intensity field of
Q26. A pressure cooker works on the principal of
Q27. Sweat mainly serves to eliminate
Q28. In which basin is Upper Joiner hydro-power project?
Q29. The acronym UNFCC represents:
Q30. Identify the correct sequence of countries with the highest biodiversity-
Q31. In the context of climate change, measures undertaken by Government of India have a target of electric power generation by renewable energy (by year 2030) of
Q32. In a poultry unit the factor most influencing the cost is the cost of :
Q33. Which of the following is not the feature of caste system?
Q34. Which state of India was the first to pass Jamindari Abolition Act?
Q35. Which of the following vegetable oils does not contain essential fatty acids?
Q36. Which of the following statements is correct?
Wheat Bran is Rich Source of Fibre
Q37. Vegetables are easily perishable because of their high content of:
Q38. Of which rivers tributaries are Phojal, Sarwati and Hansa streams
Q39. What is the archaeological style of Adi Brahama temple Khokhan?
Q40. With which region of H.P. is Bonangchyu dance associated?
Q41. Who was the court poet of Raja Dharam Chand of Kangra who wrote Dharam Chand
Q42. To which princely state did Zorawar Singh, General of Raja Gulab Singh who invaded Ladakh and Baltistan during 1834-41, belongs ?
Q43. When did the Chandrahabhaga Valley which was held as Jagir by the families of Tibetan origin come under sway of Kullu ?
Q44. The territory of which of the following princely state was divided between two cousins/scions Vijay Singh and Ram Singh during the nineteenth century?
Q45. According to 2011 census which district of H.P. has the highest number of urban households?
Q46. Around which year did captain RC Lee set up an apple orchard in Kullu area?
Q47. Which Raja of Bushahr princely state received Hang-Rang valley from Tibet as Jagir?
Q48. Around which year did Russian painter Nicholas Roerich visit Naggar in Kullu District for the First time?
Q49. Which Raja of Bangahal princely state was treacherously killed by Raja Siddha Sen of Mandi?
Q50. Which school did Amrita Shergil join in Shimla around 1924-25?
Q51. Which Raja of Sirmaur princely state shifted the capital of his state from Rajban to Kalsi in the beginning of the thirteen century?
Q52. At which place in Kullu District of Himachal Pradesh is fish farm?
Q53. Who founded Himachal Pradesh congress (HVC)?
Q54. Which two districts of H.P. are covered under Backward region Grant fund of ministry of Panchayati Raj, Government of India?
Q55. Which bank is collaborating with the Himachal Pradesh government in implementing Doodh Ganga Yojna?
Q56. Which agency is managing Himachal Pradesh state wide Area networks (HIMSWAN) services?
Q57. Who among the following persons has been appointed as UP’s Lokayukta?
Q58. Who among the following persons has secured India’s 12th Olympic quota berth for shooting for the Rio Games?
Q59. The National Airquality Index (AQI) measures air quality by taking into consideration which of the following pollutants?
Q60. Which is incorrect about Himalaya?
Q61. Who of the following has been awarded Padma Vibhushan?
Q62. Consider the following statements regarding IRNSS-1E:
Q63. The Deen Dayal Upadhyaya Gram Jyoti Yojana:
Q64. Who among the following was the constituent advisor to the Constituent Assembly?
Q65. Which one of the following festivals was most famous in Vijayanagar empire ?
Q66. Which of the following states were not annexed to the English East India Company’s dominions under Dalhousies’s Doctrine of Lapse ?
Q67. When was the Civil Disobedience Movement finally called off by Mahatma Gandhi?
Q68. Consider the following:
Q69. With regard to the recently approved "Pradhan Mantri Fasal Bima Yojana"
Q70. What has been the growth rate of GDP of India during 2014-15 at the current prices and constant and constant prices respectively?
Q71. Consider the following statements regarding Corporate Tax Avoidance deal :
Q72. Pradhan Mantri Fasal Bima Yojna was approved by Central Government on:
Q73. Swabhiman is a:
Q74. Which of the following is not a function of Reserve Bank of India?
Q75. As per HDR, 2014 the Gender Inequality Index of India is:
Q76. The 14th Finance Commission has enhanced the states' share in the central divisible pool taxes from:
Q77. As per the Census 2011, the average age is above 30 years for the following states:
Q78. What is the name of shifting cultivation in Mexico ?
Q79. Which one of the following is incorrect?
Q80. Which one of the following is incorrect ?
Q81. Which lines separates India from Afghanistan ?
Q82. Varkala beach is located in :
Q83. Which one of the following is not correctly matched ?
Q84. Which of the following statements is incorrect regarding Zika virus?
Q85. What is the population density (per sq. km) of India and Himachal Pradesh respectively as per Census 2011?
Q86. Which of the following human rights come within the ambit of Article 21 of the Indian Constitution?
Article 21
Q87. During the term of office, criminal proceedings cannot be instituted against:
Q88. Who among the following decides the question of disqualification of a member of Parliament on the grounds of defection?
Q89. Who among the following has the power to declare a caste or a tribe as scheduled caste or scheduled tribe?
Q90. Who among the following determines the area of a Lok Sabha constituency for the purpose of election?
Q91. The Central Information Commission falls under:
Q92. Which of the following statements is correct about NITI Aayog?
Q93. Which of the following committees recommended the creation of 'Nyaya Panchayats'?
Q94. Which of the following are All India Services?
Q95. Who is the Chief Economic Advisor to the Government of India?
Q96. Consider the following statements regarding International Solar Alliance (ISA) :
Q97. Consider the following statements regarding Corruption Perception Index (CPI), 2015 :
Q98. Who among the following persons has won the Man Booker Prize, 2015?
Q99. First Census was conducted in which year in India?
2014 HPAS Solved Paper
Q1. The latitude that passes through Sikkim also passes through
Q2. Which of the following does not contain silver?
Q3. Aluminum surfaces are often ‘anodized’ this means the deposition of a layer of:
Q4. When a CD (compact disc used in audio and video systems) is seen in sunlight, rainbow like colours are seen. This can be explained on the basis of the phenomenon of:
Q5. Domestic electrical wiring is basically a:
Q6. The only snake that builds a nest is:
Q7. The hormone insulin is a:
Q8. The normal temperature of human body in Kelvin scale is:
Kelvin to Celsius Formula
Q9. Which one of the following polymers is widely used for making bullet proof material?
Q10. Name the site where Harappan ploughed field has been found:
Q11. Name the place where Jaina texts were finally complied.
Q12. Which of the following Marathas was given a Manasab of 7000/7000 by Aurangzeb?
Q13. Who among the following translated Kalhana’s Rajatarangini into English?
Q14. Sidhu and Kanhu were associated with which one of the following?
Q15. NITI in NITI Aayog stands for
Q16. Which of the following is/are treated as artificial currency?
Q17. SEBI was established in the year
Q18. The working principal of a washing machine is
Q19. Which of the following drugs reduces fever?
Q20. Mars Orbitor Mission is called:
Q21. In which district of HP is Yunam-so Lake?
Q22. What is Shingni-Mingni?
Q23. According to Planning Commission estimates for 2011-12 which of the following states have maximum percentage of population below poverty line
Q24. Diego Garcia Island is located in
Q25. Great Salt Lake is located in:
Q26. Tropic of cancer does not pass through
Q27. Which is the second most spoken language in India?
Q28. Who among the following is the author of Milind Panho?
Q29. Find out among the following, the author of Muasir-i-Alamgiri?
Q30. Polworth, Corriedole, Blackpace and Bingi are varieties of:
Q31. Who was the head of the boundary commission which demarcated the boundaries of India and Pakistan?
Q32. What is the local name of shifting cultivation in Malaysia?
Q33. When was ‘Beti Hai Anmol’ Yojna launched in Himachal Pradesh?
Q34. Disguised unemployment generally means:
Q35. Which of the following amendments to the Indian Constitution has made the right of education a fundamental right?
Q36. Which of the following are the exclusive committees of Rajya Sabha?
Q37. The elected members of the state legislative assembly participate in the elections of
Q38. Which of the Commissions/Committees has recommended that there should be Minister of Panchayati Raj?
Q39. Pradhan Mantri Jan Dhan Yojna was launched on
Q40. Which of the following has the lowest sex ratio in India?
Q41. Which of the following statements is/are correct ?
Q42. Which of the following is the largest coal producer?
Q43. Which of the following is the correct order of rivers from north to south?
Q44. Which Mughal ruler is said to have given the title of ‘Mian’ to the Rajput princes of Punjab Hill states who were held hostages in the royal court?
Q45. Ruler of which of the following hill states were from a common ancestor?
Q46. Who started the Ice-Skating Rink at Shimla around 1920?
Q47. What was the annual growth rate in the economy of Himahcal Pradesh during the 2012-13 fiscal?
Q48. Which one of the following is a Pagoda style temple in HP?
Q49. Identify the correct order of density of population among the following districts of HP according to 2011 census data (in descending
Q50. Which district of HP occupies top position in the production of ginger?
Q51. India signed an agreement on transfer of sentenced persons with the Hong-Kong special administrative region in 2014.
Q52. Based upon the relations between India and Pakistan, Khurshid M. Kasuri, the Ex-foreign Minister of Pakistan has recently penned down a book. What is the title of the book?
Q53. Which of the following statements are correct about the National Human Right Commission?
Q54. Which of the following agriculture practices is eco-friendly?
Q55. According to the Wealth Health Organization, the desease which causes the death of the largest number of people today is
Q56. The Raja of Bushahar princely state was assisted by three hereditary Wazir families. One of them was Kohal. Where did it come from?
Q57. When was the Department of Public Relations created in Himachal Pradesh?
Q58. In India, which of the following have the highest share in the disbursement of credit to agriculture and allied activities?
Q59. The World Investment Report (WIR) is published by
Q60. What is the correct sequence of the following four stages in the public policy formulation process?
Q61. Annual Summit 2015 of BRICS nations was held in
Q62. The most widely used antacid is:
Q63. In which district of HP is Saryanj Sarma watershed project?
Q64. A simple machine helps a person in doing:
Q65. Which Raja of Spiti invaded Kullu and made it a tributary?
Q66. As per the IPCC 4th Assessment Report, the average sea level during the period 1961-2003 increased approximately at the rate of
Q67. Who abolished Dual Government in Bengal?
Q68. The temples located in the region between the Vindhyas and the Krishna river are known as:
Q69. Which one of the following pairs is not correctly matched?
Q70. According to Dr. B. R. Ambedkar, which of the following articles of the Indian Constitution was to remain a dead-letter?
Q71. The responsibility of law and order rests with the police commissioner in the cities of
Q72. Which one of the following functions does not figure in the 11th schedule of the constitution (73rd Amendment)?
Q73. Driving force for an eco-system is
Q74. Read the following statements regarding the recently released NTCA report on the status of tigers in India, 2014.
Q75. At present, the primary energy source in the world is
Q76. Which of the following has the maximum bio-diversity?
Q77. World Ozone day is celebrated on
Q78. In India, the forest cover percentage of the total geographical area of the country is appox.
Q79. A fuse is used in main electric supply as safety devices. Which one of the following statements about the fuse is correct?
Q80. What is the script of the earliest epigraphical rock inscriptions at Pathyar and Kanhiyara in Kangra?
Q81. How many biosphere reserves are there in India?
Q82. Earth Summit 2012 for sustainable development was held in
Q83. The sensation of fatigue in the muscle after prolonged strenuous physical work is caused by:
Q84. The UN Convention on Biodiversity was ratified by the Indian government in the year
Q85. National Sample Survey Office was established in the year :
Q86. Match the List I with List II
Q87. Recently, the government of India has decided
Q88. Modi Government re-launched the Kisan Vikas Patra (KVP) investment scheme to top house hold saving for funding infrastructure development in the country.
Q89. Shanta Kumar Committee has submitted its report on restructuring of Food Corporation of India (FCI) to Mr. Modi.
Q90. As per the population census 2011, what is the percentage of urban population in Himachal and India respectively
Q91. Which one of the following does a TV remote control unit to use to operate a TV set?
Q92. Which Raja of Mandi gave the Shivratri Fair the form of a cultural festival?
Q93. Which of the following peaks are in Himachal Pradesh?
Q94. Find the mismatch in the following pairs:
Q95. When did the number of members in the HP Vidhan Sabha increase from 36 to 41?
Q96. Almost all major oil produces including Saudi Arabian have become dependent on high prices, with break-even level usually over $90 per barrel to meet their fiscal obligations. Consider the following can be termed as explanation for the present decline in oil prices?
Q97. The world’s largest ground based telescopic observatory is located in:
Q98. Apart from Carbon-dioxide, which of the following gases also have a role in climate change?
Q99. At present, which country is the maximum emitter of Carbon-dioxide?
Q100. What is the tenure of Chief Election Commissioner of India?
2013 HPAS Solved Paper
Q1. What is the name of Pakistani Bus that plies between Lahore and Delhi?
Q2. Standard 18-carat gold sold in the market contains
Q3. Which of the following is represented by the Harrapan Civilization?
Q4. In which of the following year the Asiatic Society of Bengal was established?
Q5. Who is Shinzo Abe?
Q6. The major component of honey is
Q7. With which district of Himachal Pradesh is Kayang folk dance mainly associated?
Q8. Which lake is jointly owned by China and India?
Q9. "There is enough for everybody’s need but not enough for everybody’s greed." Who said it?
Q10. Which one of the following is not trade block?
Q11. Climatology is the science of:
Q12. Zulus tribe is found in?
Q13. Which one of the following is the largest producer of coffee in the world?
Q14. Which is the tallest dam of India?
Q15. Which of the following rivers is called ‘Nadittam’ in Rigveda?
Q16. The officer who enjoyed authority over the large pasture ground was known as during the Rigvedic period?
Q17. Usually fuels on burning causes pollution. which one of the following fuel causes minimum environmental pollution ?
Q18. What is the average fat content of "Buffalo milk"?
Q19. On the bank of which river is Nadaun town of Hamirpur District?
Q20. Which of the following is the author of Tamil Ramayana?
Q21. Jhum is a term used for:
Q22. When ants bite, they inject:
Q23. The 'Stones' formed in human kidney consist mostly of:
Q24. Which tehsil of Mahasu district was carved out to form Kinnaur district?
Q25. For which work did Mridula Garg get the 2013 Sahitya Academy Award?
Q26. According to Transparency Intentional which are the most honest countries in the world in 2012-13?
Q27. Who was crowned as Miss World in 2013 beauty contest?
Q28. The UN Human Rights Council (HRC) was set up in:
Q29. Which day is observed as Anti-Leprosy Day?
Q30. Arteries supplying blood to the:
Q31. Cryogenic engines find applications in:
Q32. Which of the following is not in the vicinity of Kullu?
Q33. In which town is Panchvaktra temple?
Q34. Ultimate source of energy for living being is:
Q35. Soil erosion can be prevented by:
Q36. With which game/sport is Deepika Kumari associated?
Q37. When was Telegraph/Telegram service closed in India?
Q38. Fat present below the skin surface in our body, acts as a barrier against:
Q39. In which year publication of Hindi daily Divya Himachal begin?
Q40. Who was the first person to vote in the First General Elections in India in 1951-52?
Q41. Which of the following has the highest sex ratio as per 2011 census?
Q42. Which Indian State has the lowest death rate according to census 2011?
Q43. Which one of the following is located in Atlantic Ocean?
Q44. Diffusion of light in the atmosphere takes place due to:
Q45. The characteristics odour of garlic is due to:
Q46. Which lake at Mandi is known for floating island?
Q47. Which of the following are the ways of acquiring citizenship of India?
Naturalization
Q48. The Right to Freedom under Article 19 of the Constitution of India may be suspended by President of India?
National emergency under article 352
Q49. Which of the following languages have been declared by the Constitution to be the languages for conducting business in Parliament?
Q50. Which is the correct measure of Infant Mortality Rate (IMR)?
infant mortality rate
Q51. After which American scientist is ‘h-index’ named?
h-index
Q52. NABARD came into existence in the year:
Q53. Who is George Alexander Louis?
Q54. Which of the following is believed to be the oldest among Shimla Hill States?
Q55. Which one of the following is taken as the Base Year for estimating GDP at constant prices in India during 2012-13?
Q56. "What was the average annual growth rate of Agricultural and Allied Sectors in India during 11th five year plan?
Q57. Which of the statements given below is/are correct?
Q58. The Targeted Public Distribution System (TDPS) was adopted in the year:
Q59. What were the main findings of the N.N. Vohra Committee?
Q60. Who among the following increased the Mansab from 5000/- to 7000/-?
Q61. Which of the following political parties was called ‘Rajadroha Ka Karkhana’ by the British?
Q62. Who among the following established the Central Hindu School at Banaras?
Q63. Rani Gidalu was one of the freedom fighter from the north-east. She hailed from one of the following states
Q64. Which of the following states did not return Congress in power after the first provincial elections in 1937?
Q65. Match names of List I with that of List II and select the correct answer from the codes given below:
Q66. Which one of the following is incorrect?
Q67. Which is not temperate grassland?
Q68. In which of the following rivers ‘Swargadwari’ is located at?
Q69. Which one of the following is incorrect?
Q70. Which is the correct descending order in terms of length (km) of river?
Q71. Which of the following pairs is not correctly matched?
Q72. What is India’s Global Position out of 187 countries as per UNDP’s Human Development Report 2011?
Q73. Who is the Chairman of Commission for Agricultural Costs and Prices (CACP) at present?
Q74. An air bubble in water will act like a:
Q75. Which one of the following is not an essential micronutrient for plants?
Q76. Identify the correct chronological order in which the following Europeans travelled in Punjab and Shimla Hill states?
Q77. Who negotiated the arrangement between the states of Kashmir and Chamba by which the later became independent of the former?
Q78. Raja of which state was treated by the wife of Dara Shikoh as her son?
Q79. When did the consultative body of ruler of princely states known as Narendra Mandal came into existence?
Q80. Who led the famous Mandi conspiracy of 1914-15?
Q81. According to Annual Status Education Report (ASER) 2012-13 what percentage of class V students of Himachal Pradesh Government and private schools can read only class II text?
Q82. What was the exact constitutional status of the Indian Republic on 26th January 1950?
Q83. Disagreement between the two Houses of Parliament on the following bills can be resolved by the Houses in a joint sitting
Q84. In a National Park, protection is provided to:
Q85. Which of the following union territories do not have any representative in the Rajya Sabha?
Q86. Which among the following committees was the first to demand constitutional recognition for Panchayats?
Q87. In which of the following states have the Panchayati Raj institutions been set up in conformity with the traditions and customs of the local people?
Q88. Who among the following former Presidents contested the election to the office of the President of India without resigning from the office of Vice-President of India?
Q89. Which one of the following models/theories views public policy as a continuation of previous government activities with some modifications?
Q90. The Khadi and Village Industries Commission was established during which of the Five Year Plans?
Q91. In which field did Frederick Sainger win Nobel Prize twice?
Frederick Sainger
Q92. Which of the following types of glasses can cut-off ultraviolet rays?
Q93. Which one of the following is an active component of clove oil?
Q94. In eye donation, which one of the following parts of donor’s eye is utilized?
Cornea
Q95. Which action of Raja Sansar Chand prompted the Hill Chiefs to invite Gurkhas to invade Kangra?
Q96. When was a Sanad granted to the Raja of Bilaspur by the British government confirming his possession on the right bank of Satluj?
Q97. Court poet of which princely state of Shimla Hills is the author of Shashi Vansh Vinod?
Q98. Which mountain pass connects Spiti to Ladakh?
Q99. With which of the following is NGO named Vishakha is associated?
2012 HPAS Solved Paper
Q1. At which place in Shimla District is Herbal garden?
Q2. Koderma, a place in Jharkhand , is famous for
Q3. What is Curiosity Rover?
Q4. The continental drift theory was propounded by
Q5. What are stalagmites?
Q6. With which of the following is Losar festival associated?
Q7. According to legend what is the vehicle of Kamdev?
Q8. The drugs used to get relief from pain are called:
Q9. Metals are good conductors of electricity because
Q10. With which game is Ravi Rampaul associated?
Q11. Against whom did Aurangzeb fight the battle at Samugarh?
Battle of Samugarh
Q12. Who was Amrita Shergil?
Q13. In which year Shimla was declared as the summer capital of British India?
Q14. Which of the following states Nathpa Jhakri Hydel Project under execution
Q15. Which country is called the 'Sugar Bowl' of the world?
Q16. Who was the first Caliph of Islam ?
Q17. What is INS Chakra?
Q18. In coming years , skin related disorders will become more common due to :
Q19. In the process of galvanization of an iron sheet is coated with:
Q20. One liter of water is equivalent to:
Q21. A transformer is used for:
Q22. Where is KEE monastry?
Q23. Which type of pollution is caused by two - wheelers?
Q24. Chernobyl disaster was caused by a :
Q25. First share market in India was established in :
Q26. Who founded the revolutionary organization named 'Abhinava Bharat' ?
Q27. At what interval is Bhunda festival of Nirmand held?
Q28. Who is the author of 'Abode of God'?
Q29. Which of the following is not a tributary of the Beas?
Q30. With which region of Himachal Pardesh is Sih ballet associated?
Q31. Which Sultan of Delhi helped Raja Megh Chand of Bilaspur to regain his lost kingdom?
Q32. Which Raja of Mandi built the Darbar Hall in 1906?
Q33. Who won the 2012 Himachal Pardesh Vidhan Sabha election by the highest margin?
Q34. Youth of which age group are eligible to avail the skill development allowance announced by the Himachal Pradesh Government in 2013-14 budget?
Q35. Match the following proposed rope-ways correctly and choose the correct answer from the codes given below:
Q36. According to 2008-09 data, which district of Himachal Pradesh has recorded the highest percentage in the production of pulses?
Q37. Which one of the following is not a precondition for implementing national programme for education of girls at the elementary level?
Q38. Which is scheme has been introduced in place of Kishori Shakti Yojana in some district of Himachal Pradesh?
Q39. Which sector had highest outlay (in % terms ) in the annual plan of 2011-2012?
Q40. Beside Kinnaur and Lahaul Spiti, which district of Himachal Pradesh has recorded the lowest percentage of urban population in the 2011 census?
Q41. What was the percentage of female literacy in Himachal Pradesh in 1951?
Q42. Match the following:
Q43. Whom did emperor Ashoka send to Lanka to promote Buddhism?
Q44. There are four Upvedas. Three of them are Ayurveda, Dhanaurveda and Sthapatya veda. Which is the fourth?
Q45. Which among the following scholar belong to the later Vedic period?
Q46. Whom did the Huns defeat to establish their in India?
Q47. The first Rani of Mewar who committed johar was Padmini. Who was the second?
Q48. At which place did the battle between Babur and Rana Sanga take place 1527 ?
Q49. Who organized the Red Shirts Movement ?
Q50. Who is associated with the moderate strand of the Khilafat movement of 1919-20?
Q51. The marked of Moshad the spy agency of Israel is that it has as its member a large number of
Q52. What is the major religion of Myanmar?
Q53. With which field is Malala Yousafzai associated?
Q54. Identify the country whose seat of legislature, executive and judiciary is at three different cities?
Q55. Which one of the following refused the Nobal Prize awarded to him?
Q56. When was the World Anti-Doping Agency(WADA) set up?
Q57. Which of the following states records the highest proportion of scheduled castes population ?
Q58. The most ideal region for the cultivation of cotton in India is
Q59. Which of the following factors is responsible for the rapid growth of sugar production in south India as compared to North India?
Q60. Which of the following state groups is the largest producer of iron-ore in india?
Q61. Whose government was defeated for the first time on a vote of confidence?
Q62. The Question Hour in the House of the people is followed by
Q63. CENVAT is assoicated with :
Q64. The 73rd and 74th constitutional Amendments were during the primeministership of:
Q65. The Indian Constitution refers to minorities based on
Q66. Which Indian company has been included for the first time in USA’s index NASDAQ-100?
Q67. UNDP has introduced a new poverty index known as:
Q68. The excess discharge of fertilizers into water bodies result in:
Q69. To which community/organisation maximum goods are exported from India?
Q70. The antiknock agent added to unleaded petrol is:
Q71. 3-4 benzopyrene causes
Q72. Natural silk is a :
Q73. Which of the following states have recently (2013) got separate high courts?
Q74. Most hazardous metal pollution of automobile exhaust is:
Q75. If BOD of a water is found very high, this means water:
Q76. Minamata disease affects the :
Q77. Chemical which cause bone cancer and degeneration of tissue is :
Q78. Who was the Chairman of the committee on pricing and taxation of petroleum products(2013)?
Q79. When was World Trade Organization set up by the member countries of the United Nations to promote trade among countries?
Q80. Which of the following commissions/committees have recommended legal protection to whistle blowers?
Q81. For which of the following posts has the Supreme court directed the central government that not only civil servants but other persons with impeccable integrity should also be considered?
Q82. Which of the following occupied the highest place in growth rate during the 11th Plan period 2007-2012?
Q83. Which of the following can be remolded time and again without producing any change?
Q84. Most commonly used tranquillizers are derivatives of:
Q85. At what capacity was P.C. Mahalanobis associated with the formulation of the economic policy of independent India ?
Q86. The range of normal human hearing lies between:
2011 HPAS Solved Paper
Q1. Who was the first Indian to declare that "Freedom is my birth right"?
Q2. At which place was Vardhman Mahavir born?
Q3. Zawar mines are important for:
Q4. Which Mughal ruler imprisoned his stepmother after the death of his father?
Q5. Who founded the All India Depressed Classes Federation?
Q6. From which place did Mahatma Gandhi ji started Dandi March to the sea?
Q7. Which of the following is not a measures of Human Development Index?
Q8. At which place in Mandi District has the Department of Ayurveda set-up herbal garden?
Q9. In which district of Himachal Pradesh is Mulgun valley?
Q10. Which statement is true about Ellora caves?
Q11. Who was the founder of Arya Samaj?
Q12. Under the constitution of India, the power to issue a writ of Habeas Corpus is vested in:
Q13. On which river is Chamba Pattan Bridge which being without pillars is first of this kind in Himachal Pradesh?
Q14. Participatory Notes (PNs) are associated with which one of the following?
Q15. Silicon carbide(SiC) is known as:
Q16. Where was the capital of Kullu in the earlier times?
Q17. In the coast of which state is Wheeler Island?
Q18. Five rings linked together constitute the symbol of Olympic Games. Each ring is of a different color. Two colors are blue and black. Which are the other three colors?
Q19. Who translated the collection of South Indian tales into Sanskrit and titled the work as ‘Kathasaritasager’?
Kathasaritasager (कथासरित्सागर)
Q20. Arrange the following ruling dynasties of ancient India in correct chronological order:
Q21. In which year was the Partition of Bengal annulled?
Partition of Bengal
Meaning of Annulled
Q22. Haem is the important biologically in the myoglobin which is used to store:
Haem
Q23. Which treaty signaled the departure of the Gurkhas and arrival of the British on the political canvas of Shimla Hill states?
Treaty of Sagauli
Q24. Mediterranean agriculture is:
Mediterranean agriculture
Crops in Mediterranean agriculture
Q25. Which city in India is called the Garden city?
Q26. Koyali is:
Gujarat Refinery (Located at Koyali in Gujarat)
Q27. Which group of rivers originates from the Himachal mountains?
Q28. The Public Accounts Committee presents its report to:
Public Accounts Committe
Q29. The Governor has the power to impose Governor’s rule in the State of:
Q30. Which Indian city has the highest population according to 2011 Census?
Q31. ‘Nirmal Bharat Abhiyan Yojna’ is associated with:
Nirmal Bharat Abhiyan
Q32. Fly ash is environment pollutant produced by:
Fly ash
Q33. What is the capital of Republic of South Sudan?
Juba
Q34. What is the name of first Indian Research Station in the Arctic?
Himadri Station
Q35. Who has been given the 2011 Sahitya Academy Award?
Sahitya Academy Award
Q36. With which activity is Magsaysay awardees Rajendra Singh associated?
Dr. Rajendra Singh
Q37. After Subhash Chandra Bose resigned from the President ship of Congress in 1939 who became the party president?
Q38. Who were the two prominent leaders of Home Rule League?
Home Rule Movement (1916 - 1918)
Major Leaders
Q39. Environment planning organization is:
NEERI
Q40. Shallow lake with rich organic products are called:
Eutrophic
Q41. Which of the following plans is meant for constructing houses for rural people?
Q42. In 1943, Muslim League ministers were installed in four provinces. Two of them were Sind and NWFP. Which were the other two?
Q43. At which venue did the 1907 A.D. split in Congress Party take place?
Q44. Tarapore Committee was associated with which one of the following?
Q45. The Western Ghats and the Eastern Ghats meet at:
Q46. Rusting of iron is:
Q47. The urine sample of a diabetic patient contains
Q48. The quality of diesel oil for use in diesel engines is determined by:
Q49. Which of the following is used as moderators in nuclear reactors?
Q50. The 40 years war between ancient Aryan king Divodas and Shamber resulted in the defeat of:
Q51. Who built the Lakshmi Narayan Temple at Chamba?
Q52. Which of the following is not a natural polymer?
Q53. When mild steel is heated to a high temprature and suddenly cooled in water, it becomes hard and brittle. The process is called:
Q54. Which Raja’s wooing of Nokhu Gaddan is depicted in the "Raja-Gaddan" love lyric?
Q55. For distribution of essential commodities all families in Himachal Pradesh have been divided into four categories. Two of them are: Below Poverty Line (BPL), and Above Poverty Line (APL) families. Which are the other two are?
Q56. Dr. Piyush Guleri’s award winning work, "Chhaunte" is a …………………….
Q57. According to 2011 census which district of Himachal Pradesh has the lowest female literacy rate?
Q58. On which issue did several Panchayats of Nichar Sub-division boycotted the December 2010-January 2011 Panchayat election?
Q59. Almost 90% of the world’s annual fish supply comes form:
Q60. A polymer which is used for making ropes and carpet fibres is:
Q61. The bleaching action of chlorine occurs in the presence of:
Q62. The maximum limit of Foreign Direct Investment (FDI) in public sector banking is
Q63. The Pennines (Europe), Appalachians (America) and the Aravallis (India) are examples of:
Q64. Three important rivers of the Indian Subcontinent have their sources near Mansarovar Lake in the Great Himalayas. These rivers are:
Q65. Which one of the following is no longer a source of Income of urban local bodies in Punjab, Haryana and Himachal Pradesh?
Q66. As per 13th Finance Commission Recommendations during 2010-15, transfers to the states in the form of ‘grant-in-aid’ are
Q67. The Production of most metallic minerals is affected by each of the following, except:
2010 HPAS Solved Paper
Q1. The definition of Biosphere is:
Q2. Rhododendron is the characteristic vegetation of:
Q3. Some reliable indicators of pollutions are:
Q4. Gas released during Bhopal gas tragedy was:
Q5. Human eye is sensitive to the wavelength in the range of:
Q6. Who built the old fort (Purana Qila) in Delhi?
Q7. Under the Constitution of India who is the guardian of fundamental rights?
Q8. MNREGA programme is related to:
Q9. Which one among the following countries is not a member of SAARC?
Q10. What is the roughly ratio between the population density of Himachal Pradesh and India as per the provisional figures of 2011 Census of India?
Q11. Asbestos is no longer preferred for commercial use because:
Q12. Hemorrhagic dengue fever is spread by:
Q13. In which district of Himachal Pradesh is Uhal Hydroelectric Project located?
Q14. Who was the founder of Satyashodhak Samaj?
Q15. Where was the Headquarters of Ghadar Party?
Q16. Which part of the world has a high density of organisms?
Q17. Carbon Dioxide absorbs strongly in Infrared region and its presence in atmosphere decreases the loss of heat from Earth by radiations. This is called?
Q18. During whose viceroyalty was the Vernacular Press Act enacted?
Q19. In which district of Himachal Pradesh is Surajtal Lake ?
Q20. Freon causes great environmental damage. They are :
Q21. What is an archipelago?
Q22. Siul stream is a tributary of river?
Q23. Balwant Rai Mehta Committee is related to:
Q24. Which term is used for imaginary lines joining the places of equal rainfall?
Q25. Freons cause great environmental damage. They are:
Q26. What is relevant to the Golden Quadrilateral in India?
Q27. Which of the following is not a plant hormone?
Q28. What is the silicon valley of United States of America famous for?
Q29. What is the density of population at national level in India according to 2011 census?
Q30. Which mountain range separates Sirmaur from Shimla?
Q31. Who amongst the following can be the chairman of National Human Rights Commission?
NHRC (National Human Rights Commission)
Chairman of NHRC
2016 Chairperson
Q32. Antibodies are found in:
B Lymphocytes
Q33. Night Blindness is caused by the deficiency of:
Night blindness (nyctalopia)
Causes of Night Blindless
Q34. India is at which state of the following stages of Demographic transition?
Q35. Consider the following related to Sarva Shiksha Abhiyan(SSA) being implemented in partnership with the States for addressing the needs of children:
Q36. Which one of the following led to the Malogaon Committee to recommend for interest rate cap for micto-finance institutions?
Q37. Consider the following in relation with Corporate tax:
Q38. Consider the following taxes:
Q39. Which of the following is the highest export-oriented handicrafts (in value terms) in India exports?
Q40. Which one of the following represents at least in part investment in human capital?
Q41. Which one of the following treaties was not covered by Uruguay Round Negotiations, which established WTO?
Q42. Who founded the Guler state?
Q43. On the basin of which river is Dodra Kawar situated?
Q44. In which year was ‘Swarna Jayanti Gram Swarozgar Yojna’ launched in Himachal Pradesh?
Q45. 'Mukhya Mantri Bal Udhar Yojna’ of Himachal Pradesh Government is restricted to the children belonging to:
Q46. What is the architectural style of Jawalamukhi temple in Kangra District?
Q47. Male literacy in the recent Census 2011 has increased to:
Q48. Indian agriculture has the following features:
Q49. Which one of the following is non-toxic?
Q50. Carbohydrates are not used in:
Q51. Nanotechnology is where particles of the following size are used:
Q52. Carbon dioxide absorbs the infrared radiations and its presence in the atmosphere decreases the loss of heat from earth by radiations.
Q53. Which trench is located in the Pacific Ocean?
Q54. In which Indian State is the Tawang Buddhist Monastery located?
Q55. For which of the following functions was V.K. Shunglu Committee constituted?
Q56. Who among the following was the speaker of first Lok Sabha?
Q57. Which ancient Indian king wrote the three dramas, i.e. Ratnavali, Priyadarshika and Nagananda?
Q58. By which act was system of ‘Dyarchy’ introduced during British rule in India?
Q59. Who was the Chairman of the first Union Commission of Backward Classes of India?
Q60. According to 2011 Census which of the following districts of the state of the state of Haryana has the lowest sex ratio?
Q61. Which of the following countries is divided into the largest number of time zones?
Q62. In which form is phosphorus predominately found in soil?
Q63. Which of the following properties is not related to LASER?
Q64. Who amongst the following is empowered to declare emergency under the Indian Constitution?
Q65. Under which of the following circumstances can an emergency be declared in India?
Q66. Which of the following is not properly matched?
Q67. Consider the following in the context of food inflation in India:
Q68. Who commanded the Mughal forces which subdued Kangra fort in 1620 A.D.?
Q69. With whose assistance is the Himachal Pradesh Government’s ‘organic farming and water harvesting project’ being implemented?
Q70. Asbestos is no longer used for commercial use because:
Q71. In which one of the following cases has the Preamble been accepted as a part of the Indian Constitution?

Citation preview

HPPSC HPAS General Studies Solved Papers Shunya Foundation [email protected] 2018-08-01

******ebook converter DEMO Watermarks*******

2018 HPAS Solved Paper This PDF Booklet contains 2018 HPAS Solved Paper. Explanation is provided by Shunya Foundation Team. 2018 HPAS Solved Paper for General Studies held on 24 June, 2018

******ebook converter DEMO Watermarks*******

Q1. Oil refinery at Bhatinda is being established by: Choose an option: (1). BP (2). HPCL (3). CRL (4). IOC Correct option : 2 Oil refinery at Bhatinda is being established by HPCL.

******ebook converter DEMO Watermarks*******

Q2. Tata Iron and Steel Company at Jamshedpur was established in: Choose an option: (1). 1914 (2). 1912 (3). 1910 (4). 1907 Correct option : 4 Tata Iron and Steel Company was founded by Jamshedji Tata. It was established by Dorabji Tata on 26 August 1907. By 1939 it operated the largest steel plant in the British Empire.

******ebook converter DEMO Watermarks*******

Q3. The term Operation Flood is associated with : Choose an option: (1). Blue Revolution (2). Golden Revolution (3). White Revolution (4). Yellow Revolution Correct option : 3 Operation Flood was started by National Dairy Development Board (NDDB) in 1970s. The result was that India became the largest producer of Milk and Milk Products. Operation flood is called White Revolution of India.

******ebook converter DEMO Watermarks*******

Q4. B.R. Ambedkar was born at : Choose an option: (1). Poona (2). Bombay (3). Mhow (4). Satara Correct option : 3 B.R. Ambedkar was born at Mhow. It is located in the Indore district in Madhya Pradesh. The town was renamed as Dr Ambedkar Nagar in 2003, by the Government of Madhya Pradesh.

******ebook converter DEMO Watermarks*******

Q5. Montague-Chelmsford Reforms were introduced in India in the year : Choose an option: (1). 1881 (2). 1902 (3). 1909 (4). 1919 Correct option : 4 The Montagu–Chelmsford Reforms were reforms introduced by the British Government in India to introduce selfgoverning institutions gradually to India. Montague-Chelmsford Reforms were introduced in 1919.

******ebook converter DEMO Watermarks*******

Q6. The first industrial policy was announced in: Choose an option: (1). 1948 (2). 1956 (3). 1964 (4). 1972 Correct option : 1 The Government of India announced its first industrial policy resolution on 6 April,1948. The policy resolution laid stress on the role of the state in the development of industry.

******ebook converter DEMO Watermarks*******

Q7. Shingo-La is the name of which pass? Choose an option: (1). Zanskar Pass (2). Bara Lacha Pass (3). Rohtang Pass (4). Jalori Pass Correct option : 1 Shingo-la is on the border between Jammu and Kashmir and Himachal Pradesh. It is at an elevation of 5.091m (16,703ft) above the sea level.

******ebook converter DEMO Watermarks*******

Q8. ‘Madho Rai ki Jaleb’ is the main attraction of which popular Himachali fair? Choose an option: (1). Dussehra (2). Lohri (3). Shivratri (4). Nalwari Correct option : 3 Madho Rai is one of the many names of Sri Krishna. The Grand Procession of Madho Rai along with the Deities (called Jaleb) is an awe-inspiring experience in Shivratri fair at Mandi.

******ebook converter DEMO Watermarks*******

Q9. What is the architectural style of the temple built on the edge of the Prashar Lake? Choose an option: (1). Cone Shaped (2). Pagoda (3). Pentroof (4). Pyramidal Shaped Correct option : 2

Fig. Prashar Lake The sage Prashar is supposed to have meditated at the bank of Prashar lake and the lake houses three-tiered Pagoda like temple dedicated to sage Prashar. The whole of the temple is built up of a single Deodar tree Bansen the king of Mandi got this temple built in the 13th century.

******ebook converter DEMO Watermarks*******

Q10. In which district Central Government approved Rs. 80 lakh in 2011 for setting up of Snow Leopard centre? Choose an option: (1). Kilba in Kinnaur (2). Kibber in Lahaul-Spiti (3). Sainj in Kullu (4). Pangi in Chamba Correct option : 2 Central Government has approved Rs. 80 lakh in 2011 for setting up of Snow Leopard centre at Kibber In Lahul & Spiti distt.

******ebook converter DEMO Watermarks*******

Q11. Who is the author of the book ‘Polyandry in the Himalayas’? Choose an option: (1). Dr Y.S. Parmar (2). C.G. Bruce (3). V.P. Menon (4). M.S. Gill Correct option : 1 The author of the book ‘Polyandry in the Himalayas’ is Dr Y.S. Parmar.

******ebook converter DEMO Watermarks*******

Q12. The term 'Outwash plain' is associated with : Choose an option: (1). Glaciers (2). Rivers (3). Sea waves (4). Wind action Correct option : 1 An outwash plain is a plain formed of glacial sediments deposited by meltwater outwash at the terminus of a glacier. As it flows, the glacier grinds the underlying rock surface and carries the debris along.

******ebook converter DEMO Watermarks*******

Q13. The concept of Normal Cycle of Erosion was given by : Choose an option: (1). Davis, W.M. (2). Penck, W. (3). Strahler, A. (4). Thornbury, W.D. Correct option : 1 W.M. Davis developed a theory of landform creation and erosion. This theory is more commonly known as the "cycle of erosion. His theory explained that mountains and landforms are created, mature, and then become old.

******ebook converter DEMO Watermarks*******

Q14. The term 'Ria' stands for : Choose an option: (1). A submerged river valley (2). A submerged glacial valley (3). An emerged river valley (4). An emerged glacial valley Correct option : 1 A ria is a coastal inlet formed by the partial submergence of an unglaciated river valley. It is a submerged river valley. It is a drowned river valley that remains open to the sea.

******ebook converter DEMO Watermarks*******

Q15. Who became first female President of Singapore? Choose an option: (1). Carrie Lam (2). Halimah Yacob (3). Jacindra Ardern (4). Angela Merkel Correct option : 2 Halimah Yacob is first female President of Singapore. She was the ninth Speaker of Parliament, from January 2013 to August 2017.

******ebook converter DEMO Watermarks*******

Q16. Why Jagmeet Singh is in news? Choose an option: (1). He is elected leader of Canada’s New Democratic Party. (2). He represents South Hall in British House of Commons. (3). He is appointed as one of the advisor to President Donald Trump. (4). He is advisor to UN Commission on Human Rights. Correct option : 1 Jagmeet Singh is elected leader of Canada’s New Democratic Party.

******ebook converter DEMO Watermarks*******

Q17. Who is appointed as New Chairman of ISRO? Choose an option: (1). G.K. Menon (2). Roddam Narasimhan (3). R. Rao (4). K. Sivan Correct option : 4 Dr K. Sivan is the chairman of the Indian Space Research Organization(ISRO). He is the former Director of Vikram Sarabhai Space Center and Liquid Propulsion centre.

******ebook converter DEMO Watermarks*******

Q18. Renewable source of energy is : Choose an option: (1). Coal (2). Fossil fuels (3). Natural gas (4). Sun Correct option : 4 Renewable energy is energy that is generated from natural processes that are continuously replenished. This includes sunlight, geothermal heat, wind, tides, water, and various forms of biomass.

******ebook converter DEMO Watermarks*******

Q19. Photosynthesis occurs in : Choose an option: (1). Nucleus (2). Mitochondria (3). Chloroplast (4). None of the above Correct option : 3 Photosynthesis takes place inside plant cells in chloroplasts. Chloroplasts contain a green substance called chlorophyll.

******ebook converter DEMO Watermarks*******

Q20. In which part of our body RBC formed? Choose an option: (1). Liver (2). Heart (3). Bone-marrow (4). Kidney Correct option : 3 Red blood cells, most white blood cells, and platelets are produced in the bone marrow. Bomne marrow is the soft fatty tissue inside bone cavities.

******ebook converter DEMO Watermarks*******

Q21. In which year 'Project Crocodile' was launched in India? Choose an option: (1). 1973 (2). 1975 (3). 1972 (4). 1980 Correct option : 2 Project Crocodile was launched in 1975 with support from the United Nations Development Programme and Food and Agriculture Organisation.

******ebook converter DEMO Watermarks*******

Q22. 'Rataundhi' is due to deficiency of which Vitamin? Choose an option: (1). Vitamin A (2). Vitamin B6 (3). Vitamin C (4). Vitamin B12 Correct option : 1 'Rataundhi' (Nightblindness) is due to deficiency of Vitamin A.

******ebook converter DEMO Watermarks*******

Q23. 'Dohru' is a traditional woollen dress worn by the women of which district? Choose an option: (1). Una (2). Kangra (3). Bilaspur (4). Kinnaur Correct option : 4 Women of Kinnauir district wear woolen clothes and wrap a woolen shawl around themselves which is called dohru. They prefer dark colors for Dohru.

******ebook converter DEMO Watermarks*******

Q24. The town of Rishra is known for the industry of: Choose an option: (1). Cotton Textile (2). Jute Textile (3). Silk Textile (4). Woolen Textile Correct option : 2 The town of Rishra near Calcutta is known for the industry of Jute Textile. The first jute mill was established at Rishra in 1855.

******ebook converter DEMO Watermarks*******

Q25. Koraput has rich deposits of : Choose an option: (1). Bauxite (2). Copper (3). Iron-ore (4). Mica Correct option : 1 Koraput is a district of Odisha. The principal economic mineral deposits of Koraput district are Limestone & Bauxite.

******ebook converter DEMO Watermarks*******

Q26. Who won the Women’s Single Wimbledon tournament in 2017? Choose an option: (1). Garbine Muguruza (2). Ekaterina Makarova (3). Elena Vesnina (4). Serena Williams Correct option : 1 Garbine Muguruza won the Women’s Single Wimbledon tournament in 2017.

******ebook converter DEMO Watermarks*******

Q27. Name the sportsperson who has been honoured with 'Bharat Ratna'? Choose an option: (1). Major Dhyan Chanda (2). Sunil Gavaskar (3). Sachin Tendulkar (4). P.T. Usha Correct option : 3 Sachin Tendulkar is a former Indian international cricketer. He is regarded as one of the greatest batsmen of all time. He is the highest run scorer of all time in International cricket. He is the youngest recipient of Bharat Ratna, India’s highest civilian award to date and the first ever sportsperson to receive the award.

******ebook converter DEMO Watermarks*******

Q28. 'Appiko Movement' was leaded by : Choose an option: (1). Sunderlal Bahuguna (2). Pandurang Hegde (3). Vandana Shiva (4). Amrita Devi Correct option : 2 On Sep.8, 1983, Pandurang Hegde started the Appiko (to hug) movement. He derived inspiration from Sunderlal Bahugana’s Chipko movement in Uttar Pradesh, in which villagers used to hug trees to save them from being felled by the State.

******ebook converter DEMO Watermarks*******

Q29. Which of the following is a non-renewable energy source? Choose an option: (1). Solar (2). Wind (3). Nuclear reactors (4). All of the above Correct option : 3 Non-renewable energy comes from sources that will run out or will not be replenished. Most non-renewable energy sources are fossil fuels: coal, petroleum, and natural gas. Even Radioactive substances used in nuclear reactors are also non-renewable.

******ebook converter DEMO Watermarks*******

Q30. Which famous football tournament was started in 1888 from Annadale? Choose an option: (1). Subroto Cup (2). Santosh Trophy (3). Durand Cup (4). Federation Cup Correct option : 3 Durand Cup was started in 1888 from Annadale. The Tournament is named after its founder, Sir Mortimer Durand, Foreign Secretary in charge of India from 1884 to 1894.

******ebook converter DEMO Watermarks*******

Q31. Which was the old capital of Kullu Rajas from where the twelve generations of Kullu Rajas ruled? Choose an option: (1). Manali (2). Jagatsukh (3). Sultanpur (4). Bajaura Correct option : 2

Pal kings administered Kullu district of Himachal Pradesh. Their capital was at Jagatsukh from where the twelve generations of Kullu Rajas ruled.

******ebook converter DEMO Watermarks*******

Q32. The liquid used in a clinical thermometer is: Choose an option: (1). Alcohol (2). Water (3). Mercury (4). Benzene Correct option : 3 The liquid used in a clinical thermometer is Mercury. It has high melting and boiling points making it a liquid over a broad range of temperature. Since it is a metal, it has a stable coefficient of expansion hence it expands uniformly and noticeably for a slight change of temperature. Mercury is a non sticky viscous metal and hence it makes a clear meniscus, helpful in readings. Mercury will not expand when it solidifies hence it doesn’t break the glass tube of the thermometer.

******ebook converter DEMO Watermarks*******

Q33. Who was awarded Best Actress award in the 64^th National Film awards function? Choose an option: (1). M. Surabhi (2). Zaira Wasim (3). Iman Chakraborty (4). Sonam Kapoor Correct option : 1 M. Surabhi won the National Film Award for Best Actress in the 64th National Film Awards for the Malayalam film, Minnaminungu.

******ebook converter DEMO Watermarks*******

Q34. Match List-I and List-II and select the correct answer from the codes given below : List-I I. Sachidanand Vatsyayan II. Aryabhatta III. Ayyankali IV. Fatima Beevi List-II i. Judge ii. Harijan leader iii. Hindi Poet iv. Astronomer Codes: (I) (II) (III) (IV)* Choose an option: (1). (iv) (iii) (i) (ii) (2). (iii) (iv) (ii) (i) (3). (iii) (i) (ii) (iv) (4). (ii) (i) (iii) (iv) Correct option : 2 Sachchidananda Vatsyayan was a pioneering writer of modern Hindi poetry, fiction, criticism and journalism. Aryabhatta was a famous Indian mathematician and astronomer. He was born in a place called Taregana, in Bihar. Ayyankali was a harijan leader who worked for the advancement of untouchables in the princely state of Travancore, British India. Fathima Beevi is a former judge of the Supreme Court of India.

******ebook converter DEMO Watermarks*******

Q35. How many sustainable developments are there in the U.N. Agenda 2030? Choose an option: (1). 14 (2). 15 (3). 16 (4). 17 Correct option : 4

Sustainable Development Goals (SDGs) The United Nations has set 17 Sustainable Development Goals, that aims to build a more prosperous, more equal, and more secure world by the year 2030. These goals are built to negotiate and adopt the global path to sustainable development for the next 15 years.

******ebook converter DEMO Watermarks*******

Q36. Which one of the following is the largest industry in India? Choose an option: (1). Cement (2). Iron & Steel (3). Petroleum (4). Textile Correct option : 4 Textile industry is the largest industry in India. 60% of the Indian textile Industry is cotton based.

******ebook converter DEMO Watermarks*******

Q37. Which one of the following states have the highest percentage of irrigated area? Choose an option: (1). Arunachal Pradesh (2). Haryana (3). Jammu & Kashmir (4). Sikkim Correct option : 2 Among Arunachal Pradesh, Haryana, Jammu & Kashmir and Sikkim, Haryana is having the highest percentage of irrigated area.

******ebook converter DEMO Watermarks*******

Q38. The term 'deflation' is associated with : Choose an option: (1). Glaciers (2). Running water Sea-water (3). Sea-waves (4). Wind action Correct option : 4 Deflation is the removal of loose sediment from the flat land surface by wind. Deflation lowers the land surface and can create a blowout, which is a depression on the land surface caused by wind erosion.

******ebook converter DEMO Watermarks*******

Q39. Which one of the following has the largest number of active volcanoes? Choose an option: (1). Alaska (2). Aleutian Islands (3). Hawaii Islands (4). Iceland Correct option : 2 The Aleutian Islands are a chain of large volcanic islands belonging to both the U.S. state and the Russian federation. The islands, with their 57 volcanoes, form the northernmost part of the Pacific Ring of Fire. These have the largest number of active volcanoes.

******ebook converter DEMO Watermarks*******

Q40. Which one of the following planets has the shortest day? Choose an option: (1). Earth (2). Mars (3). Mercury (4). Venus Correct option : 1 #

Planet

Length of day (Days)

1 Earth

1d

2 Mars

1d 0h 37m

3 Mercury

58d 15h 30m

******ebook converter DEMO Watermarks*******

Q41. Which of the following valleys lies between the Dhauladhar and Pir Panjal? Choose an option: (1). Balh Valley (2). Bara Bhangal Valley (3). Kiara Dun Valley (4). Pabbar Valley Correct option : 2 Bara Bhangal Valley lies between the Dhauladhar and Pir Panjal.

******ebook converter DEMO Watermarks*******

Q42. Which of the following Himalayan zones occupies the maximum percentage of the total geographical area of Himachal Pradesh? Choose an option: (1). Sub-Himalayan Zone (2). Mid-Himalayan Zone (3). High Hill Zone (4). Trans Himalayan Zone Correct option : 1 Sub-Himalayan Zone occupies the maximum percentage of the total geographical area of Himachal Pradesh.

******ebook converter DEMO Watermarks*******

Q43. Where does river Yamuna, after flowing through Uttrakhand, enter in Sirmaur district of Himachal Pradesh? Choose an option: (1). Tajewala (2). Khodarmanjari (3). Baru Sahib (4). Dadahu Correct option : 2 River Yamuna, after flowing through Uttrakhand, enter in Sirmaur district of Himachal Pradesh at Khodarmanjari.

******ebook converter DEMO Watermarks*******

Q44. The movement started under the aegis of ‘Kisan Sabha’ in which Shri Lakshmi Singh and Vaid Surat Singh played a pivotal role was: Choose an option: (1). Praja Mandal (2). Pajhota Andolan (3). Swaraj Sangh (4). Lokraj Sabha Correct option : 2 Pajhota Andolan started under the aegis of ‘Kisan Sabha’. In Pajhota Andolan Shri Lakshmi Singh and Vaid Surat Singh played a pivotal role.

******ebook converter DEMO Watermarks*******

Q45. ‘Democratisation of the Administration’ was the main aim of: Choose an option: (1). Praja Mandal Movement (2). Mandi Conspiracy (3). Bilaspur Struggle (4). Chamba Agitation Correct option : 1 The main aim of Praja Mandal Movement was ‘Democratisation of the Administration’.

******ebook converter DEMO Watermarks*******

Q46. When Himachal Pradesh got right to participate in the election for the first time under the new constitution? Choose an option: (1). 1952 (2). 1957 (3). 1962 (4). 1967 Correct option : 1 Under the new constitution, Himachal Pradesh got right to participate in the election for the first time in 1952.

******ebook converter DEMO Watermarks*******

Q47. Which of the following Monasteries was built by a Western Tibet ruler ‘Ye-Sashoad’ around the year 996 A.D.? Choose an option: (1). Gemur (2). Kardang (3). Tabo (4). Tayul Correct option : 3 Tabo Monastery was built by a Western Tibet ruler ‘Ye-Sashoad’ around the year 996 A.D.

******ebook converter DEMO Watermarks*******

Q48. In 1804 A.D. Rulers of Bilaspur, Mandi, Chamba and other small rulers of Kangra area invited whom to invade Kangra? Choose an option: (1). Gurkha Army Chief Amar Singh Thapa (2). British Army (3). Mughals (4). Maharaja Ranjit Singh Correct option : 1 In 1804 A.D. Rulers of Bilaspur, Mandi, Chamba and other small rulers of Kangra area invited Gurkha Army Chief Amar Singh Thapa to invade Kangra.

******ebook converter DEMO Watermarks*******

Q49. In 1848, who suppressed the rebellion of Wazir of Nurpur and then Pramod Chand and Rajas of Jaswan and Datarpur? Choose an option: (1). Mr Lawerence (2). Mr Jackson (3). Mr Moorcraft (4). Mr Williams Correct option : 1 In 1848, Mr Lawerence suppressed the rebellion of Wazir of Nurpur and then Pramod Chand and Rajas of Jaswan and Datarpur.

******ebook converter DEMO Watermarks*******

Q50. What is the approximate capacity of ‘Shontong Karcham’ Hydro Electric Power Project? Choose an option: (1). 195 MW (2). 100 MW (3). 205 MW (4). 450 MW Correct option : 4 The approximate capacity of ‘Shontong Karcham’ Hydro Electric Power Project is 450 MW.

******ebook converter DEMO Watermarks*******

Q51. To improve the Gross Enrolment Ratio (GER) in Higher Education, the government of H.P. has adopted ‘Rashtriya Uchtar Shikha Abhiyan’ the centrally scheme in the year: Choose an option: (1). 2012 (2). 2013 (2013-14) (3). 2014 (4). 2015 Correct option : 2 In the year 2013 (2013-14), the government of H.P. has adopted the centrally scheme ‘Rashtriya Uchtar Shikha Abhiyan’ to improve the Gross Enrolment Ratio (GER) in Higher Education.

******ebook converter DEMO Watermarks*******

Q52. The Mushroom Development Project in Palampur was launched in 1985 with the introduction of the technology of which country? Choose an option: (1). British (2). Dutch (3). American (4). Brazil Correct option : 2 With the introduction of the technology of Dutch country, the Mushroom Development Project in Palampur was launched in 1985.

******ebook converter DEMO Watermarks*******

Q53. What was the approximate decadal population growth rate of Himachal Pradesh in census 2011? Choose an option: (1). 13.3% (2). 12.9% (3). 12.5% (4). 12% Correct option : 2 The approximate decadal population growth rate of Himachal Pradesh in census 2011 was 12.9%.

******ebook converter DEMO Watermarks*******

Q54. What is India’s estimated GDP growth rate for 2018-19 in the Economic Survey? Choose an option: (1). 3% – 7.1% (2). 2% – 7.5% (3). 7% – 7.5% (4). 7% – 7.2% Correct option : 3 India’s estimated GDP growth rate for 2018-19 in the Economic Survey was 7% – 7.5%.

******ebook converter DEMO Watermarks*******

Q55. What was the Index of Industrial Production in December 2017? Choose an option: (1). 6.8% (2). 7.1% (3). 7.8% (4). 7.4% Correct option : 2 The Index of Industrial Production in December 2017 was 7.1%.

******ebook converter DEMO Watermarks*******

Q56. With what is 'Operation Green' related? Choose an option: (1). Use of paramilitary forces against Naxalites in Maharashtra (2). Use of paramilitary forces against Naxalites in Chhattisgarh (3). Tomato Onion Potato (4). Round Revolution Correct option : 3 'Operation Green' is related to Tomato - Onion - Potato.

******ebook converter DEMO Watermarks*******

Q57. Who is the first woman advocate whose name has been recommended by the collegium to become the Judge of Supreme Court of India? Choose an option: (1). Indu Malhotra (2). Neeru Chadha (3). Sujata V. Manohar (4). Gyan Sudha Mishra Correct option : 1 Indu Malhotra is the first woman advocate whose name has been recommended by the collegium to become the Judge of Supreme Court of India.

******ebook converter DEMO Watermarks*******

Q58. What is the bank account available to Dalits and Women under 'Standup India? Choose an option: (1). 1 – 10 Lakh (2). 10 – 50 Lakh (3). 10 Lakh – 1 Crore (4). 50 Lakh – 1 Crore Correct option : 3 Under 'Stand-up India, bank accounts available to Dalits and Women are 10 lakh – 1 crore.

******ebook converter DEMO Watermarks*******

Q59. Immunization of all children by 2020 is related to: Choose an option: (1). Mission Rainbow (2). Mission Ayushmaan Bharat (3). Mission Arogya (4). Mission Healthy Bharat Correct option : 1 Immunization of all children by 2020 is related to Mission Rainbow. Programme Mission Indradhanush is a health mission of the government of India. It aims to immunize all children under the age of 2 years, as well as all pregnant women, against seven vaccine preventable diseases. Targets The diseases being targeted are diphtheria, whooping cough, tetanus, poliomyelitis, tuberculosis, measles and Hepatitis B. Launch It was launched by Union Health Minister J. P. Nadda on 25 December 2014. Website www.missionindradhanush.in

******ebook converter DEMO Watermarks*******

Q60. According to the Global Nutrition Report 2017, how many females suffered from anaemia in India? Choose an option: (1). 49% (2). 51% (3). 53% (4). 54% Correct option : 2 According to the Global Nutrition Report 2017, 51% females are suffering from anaemia in India.

******ebook converter DEMO Watermarks*******

Q61. What is the estimated social sector expenditure in budget 2018-19? Choose an option: (1). 1.36 lakh crore (2). 1.37 lakh crore (3). 1.38 lakh crore (4). 1.39 lakh crore Correct option : 3 The estimated social sector expenditure in budget 2018-19 is 1.38 Lakh Crore.

******ebook converter DEMO Watermarks*******

Q62. How many poor families will be provided with how much medical insurance per family in 2018-19 under the Ayushmaan Bharat Yojana? Choose an option: (1). 10 crore and Rs 3 lakh (2). 5 crore and Rs 5 lakh (3). 10 crore and Rs 5 lakh (4). 3 crore and Rs 10 lakh Correct option : 3 10 Crore poor families will be provided with Rs 5 Lakh medical insurance per family in 2018-19 under the Ayushmaan Bharat Yojana.

******ebook converter DEMO Watermarks*******

Q63. How many hours per year are the people expected to contribute for cleanliness under the Swachh Bharat Abhiyaan? Choose an option: (1). 70 (2). 80 (3). 90 (4). 100 Correct option : 4 People are expected to contribute 100 hours per year for cleanliness under the Swachh Bharat Abhiyaan.

******ebook converter DEMO Watermarks*******

Q64. Match the items given in column I and II: Column-I (P) Montreal Protocol (Q) Vienna Convention ® Kyoto Protocol (S) Earth Summit Column-II 1. Protection of Ozone Layer 2. Stabilization of Greenhouse Gases 3. Environment and Sustainable Development 4. Total Elimination of Ozone Depleting Substances Match the above: Choose an option: (1). (P-iv), (Q-ii), (R-i), (S-iii) (2). (P-iv), (Q-i), (R-ii), (S-iii) (3). (P-ii), (Q-iv), (R-i), (S-iii) (4). (P-ii), (Q-i), (R-iii), (S-iv) Correct option : 2

******ebook converter DEMO Watermarks*******

Q65. In context of environment, the term “dirty dozen” refers to: 1. 12 ozone layer depleting substances 2. 12 persistent organic pollutants 3. 12 most harmful greenhouse gases 4. 12 most degraded ecosystems Select the correct answer using the codes given below: Choose an option: (1). Only 3 (2). 1, 2 and 3 (3). Only 2 (4). 1, 2, 3 and 4 Correct option : 3 “Dirty dozen” refers to 12 persistent organic pollutants.

******ebook converter DEMO Watermarks*******

Q66. Vast majority of biodiversity occur in : Choose an option: (1). Developing countries (2). Developed countries (3). Equal in both (4). None of the above Correct option : 1 Vast majority of biodiversity occur in developing countries.

******ebook converter DEMO Watermarks*******

Q67. Freshwater present on earth is : Choose an option: (1). 1.5% (2). 0.01% (3). 2.5% (4). 2.97% Correct option : 3 Freshwater present on earth is 2.5%.

******ebook converter DEMO Watermarks*******

Q68. Which industry labours are most prone to Cytosilicosis disease? Choose an option: (1). Electric (2). Computer (3). Dairy (4). Cement Correct option : 4 Cement industry labours are most prone to Cytosilicosis disease.

******ebook converter DEMO Watermarks*******

Q69. Which of the following is not a fundamental unit? Choose an option: (1). Meter (2). Litre (3). Kilogram (4). Second Correct option : 2 Litre is not a fundamental unit.

******ebook converter DEMO Watermarks*******

Q70. The image formed by a convex mirror is : Choose an option: (1). Erect and diminished (2). Erect and enlarged (3). Inverted and diminished (4). Inverted and enlarged Correct option : 1 The image formed by a convex mirror is erect and diminished.

******ebook converter DEMO Watermarks*******

Q71. The global warming has resulted in: Choose an option: (1). Increase in yield of the crop (2). Decrease in sea levels (3). Decrease in human deaths (4). Increase in sea levels Correct option : 4 The global warming has resulted in the increase in sea levels.

******ebook converter DEMO Watermarks*******

Q72. Core of a transformer is made of: Choose an option: (1). Steel (2). Soft iron (3). Copper (4). Aluminium Correct option : 2 Core of a transformer is made of soft iron.

******ebook converter DEMO Watermarks*******

Q73. Which of the following is in the deficiency in a body during 'Dengue' fever? Choose an option: (1). Hemoglobin (2). Sugar (3). Platelets (4). Water Correct option : 3 During 'Dengue' fever, there is deficiency in platelets in a body.

******ebook converter DEMO Watermarks*******

Q74. Who won the Nobel Prize for Chemistry in 2017? Choose an option: (1). Joachim Frank (2). Jacques Dubochet (3). Richard Henderson (4). All of the above Correct option : 4 Nobel Prize for Chemistry in 2017 was won by Joachim Frank, Jacques Dubochet and Richard Henderson.

******ebook converter DEMO Watermarks*******

Q75. Why Hambantota Port has been in news in 2017? Choose an option: (1). Its majority stakes are sold to China. (2). It straddles world’s busiest east-west shipping route. (3). Chinese move here created worries for India. (4). All of the above Correct option : 4 Hambantota Port straddles world’s busiest east-west shipping route. In 2017, majority stakes of Hambantota Port are sold to China. Chinese move here created worries for India.

******ebook converter DEMO Watermarks*******

Q76. Who said that, “The Constituent Assembly was a one party body in The Assembly was the Congress and the Congress was India”? Choose an option: (1). Morris Jones (2). Paul R. Brass (3). Granville Austin (4). Richard Sisson Correct option : 3 Granville Austin once said, “The Constituent Assembly was a one party body in The Assembly was the Congress and the Congress was India”.

******ebook converter DEMO Watermarks*******

Q77. Adi Ambedkar Samaj movement is related to which state? Choose an option: (1). Maharashtra (2). Punjab (3). Rajasthan (4). Uttar Pradesh Correct option : 2 Adi Ambedkar Samaj movement is related to Punjab.

******ebook converter DEMO Watermarks*******

Q78. When did Communist Party of India (Marxist) emerge a separate political party? Choose an option: (1). 1960 (2). 1964 (3). 1968 (4). 1973 Correct option : 2 In 1964, Communist Party of India (Marxist) emerge as a separate political party.

******ebook converter DEMO Watermarks*******

Q79. Who described the critical and creative role of politics in India as 'the Indian model of development'? Choose an option: (1). Rajni Kothari (2). Amartya Sen (3). Jagdish Bhagwati (4). James Manner Correct option : 1 Rajni Kothari described the critical and creative role of politics in India as 'the Indian model of development'.

******ebook converter DEMO Watermarks*******

Q80. The National Forest Policy was launched in : Choose an option: (1). 1988 (2). 1990 (3). 1994 (4). 1996 Correct option : 1 The National Forest Policy was launched in 1988.

******ebook converter DEMO Watermarks*******

Q81. The rate of decrease of temperature in the troposphere is : Choose an option: (1). 5.4 degree Celsius (2). 6.4 degree Celsius (3). 7 degree Celsius (4). 4 degree Celsius Correct option : 2 The rate of decrease of temperature in the troposphere is 6.4 degree Celsius.

******ebook converter DEMO Watermarks*******

Q82. In which case did The Supreme Court give the doctrine of ‘Basic Structure’ of the constitution? Choose an option: (1). Keshavanand Bharti case (2). Golak Nath case (3). Indira Sawhney case (4). Sankari Prasad case Correct option : 1 In Keshavanand Bharti case the Supreme Court gave the doctrine of ‘Basic Structure’ of the constitution.

******ebook converter DEMO Watermarks*******

Q83. Which states/union territories are involved in the Cauvery river dispute? Choose an option: (1). Maharashtra, Karnataka and Tamil Nadu (2). Kerala, Tamil Nadu and Puducherry (3). Tamil Nadu, Karnataka and Kerala (4). Karnataka, Tamil Nadu, Kerala and Puducherry Correct option : 4 Various states/union territories involved in the Cauvery river dispute are: Karnataka, Tamil Nadu, Kerala and Puducherry.

******ebook converter DEMO Watermarks*******

Q84. Supreme Court’s judgment in Indira Sawhney Vs. Union of India case deals with: Choose an option: (1). Reservations for OBCs (2). Reservations for SCs and STs (3). Reservations in Promotion (4). Reservations for Muslims Correct option : 1 Supreme Court’s judgment in Indira Sawhney Vs. Union of India case deals with reservations for OBCs.

******ebook converter DEMO Watermarks*******

Q85. Which of the following is not associated with electoral reforms in India? Choose an option: (1). Tarkunde Committee (2). Dinesh Goswami Committee (3). N. N. Vohra committee (4). Indrajit Gupta Committee Correct option : 3 N. N. Vohra committee is not associated with electoral reforms in India. Vohra committee studied the problem of the criminalisation of politics and of the nexus among criminals, politicians and bureaucrats in India.

******ebook converter DEMO Watermarks*******

Q86. Who described Indian Capitalism as 'Dharmshala Capitalism'? Choose an option: (1). K.R.V. Rao (2). Jagdish Bhagwati (3). Amartya Sen (4). Raj Krishna Correct option : 4 Raj Krishna has described Indian Capitalism as 'Dharmshala Capitalism'. Raj Krishna was an Indian economist who taught at the Delhi School of Economics.

******ebook converter DEMO Watermarks*******

Q87. Who said, "Indian federalism is a new kind of federation to meet India’s peculiar need’s"? Choose an option: (1). Granville Austin (2). Ivor Jennings (3). Alexanderowicz (4). Ronald Wats Correct option : 1 Granville Austin has said, "Indian federalism is a new kind of federation to meet India’s peculiar need’s"?

******ebook converter DEMO Watermarks*******

Q88. Which of the following leaders had worked for tribal upliftment? Choose an option: (1). Mahatma Gandhi (2). A.V. Thakkar (3). B.R. Ambedkar (4). Jagjivan Ram Correct option : 2 Amritlal Vithaldas Thakkar, popularly known as Thakkar Bapa had worked for tribal upliftment in Gujarat state in India.

******ebook converter DEMO Watermarks*******

Q89. Which of the following leaders was never associated with the Indian National Congress? Choose an option: (1). V.D. Savarkar (2). Lala Lajpat Rai (3). Vallabhbhai Patel (4). S. C. Bose Correct option : 1 V.D. Savarkar was never associated with the Indian National Congress. Vinayak Damodar Savarkar, was an Indian, politician, lawyer and writer. Savarkar coined the term Hindutva to create a collective "Hindu" identity as an essence of Bharat.

******ebook converter DEMO Watermarks*******

Q90. Find the correct poverty estimates for the given states in 2011-12, as per the Planning Commission? Choose an option: (1). Arunachal Pradesh 34.7% 33.7% 36.9% 34.7% (2). Bihar 39.9% 36.9% 39.9% 33.7% (3). Chhattisgarh 36.9% 34.7% 33.7% 39.9% (4). Manipur 33.7% 39.9% 34.7% 36.9% Correct option : 4 (A) (B) © (D)*

******ebook converter DEMO Watermarks*******

Q91. Which one of the following is not correctly matched? Choose an option: (1). Slate-Sandstone (2). Schist-Shale (3). Marble-Limestone (4). Quartzite-Sandstone Correct option : 1

******ebook converter DEMO Watermarks*******

Q92. Which one of the following igneous rocks has the highest density? Choose an option: (1). Basalt (2). Olivine (3). Pyroxene (4). Quartz Correct option : 1

******ebook converter DEMO Watermarks*******

Q93. Which of the following Bills was drafted by Dr. B.R. Ambedkar as the first Law Minister of India? Choose an option: (1). Hindu Code Bill (2). Indian Code Bill (3). Hindu Marriage Bill (4). Human Rights Bill Correct option : 1

******ebook converter DEMO Watermarks*******

Q94. Which of the following was not a socialist leader? Choose an option: (1). Ram Manohar Lohiya (2). Jaiprakash Narain (3). S.C. Bose (4). M.R. Jaiyakar Correct option : 4

******ebook converter DEMO Watermarks*******

Q95. Match List-I and List-II and identify the correct answer from the codes given below: List-I (Events) 1. Millennium Development 2. National Commission for Women 3. Ministry of Women and Child Development 4. National Mission for Empowerment of Women List-II (Year) 1. 2010 2. 2006 3. 1992 4. 2011 Codes: (I) (II) (III) (IV) Choose an option: (1). (i) (ii) (iv) (iii) (2). (iv) (iii) (ii) (i) (3). (i) (iv) (iii) (ii) (4). (iii) (i) (ii) (iv) Correct option : 2

******ebook converter DEMO Watermarks*******

Q96. Regarding the Interim Government of 1946 which of the following is not correct? Choose an option: (1). Jawaharlal Nehru was the Prime Minister/Head of the Executive Council (2). Jawaharlal Nehru was designated as the Vice-President of the Viceroy’s Executive Council. (3). The viceroy continue to be the Head of the Executive Council (4). The members of the Interim Government was members of Viceroy’s Executive Council Correct option : 1

******ebook converter DEMO Watermarks*******

Q97. Which of the following are statutory bodies? Select and correct answer from the codes given below : 1. University Grants Commission 2. Official Language Commission 3. Railway Board 4. Atomic Energy Commission Codes: Choose an option: (1). (i) and (ii) (2). (i), (iii) and (iv) (3). (i), (ii) and (iv) (4). (ii) and (iv) Correct option : 2

******ebook converter DEMO Watermarks*******

Q98. Match List-I and List-II and select the correct answer from the codes given below: Choose an option: (1). (iv) (ii) (i) (iii) (2). (ii) (iv) (i) (iii) (3). (i) (iii) (ii) (iv) (4). (iii) (i) (iv) (ii) Correct option : 2 List-I (Panchayati Raj Committees) 1. H. Hanumantha Rao 2. V.K. Rao 3. Ashok Mehta 4. M. Singhvi List-II (Year) 1. 1978 2. 1984 3. 1986 4. 1985 Codes: (I) (II) (III) (IV)

******ebook converter DEMO Watermarks*******

Q99. 1. Match List-I and List-II and select the correct answer from the codes given below : List-I (Books) 1. The Child and the State in India 2. The Hindu Nationalist Movement and Indian Politics 3. The Political Economy of Development in India 4. Poverty Amid Plenty in the New India List-II (Authors) 1. Atul Kohli 2. Pranab Bardhan 3. Christopher Jaferlot 4. Myron Weiner Codes: (I) (II) (III) (IV)* Choose an option: (1). (iii) (iv) (i) (ii) (2). (iv) (iii) (ii) (i) (3). (ii) (i) (iii) (iv) (4). (i) (ii) (iv) (iii) Correct option : 2

******ebook converter DEMO Watermarks*******

Q100. 18. Match the items given in column I and II : Column-I (P) Biological Diversity Act (Q) E-Waste (Management and Handling) Rules ® Hazardous Wastes (Management, Handling and Trans-boundary) Rules (S) Municipal Solid Wastes (Management and Handling Rules Column-II 1. 2000 2. 2008 3. 2011 4. 2002 Choose an option: (1). (P-i), (Q-iv), (R-ii), (S-iii) (2). (P-i), (Q-iv), (R-ii), (S-ii) (3). (P-iv), (Q-iii), (R-ii), (S-i) (4). (P-iv), (Q-iii), (R-i), (S-ii) Correct option : 3

******ebook converter DEMO Watermarks*******

2017 HPAS Solved Paper This PDF Booklet contains 2017 HPAS Solved Paper. Explanation is provided by Shunya Foundation Team. 2017 HPAS Solved Paper for General Studies held in July 2017

******ebook converter DEMO Watermarks*******

Q1. One of the better achievements of Kanha National Park is : Choose an option: (1). Conservation of tiger from near extinction (2). Conservation of swamp deer from near extinction (3). Conservation of Gaur from near extinction (4). Conservation of bison from near extinction Correct option : 2

Kanha National Park is located in Madhya Pradesh. It was created on June 1st 1955 and Kanha Tiger Reserve was created in 1973. This came into being in 1974 under Project Tiger. Kanha has contributed immensely in saving many endangered wildlife species, which nearly bordered extinction. The Park’s landmark conservation effort is the reintroduction of the rare hard ground swamp deer-Barasingha, saving it from near extinction.

******ebook converter DEMO Watermarks*******

Q2. In India, life expectancy is: Choose an option: (1). 68.3 years (2). 71.2 years (3). 75.5 years (4). 65.3 years Correct option : 1 As per year 2014 data, Life expectancy at birth is around 68 years in India.

******ebook converter DEMO Watermarks*******

Q3. United Nations has declared 2017 as the International Year of : Choose an option: (1). Sustainable tourism for development (2). Pulses (3). Light and light-based Technologies (4). Soils Correct option : 1

The United Nations General Assembly declared 2017 as the International Year of Sustainable Tourism for Development. The United Nations (UN) has declared 2017 as a year to recognize that tourism is important in fostering better understanding among people. The International Year aims to support a change in policies, business practices and consumer behavior towards a more sustainable tourism sector than can contribute effectively to the Sustainable Development Goals(SDGs).

******ebook converter DEMO Watermarks*******

Q4. AMRUT was launched on : Choose an option: (1). 25 June, 2015 to improve basic Urban infrastructure in 500 cities (2). 25 June, 2015 to improve basic amenities in 500 districts (3). 25 June, 2014 to improve basic Urban infrastructure in cities (4). 25 June, 2016, 2016 to improve availability of safe drinking water in Urban areas Correct option : 1 Atal Mission for Rejuvenation and Urban Transformation (AMRUT) was launched on 25 June, 2015. The purpose of Atal Mission for Rejuvenation and Urban Transformation (AMRUT) is to (i) ensure that every household has access to a tap with assured supply of water and a sewerage connection; (ii) increase the amenity value of cities by developing greenery and well maintained open spaces (e.g. parks); and (iii) reduce pollution by switching to public transport or constructing facilities for non-motorized transport (e.g. walking and cycling).

******ebook converter DEMO Watermarks*******

Q5. Which of the following instruments is used for measuring the depth of the ocean? Choose an option: (1). Galvanometer (2). Ammeter (3). Barometer (4). Fathometer Correct option : 4

Fathometer is a depth finder that uses sound waves to determine the depth of water in ocean. The first practical fathometer was invented by Herbert Grove Dorsey and patented in 1928.

******ebook converter DEMO Watermarks*******

Q6. Where from is litmus dye extracted? Choose an option: (1). A lichen (2). A liverwort (3). A moss (4). A pteridophyte Correct option : 1

Fig. Litchen Litmus, a natural dye is extracted from lichen. Red litmus changes to blue in a basic solution whereas blue litmus changes to red in acidic medium.

******ebook converter DEMO Watermarks*******

Q7. Which of the following are the primary colors? Choose an option: (1). Yellow, Green, Blue (2). Red, Green, Blue (3). Red, Green, White (4). Red, Magenta, Blue Correct option : 2

Fig. Primary Colours Primary Colours are colours that cannot be created through the mixing of other colours. They are colours in their own right. The three primary colours are RED - GREEN - BLUE. Primary colours can be mixed together to produce secondary colors.

******ebook converter DEMO Watermarks*******

Q8. Nail polish remover contains: Choose an option: (1). Benzene (2). Acetone (3). Acetic acid (4). Petroleum ether Correct option : 2

Nail polishes contain ingredients like resins, plasticizers, film formers and color pigments. Nail polish remover contains a solvent (like acetone) that works by dissolving the hard film that’s left on your nails by the ingredients in the polish. Acetone is made up of carbon, hydrogen, and oxygen combined.

******ebook converter DEMO Watermarks*******

Q9. Which of the following is the most promising biodiesel crop? Choose an option: (1). Sugarcane (2). Soybean (3). Switchgrass (4). Oat Correct option : 2

Biodiesel refers to a vegetable oil- or animal fat-based diesel fuel. Currently soybean oil is being used to produce soybean biodiesel for many commercial purposes such as blending fuel for transportation sectors. It can be used in compression-ignition (diesel) engines with little or no modifications. It is simple to use, biodegradable, nontoxic, and essentially free of sulfur and aromatics.

******ebook converter DEMO Watermarks*******

Q10. Light year is a unit of : Choose an option: (1). Light (2). Intensity of light (3). Astronomical distance (4). Time Correct option : 3 The light year is a unit of length used to express astronomical distances. It is the distance that light can travel in one year.

One light year is roughly equals to 9.4 x 1015 km.

******ebook converter DEMO Watermarks*******

Q11. The most abundant rare gas in the atmosphere: Choose an option: (1). Helium (2). Neon (3). Xenon (4). Argon Correct option : 4

Argon is the most abundant noble gas in the atmosphere. There’s so much Argon in the atmosphere because most of it was produced by a long-lived radioactive isotope of potassium in the earth’s crust and released over geologic time.

******ebook converter DEMO Watermarks*******

Q12. From which place does the Spiti River originate? Choose an option: (1). Kunjam Pass (2). Tandi (3). Tabo (4). Gramfu Correct option : 1

******ebook converter DEMO Watermarks*******

Spiti River

Spiti River near Kunzum Range Spiti River originates from Kunzum Range, which is located at a height of 16,000 ft above sea level. Kabzian and Tegpo streams are the main tributaries of this river.

******ebook converter DEMO Watermarks*******

Q13. Nana Saheb, who fought against the English during the Indian Mutiny of 1857, was: Choose an option: (1). The son of Peshwa Baji Rao 3rd (2). The adopted son of Peshwa Baji Rao II (3). A soldier in the army of Nawab of Awadh (4). A relation of the Raja of Benaras Correct option : 2 Nana Sahib was the adopted son of the exiled Maratha Peshwa Baji Rao II. He was a Maratha soldier and led the Kanpur rebellion during the battle of 1857. Nana Saheb in his efforts against the British was ably supported by two of his lieutenants. One was Tantia Tope, the other was Azimullah.

******ebook converter DEMO Watermarks*******

Q14. Who presided over the first session of Indian National Congress in Bombay? Choose an option: (1). A.O. Hume (2). Gopal Krishna Gokhale (3). B.G. Tilak (4). W.C. Bannerji Correct option : 4 The Indian National Congress was formed when 72 delegates from all the presidencies and provinces of India met at Bombay form 28 to 30 December 1885. Allan Octavian Hume, a retired British official in India, played a leading role in the formation of the congress. The first president of the congress was Womesh Chandra Banerjee. The objectives of the congress, as stated by him, were to bring together leaders from different parts of the country to remove all possible prejudices of race, religion and region, to discuss important problems facing the country, and to decide on the activities that the Indian leaders should take up.

******ebook converter DEMO Watermarks*******

Q15. Which Socio-Religious Movement of 19th-20th century adopted the practices of occult mysticism? Choose an option: (1). The Theosophical Society (2). The Arya Samaj (3). The Brahmo Samaj (4). The Gurdwara Reforms Movement Correct option : 1 The Theosophical Society was a Socio-Religious Movement of 19th-20th century adopted the practices of occult mysticism. The Theosophical Society is an occult organization, mainly based on the teachings of Helena P. Blavatsky, which seeks to extract the common roots of all religions in order to form a universal doctrine.

******ebook converter DEMO Watermarks*******

Q16. Where the first session of Muslim League was held? Choose an option: (1). Dacca (2). Calcutta (3). Lahore (4). Karachi Correct option : 4 All India Muslim League was founded in 1906 under the leadership of Aga Khan, the Nawab of Dhaka & Nawab Mohsin-ul-Mulk. Muslim League made no critique of colonialisation, supported the partition of Bengal, & demanded special safeguards for muslims in Govt. service. The first session of all India Muslim league was held at Karachi on 29th December, 1907 and was presided over by Adamji Peer Bhai.

******ebook converter DEMO Watermarks*******

Q17. Of the following, who shot dead Curzon Wyllie in London? Choose an option: (1). Madan Lal Dhingra (2). Udham Singh (3). Chepkar Brothers (4). None of the above Correct option : 1 Madan Lal Dhingra was a great revolutionary from Punjab,associated with the Indian Home Rule Society, The Abhinav Bharat Society and the Indian House in London. On July 1, 1909 he shot dead Curzon Wyllie an adviser to the secretary of state of India, and Cowas Lolcaca at the meeting of the Indian National Association in London to avenge the atrocities committed by the British in India.

******ebook converter DEMO Watermarks*******

Q18. Where was founded the Communist Party of India in 1920? Choose an option: (1). Moscow (2). Bombay (3). Tashkent (4). Hung Kong Correct option : 3 Communist Party of India was formed on 17th October 1920 in Tashkent by Indian revolutionaries who were outside India at that time.

******ebook converter DEMO Watermarks*******

Q19. Which event earned for Vallabhbhai Patel the title of "Sardar"? Choose an option: (1). The Khera Satyagraha (2). The integration of Indian States with India (3). The Ahmedabad Textile Labour Association (4). The Bardoli Satyagraha Correct option : 4 The Bardoli satyagraha was launched in mid-February 1928. Vallabh Bhai Patel was contacted by the local leaders to take up the leadership of movement. Sardar Patel assumed the leadership and went to Bardoli. He wrote to the government to fulfill the demands of the peasants. After the success of the campaign, Vallabhbhai Patel earned the title of "Sardar".

******ebook converter DEMO Watermarks*******

Q20. In which ocean ‘Mindanao Trench’ exists? Choose an option: (1). Atlantic Ocean (2). Indian Ocean (3). Pacific Ocean (4). Arctic Ocean Correct option : 3 Mindanao Trench also called as Philippine Trench is a submarine trench in the floor of the Philippine Sea. It is located in the western North Pacific Ocean bordering the east coast of the island of Mindanao. It is the world’s third deepest trench. This trench is called Mindanao Trench as it is located near the Mindanao Island, the second largest island of the Philippine (by area). The world’s largest deposits of heavy hydrogen, used in nuclear science, biochemistry and environmental sciences are also found in and around the Philippine Trench.

******ebook converter DEMO Watermarks*******

Q21. Who is the author of the book "The Unstable Earth"? Choose an option: (1). W.M. Davis (2). G. Taylor (3). J.I. Clark (4). J.A. Steers Correct option : 4 The book "The Unstable Earth" deals mainly with the evolution of the continents and oceans and oscillations of sealevel in recent times. It was written by J.A. Steers.

Book Cover Page

******ebook converter DEMO Watermarks*******

Q22. In order to prevent oxidation of potato chips during storage in sealed bags, chip manufacturers seal the packet in the presence of : Choose an option: (1). Oxygen (2). Nitrogen (3). Hydrogen (4). Carbon dioxide Correct option : 2

Nitrogen in Food Preservation If part of the oxygen in air is replaced with Nitrogen, oxidation slows down. Due to this principle, Nitrogen is widely used to process variety of foods, including fruits.

******ebook converter DEMO Watermarks*******

Q23. Which of the following has a pH of above 10? Choose an option: (1). Gastric juice (2). Blood (3). Milk of magnesia (4). Sodium chloride solution Correct option : 3

******ebook converter DEMO Watermarks*******

Milk of Magnesia Milk of Magnesia is a brand name for magnesium hydroxide, a medication used to treat constipation. It’s a type of laxative known as an oral osmotic; it works by drawing water into the colon from the body tissues surrounding it.

pH Values 1. milk of magnesia, pH = 10.5, it is basic. 2. pure water, pH = 7, it is neutral. 3. wine, pH = 3.0, wine is acidic.

******ebook converter DEMO Watermarks*******

Q24. Which of the following processes does not take place during photosynthesis? Choose an option: (1). Absorption of light energy by chlorophyll (2). Conversion of light to chemical energy (3). Splitting of water molecules into hydrogen and oxygen (4). Chlorophyll degradation Correct option : 4

Photosynthesis the process by which green plants and some other organisms use sunlight to synthesize nutrients from carbon dioxide and water. Photosynthesis in plants generally involves the green pigment chlorophyll and generates oxygen as a by-product.

******ebook converter DEMO Watermarks*******

Q25. If someone is suffering form the problem of acidity after overeating, what is the remedy? Choose an option: (1). Lemon juice (2). Vinegar (3). Baking soda (4). Blue berry juice Correct option : 3

Baking Soda Baking soda is a good treatment for immediate relief from acid reflux. Acid reflux is a digestive condition where stomach acid flows from the stomach back into the esophagus (the tract that connects your mouth to your stomach). This backwash of acid can irritate your esophagus and cause heartburn. Heartburn is the burning feeling that can occur anywhere from the middle of your abdomen to your throat. 1. Baking soda can be used as an antacid to help neutralize stomach acid. 2. It can quickly provide temporary relief from acid reflux. 3. Drinking a solution of baking soda too fast can cause side effects like gas and diarrhea. 4. The recommended dosage for adults is 1/2 teaspoon dissolved in a 4-ounce glass of water 5. Baking powder is different than baking soda. Baking soda contains only sodium bicarbonate, a weak base, so it is able to neutralize stomach acid. 6. Each contains sodium bicarbonate, an alkaline chemical that gives off carbon dioxide when mixed with an acid. Baking soda consists purely of sodium bicarbonate, so recipes using it must include an acidic ingredient like lemon juice, buttermilk, or brown sugar (the molasses in brown sugar is acidic) to activate it.

******ebook converter DEMO Watermarks*******

Q26. Which one of the following is a correct sequencing of the features of the Preamble of India Constitution? Choose an option: (1). Sovereign, Socialist, Secular, Democratic, Republic (2). Sovereign, Secular, Socialist, Democratic, Republic (3). Secular, Socialist, Sovereign, Republic, Democratic (4). Democratic, republic, Sovereign, Socialist, Secular Correct option : 1 TEXT OF THE PREAMBLE The Preamble in its present form reads: "We, THE PEOPLE OF INDIA, having solemnly resolved to constitute India into a SOVEREIGN SOCIALIST SECULAR DEMOCRATIC REPUBLIC and to secure to all its citizens: JUSTICE, Social, Economic and Political; LIBERTY of thought, expression, belief, faith and worship; EQUALITY of status and of opportunity; and to promote among them all; FRATERNITY assuring the dignity of the individual and the unity and integrity of the Nation; IN OUR CONSTITUENT ASSEMBLY this twenty-sixth day of November, 1949, do HEREBY ADOPT, ENACT AND GIVE TO OURSELVES THIS CONSTITUTION".

******ebook converter DEMO Watermarks*******

Q27. Which schedule of the Constitution of India contains provisions for the disqualification of a legislator on grounds of defection? Choose an option: (1). Schedule-X (2). Schedule-IX (3). Schedule-XII (4). Schedule-VIII Correct option : 1 The Tenth Schedule — popularly known as the Anti-Defection Act — was included in the Constitution in 1985 by the Rajiv Gandhi ministry and sets the provisions for disqualification of elected members on the grounds of defection to another political party.

******ebook converter DEMO Watermarks*******

Q28. Who presides over the joint sitting of the Parliament? Choose an option: (1). President of India (2). Speaker of Lok Sabha (3). Chairman of Rajya Sabha (4). None of the above Correct option : 2 Joint sitting is an extraordinary machinery provided by the Constitution to resolve a deadlock between the two Houses over the passage of a bill. A deadlock is deemed to have taken place under any one of the following three situations after a bill has been passed by one House and transmitted to the other House: 1. if the bill is rejected by the other House; 2. if the Houses have finally disagreed as to the amendments to be made in the bill; or 3. if more than six months have elapsed from the date of the receipt of the bill by the other House without the bill being passed by it. In the above three situations, the president can summon both the Houses to meet in a joint sitting for the purpose of deliberating and voting on the bill. The Speaker of Lok Sabha presides over a joint sitting of the two Houses and the Deputy Speaker, in his absence.

******ebook converter DEMO Watermarks*******

Q29. Under which provision can the Speaker allow any member of the House to speak in his/her mother language? Choose an option: (1). Article 110 (2). Article 122 (3). Article 120(1) (4). Article 126 Correct option : 3 The Constitution has declared Hindi and English to be the languages for transacting business in the Parliament. However, the presiding officer can permit a member to address the House in his mother-tongue. Under Article 120(1), the Speaker can allow any member of the House to speak in his/her mother language.

******ebook converter DEMO Watermarks*******

Q30. Power of Judicial Review, provided in the constitution, is exercised on the basis of which principle? Choose an option: (1). Due Process of Law (2). Procedure established by Law (3). Arbitration (4). Natural Justice Correct option : 2

Judicial review is the power of the Supreme Court to examine the constitutionality of legislative enactments and executive orders of both the Central and state governments. On examination, if they are found to be violative of the Constitution (ultra-vires), they can be declared as illegal, unconstitutional and invalid (null and void) by the Supreme Court. Consequently, they cannot be enforced by the Government. Judicial review is needed for the following reasons: (a) To uphold the principle of the supremacy of the Constitution. (b) To maintain federal equilibrium (balance between Centre and states) (c) To protect the fundamental rights of the citizens. Judicial Review in India is governed by the principle: ‘Procedure Established by Law’. Under it the court conducts one test, i.e., whether the law has been made in accordance with the powers granted by the Constitution to the lawmaking body and follows the prescribed procedure or not. It gets rejected when it is held to be violative of procedure established by law.

******ebook converter DEMO Watermarks*******

Q31. Which one of the following policies of Government of India is related to the availability of cooking gas to the BPL Households? Choose an option: (1). Ujjawala Yojana (2). Fasal Bima Yojana (3). Atal Pension Yojana (4). Jivan Jyothi Bima Yojana Correct option : 1

Ujjawala Yojana The Pradhan Mantri Ujjwala Yojana (PMUY) is a welfare program of the government of India, launched by Indian Prime Minister Narendra Modi in 2016. The stated objective of the program is providing 50,000,000 LPG connections (50 million) to women from families below the poverty line.

******ebook converter DEMO Watermarks*******

Q32. HDI is measured with reference to: (i) Infant Mortality (ii) Real GDP per capita (iii) Life Expectancy at birth (iv) Morbidity Choose an option: (1). (i) and (iv) are correct (2). (i), (ii) and (iii) are correct (3). (iii) and (iv) are correct (4). None of the above Correct option : 4

The Human Development Index (HDI) The Human Development Index (HDI) is a composite statistic of life expectancy, education, and per capita income indicators, which are used to rank countries into four tiers of human development.

******ebook converter DEMO Watermarks*******

Q33. Which one of the following cities is called 'Silicon Valley of India'? Choose an option: (1). Hyderabad (2). Pune (3). Mumbai (4). Bengaluru Correct option : 4 The Silicon Valley of India is a nickname of Bangalore city. It was named so because Bangalore is on the Mysore Plateau, the area is also sometimes referred to as "Silicon Plateau".

******ebook converter DEMO Watermarks*******

Q34. Which country owns the Margarita Island which was the venue of 17th NAM Summit? Choose an option: (1). Venezuela (2). Egypt (3). Brazil (4). South Africa Correct option : 1 Margarita is a Venezuelan island in the Caribbean Sea. It is located at the Northeast of Caracas, the country’s capital.

******ebook converter DEMO Watermarks*******

Q35. Tehmina Janjua is the first woman in Pakistan’s history to hold the office of : Choose an option: (1). Foreign Minister (2). Foreign Secretary (3). Judge of a High Court (4). Ambassador Correct option : 2 Tehmina Janjua is a Pakistani career diplomat who serves as the 29th Foreign Secretary of Pakistan. Previously, she served as Pakistan’s Representative to the United Nation’s Geneva and as its ambassador to Italy.

******ebook converter DEMO Watermarks*******

Q36. Who among the following is the Chancellor of Nalanda University (2017)? Choose an option: (1). Dr. George Yeo (2). Dr. Vijay Bhatkar (3). Dr. Amartya Sen (4). Dr. Arun Grover Correct option : 2 Nalanda University’s third chancellor is Vijay Bhatkar, a well-known computer scientist who is president of RSSaffiliated outfit Vijnana Bharati, which promotes ‘swadeshi’ science, and was formerly chair of IIT Delhi’s board of governors. Dr Vijay Bhatkar is an internationally acclaimed scientist, IT leader and educationist of India. He is best known as the architect of India’s first supercomputer Param.

******ebook converter DEMO Watermarks*******

Q37. To which country does the UN Secretary General, Antonio Guterres, belong? Choose an option: (1). Italy (2). Portugal (3). Spain (4). Greece Correct option : 2 Antonio Guterres is a Portuguese politician and diplomat who is serving as the ninth Secretary-General of the United Nations. Previously, he was the United Nations High Commissioner for Refugees between 2005 and 2015.

******ebook converter DEMO Watermarks*******

Q38. In which field was Juan Manuel Santos awarded the 2016 Nobel Prize? Choose an option: (1). Chemistry (2). Peace (3). Physics (4). Medicine Correct option : 2 Juan Manuel Santos is the 32nd and current President of Colombia and sole recipient of the 2016 Nobel Peace Prize.

******ebook converter DEMO Watermarks*******

Q39. Donald Trump is: Choose an option: (1). The 45th USA President (2). The 44th USA President (3). The 42th USA President (4). The 40th USA President Correct option : 1

Donald Trump is the 45th and current President of the United States. In the first months of his presidency, Trump reversed several policies of former President Barack Obama, withdrawing the United States from the Trans-Pacific Partnership and the Paris Climate Agreement and undoing parts of the Cuban Thaw.

******ebook converter DEMO Watermarks*******

Q40. Teacher’s Day is celebrated on: Choose an option: (1). 5th September (2). 6th October (3). 2nd October (4). 10th November Correct option : 1 In India Teacher’s Day is celebrated on 5th of September as a tribute to the contribution made by the teachers towards the community. 5th September is the birth anniversary of a great teacher Dr. Sarvepalli Radhakrishnan, who was a staunch believer of education, and was a well-known diplomat, scholar, the President of India and above all, a teacher.

******ebook converter DEMO Watermarks*******

Q41. Which Raja of Chamba transferred the capital of his state from Bharmaur to Chamba? Choose an option: (1). Maru Varman (2). Aditya Varman (3). Sahil Varman (4). Soma Varman Correct option : 3

Chamba is bounded on north-west by Jammu and Kashmir and on the north-east and east by Ladakh area of Jammu and Kashmir state and Lahaul and Bara-Bangal area of Himachal Pradesh. On the south-east and south, Chamba is bounded by the District Kangra and Gurdaspur District of the Punjab. In 920, Raja Sahil Varman shifted the capital of the kingdom to Chamba, following the specific request of his daughter Champavati. Chamba was named after her.

******ebook converter DEMO Watermarks*******

Q42. During the reign of which raja of Nurpur princely state the forts of Maukot, Nurpur and Taragarh were captured by the Mughals? Choose an option: (1). Jagat Singh (2). Rajrup Singh (3). Bir Singh (4). Jaswant Singh Correct option : 1 In 1641 AD, Jagat Singh had long been preparing for a struggle against Mughals, and had strongly fortified the three principal strongholds in his territory. These wer Maukot, Nurpur and Taragarh. But, Mughals captured the forts.

******ebook converter DEMO Watermarks*******

Q43. Which raja of Bushahr princely state was conferred the title of Chhatarpati by the Mughal emperor Aurangzed? Choose an option: (1). Udai Singh (2). Ram Singh (3). Kehri Singh (4). Vijay Singh Correct option : 3 Kehri Singh, the son of Chatar singh was a powerful king. His exploits in Mandi, Suket, Sirmaur and Garhwal earned him the admiration of Emperor Aurangzeb who conferred on him the title of Chhatrapati.

******ebook converter DEMO Watermarks*******

Q44. By what name did the Greek scholars call the Satluj River? Choose an option: (1). Zungtee (2). Muksang (3). Heisidrus (4). Satudri Correct option : 3 Satluj river enters Kinnaur district from the Tibetan territory by a pass and reaches the border of Shimla district near Rampur. This river is called Shatararu in Sanskrit literature, Sutundri in vedic literature, Zaradros or Heisidrus in Greek, Zungtee and Muksung in Tibetan. The tributaries of river Satluj in this district are Spiti river (razing torrent at Khab village) Ropa, Taiti, Keshang, Mulgoon, Yula, Wanger, Shorang and Rupi on the right bank where as Tirung, Gyanthing, Baspa, Duling and Saol dung are its left bank tributaries.

******ebook converter DEMO Watermarks*******

Q45. What is the approximate capacity of Sawra-Kuddu Hydro-Electric Power Project? Choose an option: (1). 65 MW (2). 100 MW (3). 111 MW (4). 130 MW Correct option : 3 The Sawra-Kuddu project is a Run-of-river scheme on the Pabbar river, a tributary of the Yamuna, in Shimla district. The approximate capacity of Sawra-Kuddu Hydro-Electric Power Project is 111 MW.

******ebook converter DEMO Watermarks*******

Q46. At which place in Shimla District of H.P. is Government Sheep Breeding Farm? Choose an option: (1). Chopal (2). Kiari (3). Jeori (4). Mandal Correct option : 3 There are 3 sheep breeding farms in Himachal Pradesh, located at Jeori in Shimla district, Tal in Hamirpur district and Sarol in Chamba district.

******ebook converter DEMO Watermarks*******

Q47. At which place in Bilaspur District of H.P. a woman Industrial Training Institute (I.T.I) has been made functional during 2015-16? Choose an option: (1). Malari (2). Jagatkhana (3). Bharari (4). Daslehra Correct option : 3

Bharari Women ITI Industrial Training Institute (ITI Women) is situated in district Bilaspur of Himachal Pradesh state (Province) in India

******ebook converter DEMO Watermarks*******

Q48. The global agreement that deals with the control of transboundary movements of hazardous wastes and their disposal is called: Choose an option: (1). Basel Convention (2). Convention on Early Notification of a Nuclear Accident (3). International Convention for the Prevention of Pollution form Ships (4). Cartagena Protocol on Biosafety Correct option : 1 Basel Convention is an international treaty that was designed to reduce the movements of hazardous waste between nations, and specifically to prevent transfer of hazardous waste from developed to less developed countries. It does not, however, address the movement of radioactive waste. The Convention was opened for signature on 22 March 1989, and entered into force on 5 May 1992. As of November 2016, 184 states and the European Union are parties to the Convention.

******ebook converter DEMO Watermarks*******

Q49. The biological oxygen demand of water is reduced mainly by: Choose an option: (1). Primary treatment (2). Secondary treatment (3). Tertiary treatment (4). None of the above Correct option : 2 Biochemical Oxygen Demand or BOD, is the amount of oxygen needed by organisms to break down the organic material in wastewater. Conventional wastewater treatment facilities remove BOD using a variety of methods in primary and secondary treatment processes. Primary treatment involves basic processes to remove suspended solid waste and reduce its biological oxygen demand (BOD). This, in turn, increases dissolved oxygen, which is good for aquatic organisms and food webs. Primary treatment can reduce BOD by 20 to 30 percent. Secondary treatment uses biological processes to catch the dissolved organic matter missed in primary treatment. Microbes consume the organic matter as food, converting it to carbon dioxide, water, and energy. Secondary treatment can remove up to 85 percent of BOD.

******ebook converter DEMO Watermarks*******

Q50. The use of carbon tax has been very successful to reduce emissions of greenhouse gases in : Choose an option: (1). The United States of America (2). China (3). Sweden (4). Australia Correct option : 3 In 1995, Sweden became one of the first countries in the work to introduce a carbon tax. This excise tax placed on car​bon-intense fuels such as oil and natural gas has helped actively reduce dependency on fossil fuels. It is considered one of the least expensive means of reducing CO2 emissions.

******ebook converter DEMO Watermarks*******

Q51. Dead zones in the world’s oceans and large lakes are characterized by: Choose an option: (1). High biological oxygen demand (2). Excessive bloom of algae (3). Excess nutrients like nitrates and phosphates (4). All of the above Correct option : 4

Less oxygen dissolved in the water is often referred to as a "dead zone" because most marine life suffocates and dies. Dead zones occur because of a process called eutrophication, which happens when a body of water gets too many nutrients, such as phosphorus and nitrogen. Thus, dead zones in the world’s oceans and large lakes are characterized by high biological oxygen demand, excessive bloom of algae and excess nutrients like nitrates and phosphates. Human activities are the main cause of these excess nutrients being washed into the ocean. For this reason, dead zones are often located near inhabited coastlines.

******ebook converter DEMO Watermarks*******

Q52. Who among the following were conferred Padma Vibhushan Awards in 2017? (1) Dr. Devi Prasad Dwivedi (2) Sadguru Jaggi Vasudev (3) Sharad Pawar (4) Sunder Lal Patwa Select the correct answer from the codes below: Choose an option: (1). (1), (2) and (3) (2). (1), (2), (3) and (4) (3). (1), (3) and (4) (4). (2), (3) and (4) Correct option : 4

2017 Padma Vibhushan Awards 1. Sadguru Jaggi Vasudev - awarded in category of Others - Spiritualism 2. Sharad Pawar - Public Affairs 3. Sunder Lal Patwah - Public Affairs

******ebook converter DEMO Watermarks*******

Q53. Which of the following represents the major loss to the human kind due to environmental degradation? Choose an option: (1). Ecosystem goods (2). Ecosystem services (3). Ecosystem-based tourism (4). Ecosystem-dependent tribes Correct option : 2 Ecosystem services are defined as the benefits that people derive from nature. Over the past 50 years, humans have changed ecosystems more rapidly and extensively than in any comparable period of time in human history, largely to meet rapidly growing demands for food, fresh water, timber, fiber and fuel. This has resulted in a substantial and largely irreversible loss in the diversity of life on Earth. The major loss to the human kind due to this environmental degradation is Ecosystem services.

******ebook converter DEMO Watermarks*******

Q54. India ratified the Stockholm Convention for protecting human health and the environment from persistent organic pollutants in : Choose an option: (1). 2005 (2). 2006 (3). 2002 (4). 2010 Correct option : 2 The Stockholm Convention is a global treaty to protect human health and the environment from persistent organic pollutants (POPs). POPs are chemicals that remain intact in the environment for long periods, become widely distributed geographically, accumulate in the fatty tissue of living organisms and are toxic to humans and wildlife. India ratified the Stockholm Convention on 13/01/2006 and entered it into force on 13/04/2006.

******ebook converter DEMO Watermarks*******

Q55. The number of wetlands in India listed under Ramsar Convention is : Choose an option: (1). 35 (2). 21 (3). 26 (4). 39 Correct option : 3 There are 26 wetlands in India listed under Ramsar Convention. According to WWF-India, wetlands are one of the most threatened of all ecosystems in India. Loss of vegetation, salinization, excessive inundation, water pollution, invasive species, excessive development and road building, have all damaged the country’s wetlands.

******ebook converter DEMO Watermarks*******

Q56. In the Budget for 2017-18, the funds allocated for MGNREGA is: Choose an option: (1). Rs. 45,000 Cr. (2). Rs. 48,000 Cr. (3). Rs. 35,000 Cr. (4). Rs. 55,000 Cr. Correct option : 2 India’s biggest anti-poverty scheme, the Mahatma Gandhi National Rural Employment Guarantee Act or MGNREGA, has been given its "highest allocation ever" of 48,000 crores in the government’s annual Budget 201718. The scheme provides jobs to an estimated 50 million households.

******ebook converter DEMO Watermarks*******

Q57. What was the unemployment rate during 2015-16 based on Usual Principal Status in India? Choose an option: (1). 8.7 percent (2). 7.8 percent (3). 5.0 percent (4). 4.2 percent Correct option : 3 The Usual Principal Status gives an idea about long-term employment (or chronic and open employment) during the reference year. The unemployment rate during 2015-16 based on Usual Principal Status in India was 5%.

******ebook converter DEMO Watermarks*******

Q58. Who conducts the periodical sample survey for estimating the poverty line in India? Choose an option: (1). NITI Aayog (2). CSO (3). NSSO (4). Ministry of Social Justice Correct option : 3 National Sample Survey Organisation (NSSO) was set up in 1950 as a permanent survey organisation to conduct national sample surveys to assist in socio-economic planning and policy-making. NSSO has been conducting sample surveys on a variety of subjects and the data have been widely used by the Government, social scientists and other users. It conducts the periodical sample survey for estimating the poverty line in India. The surveys on Consumer Expenditure, Employment – Unemployment, Social Consumption (Health, Education etc.), Manufacturing Enterprises, Service Sector Enterprises are carried out once in 5 years. And survey of Land and Livestock Holding and Debt and Investment are carried out once in 10 years.

******ebook converter DEMO Watermarks*******

Q59. For the Budget proposals 2017-18; which of the following is not correct? Choose an option: (1). Target for agriculture credit has been fixed to the tune of Rs. 10 Lac Crore (2). New Mini Labs in Krishi Vigyan Kendra will be established (3). Total allocation for rural, agriculture and allied sector is Rs. 80,000 crore (4). Achieving 100 percent village electrification by 1 May, 2018 Correct option : 3 Some key policy measures announced by Budget proposals 2017-18 are: 1. The total allocation for the rural, agriculture and allied sectors in 2017-18 is Rs 1,87,223 crore, which is 24% higher than the previous year. 2. The target for agricultural credit in 2017-18 has been fixed at a record level of Rs 10 lakh crores. 3. The coverage of Fasal Bima Yojana will be increased from 30% of cropped area in 2016-17 to 40% in 2017-18 and 50% in 2018-19. For 2017-18, provided a sum of Rs 9,000 crore

******ebook converter DEMO Watermarks*******

Q60. Stalactites and Stalagmites are: Choose an option: (1). Erosional Landforms in the Karst region (2). Depositional landforms in the Karst region (3). Transportational landforms in the Karst region (4). None of the above Correct option : 2

Fig. Stalagmites Stalactites and stalagmites are types of mineral deposits found in caves that accrue through the processes of solution and deposition. Stalactites hang downwards from the ceiling of caves, whereas stalagmites rise upwards from the floor of a cave.

******ebook converter DEMO Watermarks*******

Q61. Coral reefs are found between: Choose an option: (1). 25°N; 25°S Latitude (2). 30°N; 30°S Latitude (3). 15°N; 15°S Latitude (4). None of the above Correct option : 2 Coral reefs are found in all three of the Earth’s oceans that have portions in the tropics - the Atlantic, Pacific and Indian. They are found between 30° north and south of the equator, in areas where surface water temperatures do not drop below 16°C.

******ebook converter DEMO Watermarks*******

Q62. Savanna region of Brazil is locally called as: Choose an option: (1). Cerrado (2). Llanos (3). Pampas (4). None of the above Correct option : 1 The Cerrado is a vast tropical savanna ecoregion of Brazil. It is world’s most biologically rich savanna. The Cerrado feeds three of the major water basins in South America: the Amazon, Paraguay and São Francisco Rivers.

******ebook converter DEMO Watermarks*******

Q63. In "Concentric Zone Model" of Burgess, the fifth ring is occupied by: Choose an option: (1). Commuters Zone (2). Zone of better residence (3). Zone of independent workers home (4). CBD Correct option : 1 The concentric zone model is one of the earliest theoretical models to explain urban social structures. It was created by sociologist Ernest Burgess in 1925. The zones identified are: 1. The center with the central business district 2. The transition zone of mixed residential and commercial uses or the zone of transition, 3. Working class residential homes (inner suburbs), in later decades called inner city or zone of independent working men’s home, 4. Better quality middle-class homes (outer suburbs) or zone of better housing, 5. Commuter zone.

******ebook converter DEMO Watermarks*******

Q64. Brahmaputra, after joining Tista river; is known as: Choose an option: (1). Jamuna (2). Padma (3). Meghna (4). Dihang Correct option : 1

The Jamuna River is the downstream continuation of the Brahmaputra in Bangladesh. It is one of the largest sandbed braided rivers in the world. In fact, the western branch of Brahmputra river, which contains the majority of the river’s flow, continues due south as the Jamuna to merge with the lower Ganga, called the Padma River.

******ebook converter DEMO Watermarks*******

Q65. Match is following and choose the correct answer from the codes given below: (a) Carl Saur (i) Social Geography (b) David Harvey (ii) Cultural geography (c) F. Ratzel (iii) Political Geography (d) D H.J. Johnston (iv) Radical Geography Code: (a) (b) (c) (d) Choose an option: (1). (i) (ii) (iii) (iv) (2). (ii) (i) (iii) (iv) (3). (iv) (iii) (ii) (i) (4). (ii) (iv) (iii) (i) Correct option : 4 Correct Match 1. Carl Saur - Cultural geography 2. David Harvey - Radical Geography 3. F. Ratzel - Political Geography 4. D H.J. Johnston - Social Geography

******ebook converter DEMO Watermarks*******

Q66. Which of the following States of India has the highest poverty ratio? Choose an option: (1). Bihar (2). Jharkhand (3). Odisha (4). Uttar Pradesh Correct option : 1 State with highest poverty ratio is calculated according to the percentage of people below poverty-line and is based on MRP-consumption. Goa ranks best with least poverty of 5.09% and Chhattisgarh ranks least with 39.93%.

******ebook converter DEMO Watermarks*******

Q67. The growth rate of Himachal Pradesh during 12th Plan (2012-17) has been estimated at: Choose an option: (1). 6.8 percent (2). 7.2 percent (3). 7.8 percent (4). 5.9 percent Correct option : 2 The growth rate during the 12th Plan (2012-17) has been estimated at 7.2%, and Consumer Price Index for Himachal Pradesh has increased by 3.4% in December 2016.

******ebook converter DEMO Watermarks*******

Q68. As per Census 2011, literacy rate in Himachal Pradesh for male and female respectively is: Choose an option: (1). 90.27 and 70.08 (2). 89.53 and 75.93 (3). 89.53 and 71.53 (4). 90.27 and 71.53 Correct option : 2 Literacy rate in Himachal Pradesh has seen upward trend and is 82.80 percent as per 2011 population census. Of that, male literacy stands at 89.53 percent while female literacy is at 75.93 percent.

******ebook converter DEMO Watermarks*******

Q69. Match the following: List1 List2 (a) Article 15 (i) Freedom of Religion (b) Article 20 (ii) Constitutional Remedies (c) Article 25 (iii) Prohibition of Discrimination (d) Article 32 (iv) Protection against conviction Choose the correct answer from the following: Code: Choose an option: (1). (a)-(i), (b)-(ii), (c)-(iii), (d)-(iv) (2). (a)-(iv), (b)-(iii), (c)-(i), (d)-(ii) (3). (a)-(iii), (b)-(iv), (c)-(i), (d)-(ii) (4). (a)-(i), (b)-(iii), (c)-(iv), (d)-(ii) Correct option : 3 Article 15 of Constitution of India deals with Prohibition of discrimination on grounds of religion, race, caste, sex or place of birth. Article 20 talks about Protection in respect of conviction for offences. Article 25 guarantees the freedom to follow any religion and propagate it, yet this freedom comes with a responsibility to ensure that the public order, morality and health are not compromised in the process. Article 32 provides the right to Constitutional remedies which means that a person has right to move to Supreme Court (and high courts also) for getting his fundamental rights protected.

******ebook converter DEMO Watermarks*******

Q70. Supreme Court’s judgement in Indira Sawhney Vs. Union of India is related to which issue? Choose an option: (1). Reservation for OBC (2). Reservation for Scheduled Castes (3). Reservation for Minorities (4). Reservation for Scheduled Tribes Correct option : 1 Supreme Court’s judgement in Indira Sawhney Vs. Union of India is related to reservation for OBC.

******ebook converter DEMO Watermarks*******

Q71. Provision for Grant-in-Aid under Article 243 H for the Panchayats has been made out of which source? Choose an option: (1). Taxes collected by Union Government (2). Taxes collected by the states (3). Taxes collected by Panchayats (4). Consolidated fund of India Correct option : 2 As per Article 243H, the Legislature of a State may, by law, provide for making grants in aid to the Panchayats from the Consolidated Fund of the State. A Consolidated Fund of State has been established where all revenues ( both tax revenues such as Sales tax/VAT, stamp duty etc. and non-tax revenues such as user charges levied by State governments ) are received by the State.

******ebook converter DEMO Watermarks*******

Q72. Which of the following statements is correct in reference to the Finance Commission of India? Choose an option: (1). It facilitates the judicial distribution of finance among the PSUs. (2). It ensures transparency in financial administration. (3). It deals with the financial relations between States and Centre (4). It ensures financial prudence of State Government. Correct option : 3 The Finance Commission came into existence in 1951. It is established for the purpose of allocation of certain resources of revenue between the Union and the State Governments. The Operational duration for the finance commission is five years. The chairman of 13th finance commission was Rangarajan.

******ebook converter DEMO Watermarks*******

Q73. Which one of the following is the correct statement with regard to the Emergency Provisions of Indian Constitution? Choose an option: (1). The constitution mentions four types of emergency (2). Emergency provisions are described from Articles 340 to 356 in the Constitution. (3). Financial emergency has never been imposed in India (4). Financial emergency under Article 356 can be imposed on the failure of Constitutional Machinery at the union level Correct option : 3 If the President is satisfied that there is an economic situation in which the financial stability or credit of India is threatened, he or she can declare financial emergency. Such an emergency must be approved by the Parliament within two months. It has never been declared in India.

******ebook converter DEMO Watermarks*******

Q74. Match List1 (Mines) and List2 (Minerals) using the codes given below: List1 List2 (a) Kimberley (1) Coal (b) Wankie (2) Diamond (c) Witwatersrand (3) Copper (d) Katanga (4) Gold Codes: (a) (b) (c) (d) Choose an option: (1). 1 2 3 4 (2). 2 1 4 3 (3). 1 2 4 3 (4). 2 1 3 4 Correct option : 2 The correct match is: 1. Kimberley - Diamond 2. Wankie - Coal 3. Witwatersrand - Gold 4. Katanga - Copper

******ebook converter DEMO Watermarks*******

Q75. Who of the following, was not the Moderate leader of Indian National Congress? Choose an option: (1). Gopal Krishana Gokhale (2). M.G. Ranade (3). Aurobindo Ghosh (4). Surendra Nath Banerji Correct option : 3 Aurobindo Ghosh was a yogic philosopher, revolutionary writer, spiritual revivalist and Indian freedom fighter. In 1907 at the Surat session of Congress where moderates and extremists had a major showdown, he led with extremists along with Bal Gangadhar Tilak.

******ebook converter DEMO Watermarks*******

Q76. When was the Montague Declaration made about the introduction of responsible government in stages for India? Choose an option: (1). August 15, 1917 (2). August 20, 1917 (3). August 20, 1918 (4). August 15, 1918 Correct option : 2 Montague Declaration is a declaration of Montague, the secretary of State for India. It was made in August 1917. The declaration is in response to the Home Rule Movement. As per the declaration India is an integral part of British Empire and a responsible Government shall be established by gradual development of self governing institutions.

******ebook converter DEMO Watermarks*******

Q77. Green Revolution made the country self-sufficient in food production but had some fallouts. Consider the following statements: (1) It has successful in case of wheat, but not in the case of pulse (2) Brought inter-state disparities (3) Helped in restoring soil fertility (4) Reduced water table in some states Select the correct statements from the codes given below: Choose an option: (1). (1), (2) and (4) (2). (2) and (4) only (3). (1), (3) and (4) (4). All are correct Correct option : 1 Green Revolution refers to a large increase in crop production in developing countries achieved by the use of artificial fertilizers, pesticides, and high-yield crop varieties. The main development was higher-yielding varieties of wheat, which were developed by many scientists, including Indian geneticist M. S. Swaminathan, American agronomist Dr. Norman Borlaug, and others. The introduction of high-yielding varieties of seeds and the increased use of chemical fertilizers and irrigation led to the increase in production needed to make the country self-sufficient in food grains. Green Revolution has done lot of harm, that infact reduced the fertility of soil in longer run, for example: 1. Excessive use of chemical fertilizers decreased soil fertility. 2. Use of electric tube wells decreased groundwater table. 3. Use of insecticide and pesticide polluted the water table, and has become cause of cancer in many villages (Bathinda, etc)

******ebook converter DEMO Watermarks*******

Q78. Kol tribe is found in : Choose an option: (1). Madhya Pradesh and Uttar Pradesh (2). West Bengal and Odisha (3). Assam and Meghalaya (4). Himachal Pradesh and Rajasthan Correct option : 1 The Kol are an ancient tribal community, one of the original inhabitants of northern and central India. They are said to have migrated from central India some five centuries ago. They speak the Baghelkhandi dialect. They are located throughout the states of Uttar Pradesh, Bihar, Jharkhand, Madhya Pradesh, Maharashtra and Tripura.

******ebook converter DEMO Watermarks*******

Q79. Which of the following rivers have been termed as "living entities" by the High Court? (1) Ganga (2) Yamuna (3) Godavari (4) Brahmaputra Select the correct answer from the codes given below: Choose an option: (1). (1) and (4) (2). (1) and (2) (3). (1), (2) and (4) (4). (1), (2), (3) and (4) Correct option : 2 In March, 2017, the Uttarakhand high court declared the Ganga and Yamuna living entities, bestowing on them same legal rights as a person, a move that could help in efforts to clean the pollution-choked rivers.

******ebook converter DEMO Watermarks*******

Q80. The NOTA option was first used in the elections in : Choose an option: (1). 2004 (2). 2009 (3). 2013 (4). 2017 Correct option : 3 While deciding on a demand for providing negative voting, the Supreme Court, on September 27, 2013 had made a judgement thereby directing the Election Commission to provide the NOTA (None of the Above) option on ballot papers and EVMs.

******ebook converter DEMO Watermarks*******

Q81. According to the UN Human Development Report (2016), the world’s top three countries in terms of human development are? Choose an option: (1). Norway, Australia and Switzerland (2). Australia, Sweden and Denmark (3). Switzerland, Norway and Sweden (4). Sweden, Netherlands and Norway Correct option : 1 The 2016 Human Development Report is the latest in the series of global Human Development Reports published by the United Nations Development Programme (UNDP) since 1990 as independent, analytically and empirically grounded discussions of major development issues, trends and policies. According to the UN Human Development Report (2016), the world’s top three countries in terms of human development are Norway, Australia and Switzerland.

******ebook converter DEMO Watermarks*******

Q82. Who among the following has been appointed Director of the Central Bureau of Investigation (2017)? Choose an option: (1). Anil Kumar Sinha (2). Alok Kumar Verma (3). A.K. Patnaik (4). Sanjeev Sanyal Correct option : 2 Delhi Police commissioner Alok Kumar Verma, a 1979 batch Indian Police Service officer of the Arunachal Pradesh-Goa-Mizoram and Union Territory (AGMUT) cadre, was appointed the director of the Central Bureau of Investigation (CBI).

******ebook converter DEMO Watermarks*******

Q83. The district to which two Chief Ministers of Northern States (2017) belong, falls in the state of : Choose an option: (1). Uttar Pradesh (2). Himachal Pradesh (3). Uttarakhand (4). Punjab Correct option : 3

******ebook converter DEMO Watermarks*******

Q84. The Supreme Court issued a contempt notice to the sitting High Court Judge of: Choose an option: (1). Calcutta High Court (2). Bombay High Court (3). Madras High Court (4). Allahabad High Court Correct option : 1 In February, 2017, the Supreme Court on Wednesday issued contempt notice to Justice CS Karnan—​a sitting judge of the Calcutta High Court and ordered him to forthwith refrain from discharging any judicial or administrative functions.

******ebook converter DEMO Watermarks*******

Q85. The Chief Minister and Deputy Chief Minister(s) of the following states were not members of the State Legislative Assembly at the time of their appointments (2017): Choose an option: (1). Punjab (2). Goa (3). Uttarakhand (4). Uttar Pradesh Correct option : 4 As per norms, the chief minister and ministers, not being member of the state legislature, are required to become members of either house of legislature. Yogi Adityanath, CM and Dinesh Sharma & Keshav Prasad Maurya, Dy CMs of Uttar Pradesh were not members of the State Legislative Assembly at the time of their appointments (2017).

******ebook converter DEMO Watermarks*******

Q86. Which month of 2017 marks the 100th anniversary of Gandhi’s arrival in Champaran? Choose an option: (1). March (2). April (3). July (4). August Correct option : 2 The month of April marks the hundredth anniversary of Gandhi’s first major political intervention on Indian soil – in the Champaran district of Bihar. He spent several months here fighting for the rights of the indigo farmers.

******ebook converter DEMO Watermarks*******

Q87. Who among the following were awarded Rajiv Gandhi Khel Ratan Award for 2015-16? (1) P.V. Sindhu (2) Sakshi Malik (3) Jitu Rai (4) M. Bhupathi Select the correct answer from the codes given below: Choose an option: (1). (1), (2) and (3) (2). (2), (3) and (4) (3). (1), (2) and (4) (4). (1), (2), (3) and (4) Correct option : 1 In August, 2016, four athletes, including Olympic silver medallist P.V. Sindhu, gymnast Dipa Karmakar, shooter Jitu Rai and Olympic bronze medallist Sakshi Malik received the Rajiv Gandhi Khel Ratna Award from President Pranab Mukherjee.

******ebook converter DEMO Watermarks*******

Q88. Who among the following was conferred Jnanpith Award in 2016? Choose an option: (1). Bulaki Sharma (2). Shankha Ghosh (3). Satya Narayan (4). Kamal Vora Correct option : 2 Eminent Bengali poet and literary critic Sankha Ghosh was conferred the 52nd Jnanpith Award by President Pranab Mukherjee. The 85-year-old poet was awarded with Sahitya Akademi in the year 1977 for his work "Baabarer praarthanaa" and in 1999, for translating Kannada play "Taledanda" into Bengali as "Raktakalyan". Ghosh was also honoured with Padma Bhushan in 2011.

******ebook converter DEMO Watermarks*******

Q89. According to legend which God (Devta) killed Banasur who was the ruler of Sarahan area of upper Shimla? Choose an option: (1). Tapa Purana (2). Anna Purana (3). Deva Purana (4). Garud Purana Correct option : 3 According to legend Deva Purana killed Banasur who was the ruler of Sarahan area of upper Shimla.

******ebook converter DEMO Watermarks*******

Q90. Who was the first European to draw attention to the history of royal families of Jalandhar and Trigarta? Choose an option: (1). Moorcraft (2). Cunningham (3). Thevenot (4). Thomas Coryat Correct option : 2 Cunningham was the first European to draw attention to the history of royal families of Jalandhar and Trigarta.

******ebook converter DEMO Watermarks*******

Q91. Around which year did the Gurkhas capture the Sirmaur princely state? Choose an option: (1). 1803 AD (2). 1815 AD (3). 1820 AD (4). 1826 AD Correct option : 1 In 1803 AD, Gurkhas captured the Sirmaur princely state.

******ebook converter DEMO Watermarks*******

Q92. Which Himachali revolutionary edited Sudarshan Chakra magazine from Lahore during the pre-independence period? Choose an option: (1). Yashpal (2). Inderpal (3). Sher Jung (4). Swami Krishana Nand Correct option : 2 Inderpal edited Sudarshan Chakra magazine from Lahore during the pre-independence period.

******ebook converter DEMO Watermarks*******

Q93. What is the number of women members in the present H.P. Vidhan Sabha? Choose an option: (1). Two (2). Three (3). Four (4). Five Correct option : 2 There are 3 women members in the present H.P. Vidhan Sabha: 1. Smt. Asha Kumari, Dalhousie-4 constituency 2. Smt. Sarveen Chaudhary, Shahpur-17 constituency 3. Smt. Vidya Stokes, Theog-61 constituency

******ebook converter DEMO Watermarks*******

Q94. How much subsidy is given in H.P. to the women of eligible category for providing Gas connection under Mata Shabri Mahila Sashaktikaran Yojana? Choose an option: (1). 30 percent subject to maximum of Rupees 1,000 (2). 50 percent subject to maximum of Rupees 1,300 (3). 75 percent subject to maximum of Rupees 1,500 (4). 80 percent subject to maximum of Rupees 2,000 Correct option : 2 The objective of Mata Shabri Mahila Sashaktikaran Yojana is to help the women belonging to below poverty line(BPL) families of scheduled caste category. This scheme provides L.P.G. gas connection to the woman belonging to below poverty line(BPL) families of scheduled caste category. Under this scheme, 50% subsidy is given to the eligible woman for the purchase of gas connection subject to the maximum of Rupees 1300/- per family.

******ebook converter DEMO Watermarks*******

Q95. What is the architectural style of Hateshwari Temple at Hatkoti (in Jubbal Tehsil of Shimla District)? Choose an option: (1). Pentroof (2). Pagoda (3). Cone shaped (4). Pyramidal shaped Correct option : 4 The historical temple of Mata Hateshwari is located in Hatkoti village that lies on the banks of Pabbar River, which is approximately 100 Km far from Shimla. The temple is a mixture of Pahari and pyramidal style in construction.

******ebook converter DEMO Watermarks*******

Q96. Which of the following subject matter is not found in the Kangra Kalam style Kangra paintings of eighteenth century? Choose an option: (1). Geet Gobind of Jai Deva (2). Satsai of Bihari (3). Padmavat of Jayasee (4). Rasik Priya of Keshav Das Correct option : 3 Padmavat is an epic poem written in 1540 CE by Malik Muhammad Jayasi in the Awadhi language. It is not the subject matter in the Kangra Kalam style Kangra paintings of eighteenth century.

******ebook converter DEMO Watermarks*******

Q97. In which two Districts of H.P. will the H.P. Forest Ecosystem Climate Proofing Project be implemented? Choose an option: (1). Bilaspur and Solan (2). Shimla and Kinnaur (3). Una and Hamirpur (4). Kangra and Chamba Correct option : 4 HP Forest Eco System Climate Proofing Project will expedite the implementation of Rs. 316 crore in two districts i.e. Kangra and Chamba of Himachal Pradesh. This project has been envisaged in German Collaboration with the objective of minimizing and mitigating the negative impact of climate change, resulting in an increase in biodiversity and sustained income in rural areas through sustainable management of forest resources.

******ebook converter DEMO Watermarks*******

Q98. According to 2010-11 Agricultural census what is the percentage of small land holding in H.P.? Choose an option: (1). 8.84 (2). 12.06 (3). 17.72 (4). 18.17 Correct option : 4 According to 2010-11 Agricultural census, the percentage of small land holding in H.P. is 18.17.

******ebook converter DEMO Watermarks*******

Q99. Which cement company has been given a prospecting license to carry out a detailed study for proving the quantity of lime-stone deposits in SugrathiThangar-Kura Khera-paulikhera-Kandal–dedera of Tehsil Chopal in Shimla District, H.P.? Choose an option: (1). M/s. Reliance Cementation (2). M/s. Associated Cement Companies (3). M/s. Asian Cement Company (4). M/s. Ambuja Cement Company Correct option : 1 M/s. Reliance Cementation has been given a prospecting license to carry out a detailed study for proving the quantity of lime-stone deposits in Sugrathi-Thangar-Kura Khera-paulikhera-Kandal–dedera of Tehsil Chopal in Shimla District, H.P.

******ebook converter DEMO Watermarks*******

Q100. Who is the author of book Kullu and Lahaul? Choose an option: (1). C.G. Bruce (2). E.J. Buck (3). R. M. Bernier (4). W.G. Archer Correct option : 1 Kullu and Lahaul by C.G. Bruce is one of the earliest books, particularly those dating back to the 1900s. It was published by University of California Libraries on 1 January 1914.

******ebook converter DEMO Watermarks*******

2015 HPAS Solved Paper This PDF Booklet contains 2015 HPAS Solved Paper. Explanation is provided by Shunya Foundation Team. 2015 Solved Paper for HPAS Exam

******ebook converter DEMO Watermarks*******

Q1. Which of the following is not a freshwater ecosystem? Choose an option: (1). Pond (2). Lake (3). Wetland (4). Open Ocean Correct option : 4

Aquatic ecosystems provide human beings with a wealth of natural resources. The aquatic ecosystems constitute the marine environments of the seas/open oceans and the fresh water systems in lakes, rivers, ponds and wetlands.

******ebook converter DEMO Watermarks*******

Q2. Which of these is not an ecosystem service? Choose an option: (1). Climate control (2). Recreational Benefits (3). Winds (4). Food Correct option : 3 Ecosystem services are the benefits people obtain from ecosystems. Ecosystem services are divided into four categories: 1. provisioning services, such as food and water 2. regulating services, such as flood and disease control 3. supporting services, such as nutrient cycling and climate control that maintain the conditions for life on Earth, and 4. cultural services, such as spiritual, recreational, and cultural benefits.

Fig. Forests provide flood control feature. Human interventions have dramatically increased food provisioning services through the spread of agricultural technologies, although this has resulted in changes to other services such as water regulation.

******ebook converter DEMO Watermarks*******

Q3. Desert ecosystem are characterized by annual rainfall of less than Choose an option: (1). 25 cm (2). 30 cm (3). 40 cm (4). 50 cm Correct option : 1

Deserts are formed in regions with less than 25 cm of annual rainfall or sometimes in hot regions where there is more rainfall, but unevenly distributed in the annual cycle.

******ebook converter DEMO Watermarks*******

Q4. Which of these is not a factor threatening extinction of several species in India? Choose an option: (1). Population explosion (2). Climate change (3). Laxity in implementation of environment policies (4). Socio-cultural practices Correct option : 4 Major threats to biodiversity in India are as follows: 1. Population explosion 2. Habitat Destruction (Deforestation) 3. Overuse of Natural Resources 4. Climate Change 5. Laxity in implementation of environment policies

Fig. Deforestation

******ebook converter DEMO Watermarks*******

Q5. Which of these places is reputed to be world’s most biologically diverse? Choose an option: (1). Yasuni tropical rain forest (2). Western Ghats (3). Eastern Himalayan (4). Indo-Burma region Correct option : 1 Yasuni National Park is in Ecuador between the Napo and Curaray rivers. It is the most biologically diverse spot on Earth. The Park covers 9,820 square kilometers. Located in the eastern foothills of the Andes, the park slopes down gently from west to east and is known for the many rivers and creeks that carry water from the mountains into the Amazon system.

******ebook converter DEMO Watermarks*******

Q6. India is to home to how many (approximately) mammalian species? Choose an option: (1). 1320 (2). 628 (3). 350 (4). 3560 Correct option : 3

Fig - Lion - A Mammal Mammal is any animal of which the female feeds her young on milk from her own body; such as humans, dogs , lions , and whales. According to an estimate, there are 400 species of mammals, 1200 species of birds, 350 species of reptiles and more than 29,70,000 species of insects.

******ebook converter DEMO Watermarks*******

Q7. According to Paris agreement on climate change, the rise in average temperature of earth’s surface above pre-industrial levels should be limited to: Choose an option: (1). 1 Degree Celsius (2). 2 Degree Celsius (3). 2.5 Degree Celsius (4). 3 Degree Celsius Correct option : 2 The Paris Agreement’s central aim is to strengthen the global response to the threat of climate change by keeping a global temperature rise this century well below 2 degrees Celsius above pre-industrial levels and to pursue efforts to limit the temperature increase even further to 1.5 degrees Celsius.

Fig. Receding glaciers due to climate change

******ebook converter DEMO Watermarks*******

Q8. In which of the following states, ‘Hornbill’ festival has been observed to uphold Unity in Diversity? Choose an option: (1). Nagaland (2). Manipur (3). Assam (4). Mizoram Correct option : 1

Fig. Hornbill bird The Hornbill Festival is a colorful festival organized every year in Nagaland. With the idea to promote unity and diversity, Hornbill Festival was aptly named in collective reverence to the bird enshrined in the cultural ethos of the Nagas.

******ebook converter DEMO Watermarks*******

Q9. Which of the following is not correct about the Harappa’s contacts with its contemporary civilizations? Choose an option: (1). Similarity of some of its seals with them. (2). The term Melukha used in Mesopotamian tablets perhaps refers to Sindhu. (3). The Harappan seals mention its relations with them. (4). The Mesopotamians called cotton by the name of Sindhu. Correct option : 3 The Harappan seal is the earliest form of writing known in Indian subcontinent. They have not been deciphered so far.

******ebook converter DEMO Watermarks*******

Q10. Who built the Sudarshana lake in the Saurashtra region? Choose an option: (1). Pushyagupta (2). Dhanananda (3). Kharavela (4). Samudragupta Correct option : 1 Chandragupta’s viceroy or governor, Pushyagupta is said to have constructed the famous Sudarshana Lake in Surashtra or Kathiawar.

******ebook converter DEMO Watermarks*******

Q11. Which of the following Bhakati poets of Maharashtra conveyed his teachings through the numerous Abhangas? Choose an option: (1). Tukaram (2). Namdev (3). Eknath (4). Ramdas Correct option : 1 Tukaram was a 17th-century poet-saint of the Bhakti movement in Maharashtra. He conveyed his teachings through the numerous Abhangas.

******ebook converter DEMO Watermarks*******

Q12. Dara Shikoh was the disciple of which sufi saint? Choose an option: (1). Mirza Galib (2). Mian Mir (3). Mullah Shah (4). Mir Taki Mir Correct option : 2 Mian Mir was a famous Sufi Muslim saint who resided in Lahore. His most famous disciple was Dara Shikoh.

******ebook converter DEMO Watermarks*******

Q13. Who established the East India association in 1866 in London? Choose an option: (1). Raja Ram Mohan Roy (2). Dada Bhai Nairoji (3). Surendra Nath Banerjee (4). Lala Lajpat Rai Correct option : 2 The East India Association was founded by Dadabhai Naoroji in 1866, in collaboration with Indians and retired British officials in London.

******ebook converter DEMO Watermarks*******

Q14. How much time did Lord Mountbatten give to Sir C. Radcliff to delineate the international border of India in 1947? Choose an option: (1). 1 and a half months (2). 3 and a half months (3). 4 months (4). 6 months Correct option : 1 On 17th August 1947 the Radcliffe Line was declared as the boundary between India and Pakistan, following the Partition of India. The idea behind the Radcliffe Line was to create a boundary which would divide India along religious demographics, under which Muslim majority provinces would become part of the new nation of Pakistan and Hindu and Sikh majority provinces would remain in India. Lord Mountbatten gave 1 and a half months to Sir C. Radcliff to delineate the international border of India in 1947.

******ebook converter DEMO Watermarks*******

Q15. India celebrated its first constitution Day on? Choose an option: (1). 15th august, 1947 (2). 26th January, 1950 (3). 26th November, 1949 (4). None of the above Correct option : 3 Constitution Day is celebrated on 26 November every year to commemorate the adoption of Constitution of India. On this day in 1949, the Constituent Assembly of India adopted the Constitution of India, and it came into effect on 26 January 1950.

******ebook converter DEMO Watermarks*******

Q16. In which year was the Protection of Women from Domestic Violence Act passes? Choose an option: (1). 1998 (2). 2001 (3). 2004 (4). 2005 Correct option : 4 The Protection of Women from Domestic Violence Act 2005 provides a legal recourse to women (wives and live in partners) facing domestic violence be it physical, mental, sexual, emotional or economical. It is an Act of the Parliament of India enacted to protect women from domestic violence. It was brought into force by the Indian government from 26 October 2006.

******ebook converter DEMO Watermarks*******

Q17. Ashish Bose is well known for his concept Choose an option: (1). Demographic Transition (2). Population Explosion (3). BIMARU States (4). Sustainable Development Correct option : 3 Ashish Bose was best known for coining the term BIMARU (in a paper presented to then Prime Minister Rajiv Gandhi) to refer to the four grossly under-developed states of (undivided) Bihar, Madhya Pradesh, Rajasthan and Uttar Pradesh in the 1980s. Later, Orissa was also included, to expand it to BIMAROU. Bose’s contention was if the government wants to control the population, it should stress on these states.

******ebook converter DEMO Watermarks*******

Q18. Bombyx mori is a Choose an option: (1). Fish (2). Silkworm (3). Ringworm (4). Moth Correct option : 2 Bombyx mori, also known as Silkworm is an economically important insect, having been domesticated for sericulture (silk production). Few Key Points 1. Silkworm is the primary producer of raw silk. 2. Sericulture, the practice of breeding silkworms for the production of raw silk, has been under way for at least 5,000 years in China, from where it spread to Korea and Japan, India and later the West.

Silkworm Image

******ebook converter DEMO Watermarks*******

Q19. Which of the following Indian States will get the first ever solar-stellar observatory? Choose an option: (1). Rajasthan (2). Gujarat (3). Maharashtra (4). Karnataka Correct option : 2 Gujarat is all set to get its first-ever solar as well stellar observatories, with government-recognised Indian Planetary Society (IPS) selecting Kutch district for embarking on this project at a cumulative cost of Rs. 12 crores.

******ebook converter DEMO Watermarks*******

Q20. Which is not an indicator of Human Development Index? Choose an option: (1). Life expectancy (2). Education component (3). National per capita income (4). Level of Urbanization Correct option : 4 The Human Development Index (HDI) is a composite statistic of life expectancy, education, and per capita income indicators, which are used to rank countries. HDI of India - UNDP, the sponsor of Human Development Index methodology since 1990, reported India’s HDI to be 0.554 for 2012, an 18% increase over its 2008 HDI. United Nations Declared India’s HDI is 0.586 in 2014, an 5.77% increase over 2012. As for the year 2015, HDI for India stood at 0.609.

******ebook converter DEMO Watermarks*******

Q21. Which soil is best for cotton cultivation? Choose an option: (1). Alluvial Soil (2). Laterite Soil (3). Red Soil (4). Black soil Correct option : 4

Fig. Black Soil Suitable soil for cotton should have maximum proportion of silt and clay. The soils having these contents have good water holding capacity and these soils can protect the crop and hold it well. Black soil is best suited for cotton cultivation.

******ebook converter DEMO Watermarks*******

Q22. Bermuda Triangles is located in? Choose an option: (1). Atlantic Ocean (2). Pacific Ocean (3). Indian Ocean (4). China Ocean Correct option : 1 The Bermuda Triangle is a region in the western part of the North Atlantic Ocean.

******ebook converter DEMO Watermarks*******

Q23. Which is the longest river of the World? Choose an option: (1). Nile (2). Amazon (3). Yangtze (4). Congo Correct option : 1

Nile River, Africa Flowing northward through the tropical climate of eastern Africa and into the Mediterranean Sea, the Nile river is the longest river in the world measuring about 6,650 kilometers. That is twice the width of India from north to south (Kashmir to Kanyakumari).

******ebook converter DEMO Watermarks*******

Q24. When a body is taken from the earth to moon Choose an option: (1). Mass change but weight remain the same (2). Weight change but mass remain the same (3). Both weight and mass change (4). Both weight and mass remain the same Correct option : 2

Fig. The Moon The moon is 1/4 the size of Earth, so the moon’s gravity is much less than that of earth, 1/6 to be exact. So the mass remains same but weight decreases. Mass of a body never changes due to gravity effect.

******ebook converter DEMO Watermarks*******

Q25. Decibel is used to measure the intensity field of Choose an option: (1). Magnetic field (2). Sound (3). Light (4). Heat Correct option : 2

The loudness or intensity field of sound is expressed in a unit called decibel (dB). Decibel (dB) is a unit used to measure the intensity of a sound or the power level of an electrical signal by comparing it with a given level on a logarithmic scale. The decibel ( dB) is used to measure sound level, but it is also widely used in electronics, signals and communication. Examples 1. 0 dB threshold for healthy hearing 2. 10 dB for Breathing 3. 20 dB for whisper or rustling leaves 4. 60 db for Air Conditioner Noise 5. 100 dB for Jet take-off (at 305 meters) 6. 145 dB for Firecrackers 7. 180 dB for Rocket launch

******ebook converter DEMO Watermarks*******

******ebook converter DEMO Watermarks*******

Q26. A pressure cooker works on the principal of Choose an option: (1). Elevation of boiling point of water by application of pressure (2). Making the food- grains inside softer (3). Making the food-grains softer by application of pressure and temperature (4). Keeping the food-grains inside steam for a longer time Correct option : 1 In a sealed pressure cooker, the boiling point of water increases as the pressure rises, resulting in super-heated water. Without pressure cooker, there is no way that you can increase boiling water temperature beyond 100 degree Celsius. Do you know? Cooking takes more time on hills (high altitude place) compared to plains because atmospheric pressure is low at hills which reduces boiling temperature of water.

******ebook converter DEMO Watermarks*******

Q27. Sweat mainly serves to eliminate Choose an option: (1). Water and NaCl (2). NaCl and Fats (3). Water and Dirt (4). Serum and Water Correct option : 1 Sweat is: 1. 99% water 2. NaCI 3. Vitamin C 4. Antibodies 5. Metabolic wastes 6. Acidic nature inhibits bacteria growth Tip Why sweating happens? Sweating is your body’s built-in cooling system. When the temperature rises, millions of tiny sweat glands in your skin are activated and sweat is released through your pores. As your sweat evaporates, it cools you off. Yet leftover sweat can leave your skin soaked - especially on humid days when the air is already so saturated with moisture that your sweat evaporates more slowly.

******ebook converter DEMO Watermarks*******

Q28. In which basin is Upper Joiner hydro-power project? Choose an option: (1). Ravi (2). Satluj (3). Beas (4). Yamuna Correct option : 1 Upper Joiner hydro-power project is in Ravi basin.

******ebook converter DEMO Watermarks*******

Q29. The acronym UNFCC represents: Choose an option: (1). United Nation Framework on Climate Change (2). United Nation Fund for Climate control (3). United Nation Fund for Climate change (4). United Nation Framework for climate change Correct option : 1 In 1992, countries joined an international treaty, the United Nations Framework Convention on Climate Change(UNFCCC), as a framework for international cooperation to combat climate change by limiting average global temperature increases and the resulting climate change, and coping with impacts that were, by then, inevitable. (This question is wrongly asked It should be UNFCCC instead of UNFCC.)

******ebook converter DEMO Watermarks*******

Q30. Identify the correct sequence of countries with the highest biodiversityChoose an option: (1). Brazil > Colombia > China > Indonesia (2). Brazil > Colombia > India > Indonesia (3). Bolivia > Indonesia > Colombia > India (4). Bolivia > Venezuela> Indonesia > Colombia Correct option : 1

Fig. Iguazu Falls, Brazil The megadiverse countries are a group of countries that harbor the majority of Earth’s species and high numbers of endemic species. The sequence of countries with the highest biodiversity is : Brazil > Colombia > China > Indonesia

******ebook converter DEMO Watermarks*******

Q31. In the context of climate change, measures undertaken by Government of India have a target of electric power generation by renewable energy (by year 2030) of Choose an option: (1). 350 GW (2). 175 GW (3). 500 GW (4). 250 GW Correct option : 1

Fig. Wind Mill - A source of renewable energy In the context of climate change, measures undertaken by Government of India have a target of electric power generation by renewable energy (by year 2030) of 350 GW.

******ebook converter DEMO Watermarks*******

Q32. In a poultry unit the factor most influencing the cost is the cost of : Choose an option: (1). Feed (2). Transportation (3). Chicks (4). Medicines Correct option : 1 The financial needs for poultry industry depends on- the number of birds they are maintained in the unit concerned. For that, feed is the major item of expenditure in poultry production.

******ebook converter DEMO Watermarks*******

Q33. Which of the following is not the feature of caste system? Choose an option: (1). Hierarchy (2). Hereditary occupation (3). Exogamous marriages (4). Accountability Correct option : 3 Exogamous marriages or Exogamy refers to the custom of marrying to persons outside of specific community or groups. It is not a feature of caste system.

******ebook converter DEMO Watermarks*******

Q34. Which state of India was the first to pass Jamindari Abolition Act? Choose an option: (1). Bihar (2). Uttar Pradesh (3). RajasUttthan (4). Karnataka Correct option : 2 Uttar Pradesh had passed Jamindari Abolition Act in 1951.

******ebook converter DEMO Watermarks*******

Q35. Which of the following vegetable oils does not contain essential fatty acids? Choose an option: (1). Sunflower oil (2). Mustard oil (3). Coconut oil (4). Groundnut oil Correct option : 1 Fatty acids are key constituent of lipids. Fatty acids are used to help with essential body functions such as blood clotting, immune response, or blood pressure. Essential fatty acids are not present in sunflower oil.

******ebook converter DEMO Watermarks*******

Q36. Which of the following statements is correct? Choose an option: (1). Wheat bran is rich in carbohydrates (2). Fungal fermentation of rice is used for the preparation of alcoholic drinks (3). Rice has antibiotic properties (4). Wheat contains 80-85% carbohydrates Correct option : 2

******ebook converter DEMO Watermarks*******

Wheat Bran is Rich Source of Fibre 1. Wheat bran is the by-product of wheat milling industries and is valued as a rich source of dietary fibre. White wheat flour has a high carbohydrate content, but wheat bran is mainly fibre and as such is low in carbohydrates. As the insoluble wheat bran passes through your system, it aids digestion, adds bulk to stools and prevents constipation. This is also why it is widely used in breakfast cereals. 2. Fermentation of Rice by yeast (fungi) is used to make Sake, a alcoholic beverage (wine). 3. Whole wheat contains about 70% carbohydrates. 4. Rice does not have anti-biotic properties.

******ebook converter DEMO Watermarks*******

Q37. Vegetables are easily perishable because of their high content of: Choose an option: (1). Sugars (2). Water (3). Vitamins (4). Enzymes Correct option : 2

Fig, Vegetables Perishable foods are likely to spoil, decay or become unsafe to consume if not kept refrigerated. Refrigeration slows bacterial growth and freezing stops it. Vegetables are easily perishable because of their high content of water.

******ebook converter DEMO Watermarks*******

Q38. Of which rivers tributaries are Phojal, Sarwati and Hansa streams Choose an option: (1). Satluj (2). Beas (3). Ravi (4). Yamuna Correct option : 2

Phojal, Sarwati and Hansa streams are tributaries of Beas river.

******ebook converter DEMO Watermarks*******

Q39. What is the archaeological style of Adi Brahama temple Khokhan? Choose an option: (1). Pagoda (2). Pent-roof (3). Pyramid-roof (4). Pyramid Correct option : 1 Adi Brahma Temple, dating back to the 14th century, is located 12 kms from Kullu, at Khokhan village. Built on the Pagoda style with a total of four floors, the temple is believed to have taken ten years to complete.

******ebook converter DEMO Watermarks*******

Q40. With which region of H.P. is Bonangchyu dance associated? Choose an option: (1). Chamba (2). Sirmour (3). Kinnaur (4). Kangra Correct option : 3 The Banyangchu is generally a male dance in Kinnaur district. It is a free style dance. Women sometimes provide vocal music from outside.

******ebook converter DEMO Watermarks*******

Q41. Who was the court poet of Raja Dharam Chand of Kangra who wrote Dharam Chand Natak in around 1562 AD? Choose an option: (1). Ganesh Singh (2). Kanhyaya Lal Sharma (3). Manik Chand (4). Padam Chand Guleri Correct option : 3 Dharam Chand Natak was written by Manik Chand in the reign of Raja Dharam Chand in 1562 AD. In Dharam Chand Natak, refernce is made to the capture of Kangra Fort by Firoz Shah Tughlaq in 1365 AD.

******ebook converter DEMO Watermarks*******

Q42. To which princely state did Zorawar Singh, General of Raja Gulab Singh who invaded Ladakh and Baltistan during 1834-41, belongs ? Choose an option: (1). Kangra (2). Sirmaur (3). Jubbal (4). Kehlur Correct option : 4 Zorawar Singh was born in Kahlur State in Bilaspur. In reference to his legacy of conquests in the Himalayas including Ladakh, Tibet, Baltistan and Iskardu as General and Wazir (Minister), he has been referred to by historians as "Conqueror of Ladakh" and the "Napoleon of India".

******ebook converter DEMO Watermarks*******

Q43. When did the Chandrahabhaga Valley which was held as Jagir by the families of Tibetan origin come under sway of Kullu ? Choose an option: (1). Thirteenth century (2). Fifteenth century (3). Seventeenth century (4). Eighteenth century Correct option : 3 Chandrahabhaga Valley was held as Jagir by the families of Tibetan origin. It came under sway of Kullu in Seventeenth century.

******ebook converter DEMO Watermarks*******

Q44. The territory of which of the following princely state was divided between two cousins/scions Vijay Singh and Ram Singh during the nineteenth century? Choose an option: (1). Daturpur (2). Siba (3). Guler (4). Jaswan Correct option : 2 After the death of Raja Gobind Singh, his son Raja Ram Singh became Raja of Siba state. The territory of Siba state was divided between two cousins/scions Vijay Singh and Ram Singh during the nineteenth century. On appeal to the British Vice Royal at Lahore, the share of Siba jagir i.e. 1/4th of the Siba state was later on restored to Vijay Singh, son of Mian Devi Singh.

******ebook converter DEMO Watermarks*******

Q45. According to 2011 census which district of H.P. has the highest number of urban households? Choose an option: (1). Kangra (2). Shimla (3). Mandi (4). Solan Correct option : 2 According to 2011 census Shimla district has the highest number of urban households.

******ebook converter DEMO Watermarks*******

Q46. Around which year did captain RC Lee set up an apple orchard in Kullu area? Choose an option: (1). 1870 AD (2). 1895 AD (3). 1905 AD (4). 1910 AD Correct option : 1

Kullu apples are well known throughout the country. Pioneer of fruit growing in Kullu area was late Captain RC Lee of Bundrole orchard. He obtained apples from UK and set up an orchard in the year 1870.

******ebook converter DEMO Watermarks*******

Q47. Which Raja of Bushahr princely state received Hang-Rang valley from Tibet as Jagir? Choose an option: (1). Prem Singh (2). Karam Singh (3). Bhup Singh (4). Kehri Singh Correct option : 4 Raja Kehri Singh of Bushahar state is described as the highest skilled warrior of the time. The Tibetan-LadakhiMughal war took place in 1681-83. In the war, Kehri Singh sided with Tibet against Ladakh. As per the treaty, Tibet and Bushahar would remain friends and the traders from both sides would enjoy all facilities without payment of any tax. Kehri Singh also received 'Hangrang valley' from Tibet as a Jagir.

******ebook converter DEMO Watermarks*******

Q48. Around which year did Russian painter Nicholas Roerich visit Naggar in Kullu District for the First time? Choose an option: (1). 1915 (2). 1923 (3). 1926 (4). 1930 Correct option : 2 Late Russian painter Nicolas Roerich visited Kullu for the first time in 1923 and painted enchanting landscapes on canvas. He liked Naggar so much that he returned in 1929 and brought what was then called 'The Hall Estate' with its cottage.

******ebook converter DEMO Watermarks*******

Q49. Which Raja of Bangahal princely state was treacherously killed by Raja Siddha Sen of Mandi? Choose an option: (1). Raghunath Pal (2). Dalel Pal (3). Prithi Pal (4). Man Pal Correct option : 3 Prithi Pal was the son-in-law of Sidh Sen of Mandi. Sidh Sen had covetous eyes on Prithi Pal’s principlity. Once he invited him to Mandi and received with great honour, but was killed treacherously within a month.

******ebook converter DEMO Watermarks*******

Q50. Which school did Amrita Shergil join in Shimla around 1924-25? Choose an option: (1). Tara Hall (2). Alfred Corpet (3). Santa Annciate (4). Jesus and Mary Correct option : 4 Amrita Shergil joined Jesus and Mary Convent school in Shimla. It was a Roman Catholic institution.

******ebook converter DEMO Watermarks*******

Q51. Which Raja of Sirmaur princely state shifted the capital of his state from Rajban to Kalsi in the beginning of the thirteen century? Choose an option: (1). Udit Prakesh (2). Kaul Prakash (3). Maha Prakash (4). Sumer Prakash Correct option : 1 Raja Udit Prakash ascended the throne of Sirmaur state in the year 1217. He shifted the capital of his state from Rajban to Kalsi. He abdicated in favour of his son Kaul Prakash in 1227 AD.

******ebook converter DEMO Watermarks*******

Q52. At which place in Kullu District of Himachal Pradesh is fish farm? Choose an option: (1). Patil Kuhl (2). Moheli (3). Nagini (4). All of the Above Correct option : 4

Fig. Fish The important varities of fish found in Kullu district are Trout and Himalayan Barbel. The fish farms in Kullu district are located at Patli Kuhl, Moheli and Nagini.

******ebook converter DEMO Watermarks*******

Q53. Who founded Himachal Pradesh congress (HVC)? Choose an option: (1). Maheshwar Singh (2). Thakur Ram Lal (3). Dr. Salig Ram (4). Pandit Sukh Ram Correct option : 4 Himachal Vikas Congress(HVC) was a regional political party in Himachal Pradesh. It was founded by Pandit Sukh Ram. HVC was formed when Sukh Ram split from the Indian National Congress. HVC merged with Congress in 2004.

******ebook converter DEMO Watermarks*******

Q54. Which two districts of H.P. are covered under Backward region Grant fund of ministry of Panchayati Raj, Government of India? Choose an option: (1). Kinnaur and Lahul Spiti (2). Lahul-Spiti and Chamba (3). Chamba and Sirmour (4). Sirmour and Kinnaur Correct option : 3 The Planning Commission had identified two districts namely Chamba & Sirmour of Himachal Pradesh on the basis of their incidence of poverty and level of other objective Socio-Economic indicators. The Planning Commission, Government of India has approved to cover both these districts under RSVY on the recommendation of State Government. The RSVY was earlier implemented by Planning Commission, GOI but the programme has been handed over to Ministry of Panchayati Raj, GOI during the year 2007-08 and replaced by Backward Regions Grant Fund (BRGF).

******ebook converter DEMO Watermarks*******

Q55. Which bank is collaborating with the Himachal Pradesh government in implementing Doodh Ganga Yojna? Choose an option: (1). UCO Bank (2). State Bank of India (3). NABARD (4). H.P. state Cooperative Bank Correct option : 3 Government of India’s dairy business enterprise scheme is Doodh Ganga Yojana. This scheme offers fractional interest free credits & capital grant provisions to encourage planned dairy farming & generate service openings in Himachal Pradesh. The scheme is implemented through National Bank for Agriculture and Rural Development (NABARD), who will be the nodal agency for the scheme.

******ebook converter DEMO Watermarks*******

Q56. Which agency is managing Himachal Pradesh state wide Area networks (HIMSWAN) services? Choose an option: (1). M/s KPMG (2). M/s Orange Business services (3). M/s E&Y HPSDC (4). M/s Vyam Technologies Correct option : 2 With the launch of Himachal Pradesh State Wide Area Network (HIMSWAN) on 5th Feb, 2008 by Prof. Prem Kumar Dhumal, Chief Minister of Himachal Pradesh, Himachal Pradesh became the first State in the country to launch State Wide Area Network under National e-Governance Programme. The State entered an era of Hi-tech connectivity down to the level of tehsils, sub-tehsils, blocks and even Panchayats. M/s Orange Business Services is managing Himachal Pradesh State Wide Area Network (HIMSWAN) services.

******ebook converter DEMO Watermarks*******

Q57. Who among the following persons has been appointed as UP’s Lokayukta? Choose an option: (1). Prabhu Nath Singh (2). Sanjay Mishra (3). Virender Singh (4). N.K. Mehrotra Correct option : 2 Former judge of the Allahabad High Court, Justice (Retired) Sanjay Mishra, on Sunday took oath as the new Lokayukta of Uttar Pradesh. An apex court bench headed by Justice Ranjan Gogoi had appointed Justice Mishra as the Lokayukta.

******ebook converter DEMO Watermarks*******

Q58. Who among the following persons has secured India’s 12th Olympic quota berth for shooting for the Rio Games? Choose an option: (1). Napis Tortungpanich (2). Sanjay Rajput (3). Vitaliy Dovgun (4). Jitu Rai Correct option : 2 Sanjeev Rajput secured India’s 12 quota from shooting for the Rio Games by finishing fourth in men’s 50m rifle 3 positions at the Asian Olympic Qualifying Competition. Sanjeev qualified for the finals in fourth position shooting a score of 1163. There were three quota places available in the finals and the other two went to Kazakhstan’s Vitaliy Dovgun and Thailand’s 20-year old Napis Tortungpanich.

******ebook converter DEMO Watermarks*******

Q59. The National Airquality Index (AQI) measures air quality by taking into consideration which of the following pollutants? 1) Ammonia 2) Nitrogen dioxide 3) Sulphur dioxide 4) Carbon dioxide 5) Ozone 6) Lead Select the correct answer from the given codes: Choose an option: (1). (1), (3) and (5) only (2). (1), (2), (4) and (6) only (3). (1), (2), (3), (4), (5) and (6) (4). (1), (5) and (6) only Correct option : 3

******ebook converter DEMO Watermarks*******

Q60. Which is incorrect about Himalaya? Choose an option: (1). Himalaas are young folded mountains. (2). Before Himalayas, there was a Tethys sea. (3). Highest peak of the world is in Himalayan range (4). Its width varies from 200km in Kashmir to 500 km in Arunachal Pradesh. Correct option : 4 The Himalaya Mountains lie between the Indus River in the west and the Brahmaputra River in the east. The width of the Himalayas vary from 500 km in Kashmir to 200 km in Arunachal Pradesh.

Fig. Himalayas in Kullu district

******ebook converter DEMO Watermarks*******

Q61. Who of the following has been awarded Padma Vibhushan? Choose an option: (1). Avinash Dixit (2). Sania Mirza (3). Priyanka Chopra (4). Anjum Chopra Correct option : 1 Avinash Kamalakar Dixit is an Indian-American economist. He is currently John J. F. Sherrerd '52 University Professor of Economics Emeritus at Princeton University, Distinguished Adjunct Professor of Economics at Lingnan University, Senior Research Fellow at Nuffield College, Oxford and Sanjaya Lall Senior Visiting Research Fellow at Green Templeton College, Oxford. Born: 06 Aug 1944 (age 72) · Mumbai, India Awards: Padma Vibhushan (2016) · Guggenheim Fellowship for Social Sciences, US & Canada (1991) Education: Massachusetts Institute of Technology · University of Cambridge · University of Mumbai

******ebook converter DEMO Watermarks*******

Q62. Consider the following statements regarding IRNSS-1E: 1) It is India’s fifth navigational satellite 2) It was launched from PSLV-C31 3) It was successfully launched from Satish Dhawan Space Centre, SHAR, Sriharikota 4) It is an independent regional satellite system designed to provide information in the Indian region and 1500 km around the Indian Mainland Which of the above statements are correct? Choose an option: (1). (1), (2) and (3) (2). (2), (3) and (4) (3). (1), (2) and (4) (4). All of the above are correct Correct option : 4 IRNSS-1E is India’s fifth navigational satellite. It was launched from PSLV-C31. It was successfully launched from Satish Dhawan Space Centre, SHAR, Sriharikota. It is an independent regional satellite system designed to provide information in the Indian region and 1500 km around the Indian Mainland.

******ebook converter DEMO Watermarks*******

Q63. The Deen Dayal Upadhyaya Gram Jyoti Yojana: 1. Focuses on feeder separation (rural households and agricultural) 2. Seeks to electrify rural and urban areas both 3. Will nationalise all power distribution companies in rural areas 4. Seeks to provide uninterrupted power supply to industries set up in the rural areas Which of the above statement/s is/are correct? Choose an option: (1). Only 3 (2). Only 1 (3). Only 2 (4). All of the above Correct option : 2 The Deen Dayal Upadhyaya Gram Jyoti Yojana(DDUGJY) focuses on feeder separation (rural households & agricultural) and strengthening of sub-transmission & distribution infrastructure including metering at all levels in rural areas. This will help in providing round the clock power to rural households and adequate power to agricultural consumers.

******ebook converter DEMO Watermarks*******

Q64. Who among the following was the constituent advisor to the Constituent Assembly? Choose an option: (1). Dr. B.R. Ambedkar (2). Dr. Rajendra Prasad (3). K.M. Munshi (4). Sir. B.N.Rao Correct option : 4 BN Rao was appointed as the Constitutional Adviser to the Constituent Assembly in formulating the Indian Constitution in 1946. He was responsible for the general structure of its democratic framework of the Constitution and prepared its initial draft in February 1948.

******ebook converter DEMO Watermarks*******

Q65. Which one of the following festivals was most famous in Vijayanagar empire ? Choose an option: (1). Vasant Ritu (2). Brahmotsavam (3). Mahanavami (4). Ramana Vaidi Correct option : 3 An outstanding feature of life in the Vijayanagar empire was the great Mahanavami festival that took place each year at the end of the rainy season in September-October.

******ebook converter DEMO Watermarks*******

Q66. Which of the following states were not annexed to the English East India Company’s dominions under Dalhousies’s Doctrine of Lapse ? Choose an option: (1). Satara (2). Nagpur (3). Jhansi (4). Jodhpur Correct option : 4 The Doctrine of Lapse was applied by Dalhousie to Satara and it was annexed in 1848. Jhansi and Nagpur were annexed in 1854.

******ebook converter DEMO Watermarks*******

Q67. When was the Civil Disobedience Movement finally called off by Mahatma Gandhi? Choose an option: (1). 1931 (2). 1932 (3). 1933 (4). 1934 Correct option : 4 Civil Disobedience movement continued till April 1934 when Gandhi formally withdrawn it.

******ebook converter DEMO Watermarks*******

Q68. Consider the following: a. The Nehru Report b. The Rowlatt Satyagraha c. The MacDonald Award d. Formation of Congress Socialist Party Arrange the above in chronological order by using the codes given below : Choose an option: (1). (a), (b), (d), (c) (2). (b), (i), (c), (d) (3). (b), (d), (c), (i) (4). (c), (b), (d), (i) Correct option : 2

******ebook converter DEMO Watermarks*******

Q69. With regard to the recently approved "Pradhan Mantri Fasal Bima Yojana" Which of the following statement is/are incorrect? 1) For annual commercial and horticultural crops, the premium to be paid by farmers will be only 5%. 2) There will be a uniform premium of only 2% to be paid by farmers for all Kharif crops. 3) For Rabi crops, premium to be paid by farmers is 1.5%. Select the correct answer from the following codes: Choose an option: (1). (1) only (2). (2) only (3). (3) only (4). None of the above Correct option : 4 As per 'Pradhan Mantri Fasal Bima Yojana' : 1. For annual commercial and horticultural crops, the premium to be paid by farmers will be only 5%. 2. There will be a uniform premium of only 2% to be paid by farmers for all Kharif crops. 3. For Rabi crops, premium to be paid by farmers is 1.5%.

******ebook converter DEMO Watermarks*******

Q70. What has been the growth rate of GDP of India during 2014-15 at the current prices and constant and constant prices respectively? Choose an option: (1). 10.5% and 6.9 % (2). 10.5% and 7.3% (3). 9.5% and 7.3% (4). 11% and 7.3% Correct option : 2

******ebook converter DEMO Watermarks*******

Q71. Consider the following statements regarding Corporate Tax Avoidance deal : 1. A total of 31 Organisation for Economic CO-operation and Development (OECD) members agreed to sign this deal. 2. It will be harder for firms to hide money in tax havens or play one country’s tax authorities against another. 3. France is not among 31 members who agreed to sign the deal. Which of the above statements are correct? Choose an option: (1). Only (1) and (2) (2). Only (2) and (3) (3). Only (1) and (3) (4). All the above are correct Correct option : 1

******ebook converter DEMO Watermarks*******

Q72. Pradhan Mantri Fasal Bima Yojna was approved by Central Government on: Choose an option: (1). 13th January, 2016 (2). 23rd January, 2016 (3). 13th January, 2015 (4). 25th December,2016 Correct option : 1 The Pradhan Mantri Fasal Bima Yojana was launched by Prime Minister of India Narendra Modi on 13 January 2016. It envisages a uniform premium of only 2 per cent to be paid by farmers for Kharif crops, and 1.5 per cent for Rabi crops.

******ebook converter DEMO Watermarks*******

Q73. Swabhiman is a: Choose an option: (1). Financial inclusion initiative (2). Pension initiative for BPL families (3). Insurance Policy for Women (4). Rural Infrastructure Project Correct option : 1 Swabhimaan is a Unique Financial Inclusion Initiative. It is providing banking facilities across length and breadth of the country. It aims at providing branchless banking through the use of technology.

******ebook converter DEMO Watermarks*******

Q74. Which of the following is not a function of Reserve Bank of India? Choose an option: (1). Regulation of foreign exchange (2). Regulation of Credit (3). Banker of the government and Commercial Bank (4). Management of Fiscal Deficit Correct option : 4

******ebook converter DEMO Watermarks*******

Q75. As per HDR, 2014 the Gender Inequality Index of India is: Choose an option: (1). 127 (2). 120 (3). 137 (4). 122 Correct option : 1 As per HDR, 2014 India ranks 127th on gender inequality index and 114th on gender gap in the world.

******ebook converter DEMO Watermarks*******

Q76. The 14th Finance Commission has enhanced the states' share in the central divisible pool taxes from: Choose an option: (1). 30 percent to 42 percent (2). 32 percent to 42 percent (3). 32 percent to 40 percent (4). 31 percent to 41 percent Correct option : 2 The 14th Finance Commission has recommended a record 10% increase in the states' share in the Union taxes to 42% from 32%.

******ebook converter DEMO Watermarks*******

Q77. As per the Census 2011, the average age is above 30 years for the following states: Choose an option: (1). Kerala, Goa, Tamilnadu and HP (2). UP, Bihar, Meghalaya and WB (3). UP, Bihar, Meghalaya and Jharkhand (4). Kerala, Goa, UP and Bihar Correct option : 1

******ebook converter DEMO Watermarks*******

Q78. What is the name of shifting cultivation in Mexico ? Choose an option: (1). Milpa (2). Jhumming (3). Roca (4). Caingin Correct option : 1 Shifting cultivation is called by different names in different parts of the world, as: 1) Milpa in Central America and Mexico 2) Ray in Vietnam 3) Conuco in Venezuela

******ebook converter DEMO Watermarks*******

Q79. Which one of the following is incorrect? Choose an option: (1). Sariska Tiger - Rajasthan (2). Hemis National Park - J&K (3). Periyar National Park - Odisha (4). Rajaji National Park - Uttarakhand Correct option : 3 Periyar National Park, also known as Periyar Tiger Reserve, is in the mountainous Western Ghats of Kerala, southern India. This wildlife sanctuary is home to tigers and a significant elephant population, as well as rare liontailed macaques, sambar deer, leopards and Indian bison. Periyar is one of the 27 tiger reserves in India, s=covering an area of 925 square KMs.

******ebook converter DEMO Watermarks*******

Q80. Which one of the following is incorrect ? City - River Choose an option: (1). Jaunpur - Gomati (2). Nashik - Godavari (3). Lucknow - Gandak (4). Ujjain - Shipra Correct option : 3

******ebook converter DEMO Watermarks*******

Q81. Which lines separates India from Afghanistan ? Choose an option: (1). Radcliffe line (2). Durand line (3). McMohan line (4). None of the above Correct option : 1 India shares a 106-km border with Afghanistan along Jammu & Kashmir. Durand Line is the line demarcating the boundaries of India and Afghanistan. It was drawn up in 1896 by Sir Mortimer Durand.

******ebook converter DEMO Watermarks*******

Q82. Varkala beach is located in : Choose an option: (1). Karnataka (2). Puducherry (3). Kerala (4). Tamil Nadu Correct option : 3

******ebook converter DEMO Watermarks*******

Q83. Which one of the following is not correctly matched ? Country - Capital Choose an option: (1). Burkina - Faso Ouagadougou (2). Finland - Helsinki (3). Vietnam - Ho-Chi-Minh City (4). Taiwan - Taipei Correct option : 3

******ebook converter DEMO Watermarks*******

Q84. Which of the following statements is incorrect regarding Zika virus? Choose an option: (1). The virus was first isolated in April 1947 at Brazil (2). It is transmitted by daytime active Aedes mosquitoes such as A. aegypti (3). It is a member of the virus family flaviridae and the genus flavivirus (4). The most common symptoms of Zika arefever, rash, joint pain, muscle pain, headache, etc Correct option : 1 Zika virus is spread by daytime-active Aedes mosquitoes. Its name comes from the Zika Forest of Uganda, where the virus was first isolated in 1947.

******ebook converter DEMO Watermarks*******

Q85. What is the population density (per sq. km) of India and Himachal Pradesh respectively as per Census 2011? Choose an option: (1). 282 and 132 (2). 282 and 123 (3). 382 and 123 (4). 282 and 133 Correct option : 3 As per Census 2011, density of Himachal Pradesh is 123 per sq km which is lower than national average 382 per sq km.

******ebook converter DEMO Watermarks*******

Q86. Which of the following human rights come within the ambit of Article 21 of the Indian Constitution? 1) Right to shelter 2) Right to pure drinking water 3) Right to reputation 4) Right to privacy Choose an option: (1). 1 and 4 (2). 2 and 4 (3). 1, 3 and 4 (4). 1, 2, 3 and 4 Correct option : 4

Article 21 "No person shall be deprived of his life or personal liberty except according to a procedure established by law." Right to life and liberty includes: 1. Right against sexual harassment at workplace. 2. Right to privacy. 3. Right to education. 4. Right to free legal aid. 5. Right to reputation. 6. Right to shelter. 7. Right to healthy environment. This is applicable for both citizen and non-citizen.

******ebook converter DEMO Watermarks*******

Q87. During the term of office, criminal proceedings cannot be instituted against: Choose an option: (1). The President and the Governor (2). The President, the Vice-President and the Governor (3). The President, VP, Speaker of Lok Sabha and Governor (4). The President, the VP, the PM and the Governor Correct option : 1 During the term of office, criminal proceedings cannot be instituted against the President and the Governor.

******ebook converter DEMO Watermarks*******

Q88. Who among the following decides the question of disqualification of a member of Parliament on the grounds of defection? Choose an option: (1). The President of India (2). Election Commission (3). The Supreme court of India (4). Presiding Officer of the House Correct option : 4 The question as to whether a member of a House of Parliament or State Legislature has become subject to the disqualification will be determined by the presiding officer of the House.

******ebook converter DEMO Watermarks*******

Q89. Who among the following has the power to declare a caste or a tribe as scheduled caste or scheduled tribe? Choose an option: (1). President of India (2). Parliament (3). Home Minister (4). Chairman of the National Commission of SC or ST Correct option : 1 Constitution does not define as to who are the persons who belong to scheduled castes and scheduled tribes. However, Articles 341 and 342, empower the President of India to draw up a list of these castes and tribes.

******ebook converter DEMO Watermarks*******

Q90. Who among the following determines the area of a Lok Sabha constituency for the purpose of election? Choose an option: (1). President (2). Election Commission (3). Delimitation Commission (4). Census Commission Correct option : 3 The Delimitation Commission demarcates the boundaries of the Parliamentary Constituencies as per provisions of the Delimitation Act.

******ebook converter DEMO Watermarks*******

Q91. The Central Information Commission falls under: Choose an option: (1). The Ministry of Communications and Information Technology (2). The Ministry of Personnel (3). The Ministry of Law (4). The Ministry of Human Resource Correct option : 2 The Central Information Commission entertains complaints and appeals pertaining to offices, financial institutions, public sector undertakings, etc., under the Central Government and the Union Territories. It falls under the Ministry of Personnel.

******ebook converter DEMO Watermarks*******

Q92. Which of the following statements is correct about NITI Aayog? Choose an option: (1). It is a constitution body (2). It is a statutory body (3). It is neither a constitutional body nor a statutory body (4). It is a constitutional body Correct option : 3 NITI Aayog has replaced Planning Commission by a "Cabinet Resolution". Hence, it is neither a constitutional body nor a statutory body.

******ebook converter DEMO Watermarks*******

Q93. Which of the following committees recommended the creation of 'Nyaya Panchayats'? 1. Ashok Mehta Committee 2. LM Singhvi Committee 3. GVK Rao Committee 4. Balwant Rai Mehta Committee Select the correct answer form the codes given below: Codes: Choose an option: (1). 1 only (2). 1 and 2 (3). 2 and 3 (4). 1, 2 and 4 Correct option : 2

******ebook converter DEMO Watermarks*******

Q94. Which of the following are All India Services? 1. Indian Police Services 2. Indian Foreign Services 3. Indian Forest Services 4. Indian Revenue Services Select the correct answer: Codes: Choose an option: (1). 1 and 2 (2). 1 and 3 (3). 1, 2 and 4 (4). 1, 2, 3 and 4 Correct option : 2 The All-India Services are in addition to the Central and State Services. At present, there are three All-India Services. They are: Indian Administrative Service (IAS), Indian Police Service (IPS) and Indian Forest Service (IFS). Indian Foreign Service and Indian Revenue Service (Income Tax) comes under the Group A Central Services.

******ebook converter DEMO Watermarks*******

Q95. Who is the Chief Economic Advisor to the Government of India? Choose an option: (1). Raghuram Rajan (2). Arvind Panagariya (3). Arvind Subramanian (4). Y.V. Reddy Correct option : 3 Arvind Subramanian is an Indian economist and the current Chief Economic Adviser to the Government of India, having taken charge of the position on 16 October 2014 succeeding Raghuram Rajan.

******ebook converter DEMO Watermarks*******

Q96. Consider the following statements regarding International Solar Alliance (ISA) : (1) Indian Prime Minister Narender Modi and French President Francois Hollande has jointly laid the foundation stone of the International Solar Alliance Headquarters (2) The interim Secretariat of the ISA is located at Gurgaon. (3) ISA has 121 member-countries. Which of the above statements are correct? Choose an option: (1). Only (1) and (2) (2). Only (2) and (3) (3). Only (1) and (3) (4). All the above are correct Correct option : 4 Indian Prime Minister Narender Modi and French President Francois Hollande has jointly laid the foundation stone of the International Solar Alliance(ISA) Headquarters. The interim Secretariat of the ISA is located at Gurgaon. ISA has 121 member-countries.

******ebook converter DEMO Watermarks*******

Q97. Consider the following statements regarding Corruption Perception Index (CPI), 2015 : 1. The Corruption Perception Index, 2015 was released by Berlin-based corruption watch-dog transparency international. 2. India ranks 76th out of 194 countries in the CPI 2015. 3. The list is topped by Denmark followed by Finland and Sweden. Which of the above statements are correct? Choose an option: (1). Only (1) and (2) (2). Only (1) and (3) (3). Only (2) and (3) (4). All the above are correct Correct option : 2

******ebook converter DEMO Watermarks*******

Q98. Who among the following persons has won the Man Booker Prize, 2015? Choose an option: (1). Marlon James (2). Sanjeev Sahota (3). James Tott (4). Ali Smith Correct option : 1 Marlon James has won the Man Booker Prize, 2015. Hehas become the first Jamaican writer to win the Man Booker prize.

******ebook converter DEMO Watermarks*******

Q99. First Census was conducted in which year in India? Choose an option: (1). 1872 (2). 1891 (3). 1911 (4). 1951 Correct option : 1 The first census was conducted in India in 1872, but regular census started in 1881.

******ebook converter DEMO Watermarks*******

2014 HPAS Solved Paper This PDF Booklet contains 2014 HPAS Solved Paper. Explanation is provided by Shunya Foundation Team. 2014 HPAS Solved Paper with full explanation

******ebook converter DEMO Watermarks*******

Q1. The latitude that passes through Sikkim also passes through Choose an option: (1). Himachal Pradesh (2). Jammu and Kashmir (3). Rajasthan (4). Bihar Correct option : 3

Latitude of Sikkim is between 27N to 28N. The latitude that passes through Sikkim also passes through the following states: 1. Uttar Pradesh 2. Haryana 3. Rajasthan

******ebook converter DEMO Watermarks*******

Q2. Which of the following does not contain silver? Choose an option: (1). Horn Silver (2). German Silver (3). Ruby Silver (4). Lunar Silver Correct option : 2 Horn Silver - It is a very heavy halide mineral composed of silver chloride (AgCl). it is an ore of silver. German silver - It is a mixture of copper, zinc and nickel. Ruby Silver - It is a sulfosalt mineral consisting of silver sulfantimonide. It is an important source of the metal.

******ebook converter DEMO Watermarks*******

Q3. Aluminum surfaces are often ‘anodized’ this means the deposition of a layer of: Choose an option: (1). Chromium Oxide (2). Aluminum Oxide (3). Nickel Oxide (4). Zinc Oxide Correct option : 2 Anodising is essentially a process where a thick film of aluminum oxide is built up on the surface of the aluminum through the use of a direct current electrical supply. Aluminum is often anodized for decorative purposes (coloring), to harden the surface or to increases its resistance towards atmospheric corrosion.

******ebook converter DEMO Watermarks*******

Q4. When a CD (compact disc used in audio and video systems) is seen in sunlight, rainbow like colours are seen. This can be explained on the basis of the phenomenon of: Choose an option: (1). Reflection and diffraction (2). Reflection and transmission (3). Diffraction and transmission (4). Refraction, diffraction and transmission Correct option : 4

The reason CDs reflect rainbow colors is because they have a clear plastic coating on top of a mirrorized surface. Light refracts (bends) when it moves from one medium to another with a different optical density. Every color has a different wave length. Different wave lengths of light travel at different speeds, so that full spectrum appears when white light passes from the air through the plastic surface of a CD, separated light rays which are then reflected back to us by the mirrorized center surface of a CD. Here the diffraction and transmission also takes place because diffraction of light rays occur when it strikes the surface of CD and transmission is obvious when light enters from one medium to another.

******ebook converter DEMO Watermarks*******

Q5. Domestic electrical wiring is basically a: Choose an option: (1). Series connection (2). Parallel connection (3). Combination of series and parallel connection (4). Series connection within each room and parallel connection elsewhere Correct option : 2

Fig. Domestic wiring Parallel arrangement used in domestic wiring because of the following : (i) All the appliances works at the same voltage as that of the electric supply. (ii) If one of the appliances is out of order, e.g. if a bulb get fused, all other appliances keep on working as the circuit is not broken in parallel arrangement of devices.

******ebook converter DEMO Watermarks*******

Q6. The only snake that builds a nest is: Choose an option: (1). Chain viper (2). King Cobra (3). Krait (4). Saw-scaled viper Correct option : 2

Fig. King Cobra King Cobra is the only snake in the world that builds a nest. It is an oviparous snake and lays its eggs in the nest and guards the nest until they are hatched.

******ebook converter DEMO Watermarks*******

Q7. The hormone insulin is a: Choose an option: (1). Glycolipid (2). Fatty acid (3). Peptide (4). Sterol Correct option : 3

Peptide hormones are those made up from polypeptide chains. Peptide hormones can either have short polypeptide chains, such as in oxytocin, or longer protein chains such as those in growth factors. Peptides that are to be secreted are stored with in vesicles inside the cell. Their release is initiated in response to stimuli. An example of one of these proteins is insulin, it is released in response to rising blood glucose levels. The body needs insuline for the correct use of food and energy. All hormones are made by glands. The gland that makes insulin is the pancreas. People with diabetes don’t make enough insulin. Luckily, we have a way to replace the insulin the pancreas can’t make. Insulin is also a protein. It can’t be taken by mouth. It would be digested, just like the food you eat. That is why insulin is taken by injection.

******ebook converter DEMO Watermarks*******

Q8. The normal temperature of human body in Kelvin scale is: Choose an option: (1). 280 (2). 290 (3). 300 (4). 310 Correct option : 4

Fig. Thermometer The normal temperature of human body is 37°C. After converting, 37 degrees Celsius is approximately equal to 310 kelvins.

Kelvin to Celsius Formula T(k) = T(c) + 273.15

******ebook converter DEMO Watermarks*******

Q9. Which one of the following polymers is widely used for making bullet proof material? Choose an option: (1). Polyvinyl chloride (2). Polyamides (3). Polyethylene (4). Polycarbonates Correct option : 4 Bullet-resistant glass is usually constructed using polycarbonate, thermoplastic, and layers of laminated glass. The aim is to make a material with the appearance and clarity of standard glass but with effective protection from small arms. Bullet proof glass is made by layering a polycarbonate material between pieces of ordinary glass in a process called lamination.

******ebook converter DEMO Watermarks*******

Q10. Name the site where Harappan ploughed field has been found: Choose an option: (1). Harappa (2). Banawali (3). Rakhigarhi (4). Kalibanga Correct option : 4

Kalibangan is located at Pilibangan, between Suratgarh and Hanumangarh in Hanumangarh district of Rajasthan. Archaeologists have found evidence of a ploughed field here, associated with Early Harappan levels. The field had two sets of furrows at right angles to each other, suggesting that two different crops were grown together.

******ebook converter DEMO Watermarks*******

Q11. Name the place where Jaina texts were finally complied. Choose an option: (1). Pawagrah (2). Kalinga (3). Sharvanbelgola (4). Vallabhi Correct option : 4 The teachings of Mahavira and his followers were transmitted orally for several centuries. They were written down in the form in which they are presently available at a place called Valabhi, in Gujarat, about 1500 years ago.

******ebook converter DEMO Watermarks*******

Q12. Which of the following Marathas was given a Manasab of 7000/7000 by Aurangzeb? Choose an option: (1). Shivaji (2). Sambhaji (3). Rajaram (4). Shahuji Correct option : 4 In this system of mansabdari, the emperor assigned mansabs or ranks, ranging from 10 to 10,000 and above, to the functionaries who administered the realm, maintained and led its armed forces, and pushed its boundaries forward. Aurangzeb gave a Manasab of 7000/7000 to Shahuji.

******ebook converter DEMO Watermarks*******

Q13. Who among the following translated Kalhana’s Rajatarangini into English? Choose an option: (1). Sir Alexander Cunningham (2). Sir Aurel Stein (3). Jonaraja (4). Sirivara Correct option : 2 Rajatarangini is a metrical legendary and historical chronicle of the north-western Indian subcontinent, particularly the kings of Kashmir. It was written in Sanskrit by Kashmiri historian Kalhana in the 12th century. Sir Aurel Stein translated Kalhana’s Rajatarangini into English.

******ebook converter DEMO Watermarks*******

Q14. Sidhu and Kanhu were associated with which one of the following? Choose an option: (1). The Kol uprising (2). The Munda rebellion (3). The Khasi Rising (4). The Santhal rebellion Correct option : 4 Among the numerous tribal revolts, the Santhal uprising was the most massive. By 1854, it was decided to raise the banner of revolt, get rid of the outsiders and their colonial masters. Sido and Kanhu, the principal rebel leaders of Santhal uprising, claimed that Thakur (God) had communicated with them and told them to take up arms and fight for independence.

******ebook converter DEMO Watermarks*******

Q15. NITI in NITI Aayog stands for Choose an option: (1). National Institution for Transforming India (2). National Institute of Transforming India (3). National Initiative for Transforming India (4). National Institution for Translating India Correct option : 1 NITI Aayog or the National Institution for Transforming India is a Government of India policy think-tank established by the Narendra Modi government to replace the Planning Commission which followed the top-down model. The stated aim for NITI Aayog’s creation is to foster involvement and participation in the economic policy-making process by the State Governments of India. The emphasis is on bottom-up approach and make the country to move towards cooperative federalism. The Union Government of India announced the formation of NITI Aayog on 1 January 2015, and the first meeting was held on 8 February 2015. The Prime Minister serves as the Ex-officio chairman.

******ebook converter DEMO Watermarks*******

Q16. Which of the following is/are treated as artificial currency? Choose an option: (1). ADR (2). SDR (3). GDR (4). Both ADR and GDR Correct option : 2 Special drawing rights (SDR) is essentially an artificial currency and a substitute for an actual currency that is used in economic transactions. SDR refer to an international type of monetary reserve currency created by the International Monetary Fund (IMF) in 1969 that operates as a supplement to the existing reserves of member countries.

******ebook converter DEMO Watermarks*******

Q17. SEBI was established in the year Choose an option: (1). 1988 (2). 1990 (3). 1991 (4). 1985 Correct option : 1 The Securities and Exchange Board of India (SEBI) is the regulator for the securities market in India. It was established in the year 1988 and given statutory powers on 30 January 1992 through the SEBI Act, 1992.

******ebook converter DEMO Watermarks*******

Q18. The working principal of a washing machine is Choose an option: (1). Centrifugation (2). Dialysis (3). Reverse osmosis (4). Diffusion Correct option : 1 Centrifugation Centrifugal force is the force that seems to make things shoot outward when they go round in a circle. The washing machine dryer consists of a perforated cylindrical vessel which can rotate about its axis at a very high seed. he clothes to be dried are placed in the cylinder ans are rotated with high speed. The adhesive force between water and the cloth is less compared o the centrifugal force acting on the water molecules. his centrifugal force, forces the water molecules to escape through the holes of the cylindrical vessel. So, when your clothes are spinning in the drum, it’s centrifugal force that throws the water out through the little holes so your washing ends up much drier. Applications 1. Spinner of washing machine works on this principle. 2. Separating chalk powder from water. 3. Removing Fat from milk to produce skimmed milk. 4. The clarification and stabilization of wine. 5. Separation of urine components and blood components in forensic and research laboratories.

******ebook converter DEMO Watermarks*******

Q19. Which of the following drugs reduces fever? Choose an option: (1). Analgesic (2). Antipyretic (3). Antibiotic (4). Tranquilizer Correct option : 2 Analgesics reduce or abolish pain without causing impairment of consciousness, mental confusion, incoordination or paralysis or some other disturbances of nervous system. These are classified as follows: (i) Non-narcotic (non-addictive) analgesics (ii) Narcotic drugs (i) Non-narcotic (non-addictive) analgesics: Aspirin and paracetamol belong to the class of non-narcotic analgesics. Aspirin is the most familiar example. Aspirin inhibits the synthesis of chemicals known as prostaglandins which stimulate inflammation in the tissue and cause pain. These drugs are effective in relieving skeletal pain such as that due to arthritis. These drugs have many other effects such as reducing fever (antipyretic) and preventing platelet coagulation. Because of its anti blood clotting action, aspirin finds use in prevention of heart attacks.

******ebook converter DEMO Watermarks*******

Q20. Mars Orbitor Mission is called: Choose an option: (1). Pushpyan (2). Spaceyan (3). Mangalyan (4). Missionyan Correct option : 3 The Mars Orbiter Mission (MOM), also called Mangalyaan is a space probe orbiting Mars since 24 September 2014. It was launched on 5 November 2013 by the Indian Space Research Organisation (ISRO). It is India’s first interplanetary mission and ISRO has become the fourth space agency to reach Mars, after the Soviet space program, NASA, and the European Space Agency. It is the first Asian nation to reach Mars orbit, and the first nation in the world to do so in its first attempt.

******ebook converter DEMO Watermarks*******

Q21. In which district of HP is Yunam-so Lake? Choose an option: (1). Lahaul-Spiti (2). Kinnaur (3). Chamba (4). Kullu Correct option : 1

The Yunam-so Lake is situated in Lahul and Spiti district at an altitude of 4300m forming a valley system between Himalayas and Zaskar ranges. For its beautiful vegetation and beautiful flora and fauna, the place is famous amongst the tourists. The deep blue color of the water mesmerizes the viewer with its intense beauty.

******ebook converter DEMO Watermarks*******

Q22. What is Shingni-Mingni? Choose an option: (1). A variety of fish (2). A herbal-medicinal plant (3). A folk dance of Kinnaur (4). A folk song popular in Sirmaur Correct option : 2 Shingni Mingni is a common medicinal herb used in Lahaul Spiti region.

******ebook converter DEMO Watermarks*******

Q23. According to Planning Commission estimates for 2011-12 which of the following states have maximum percentage of population below poverty line Choose an option: (1). Chhattisgarh (2). Bihar (3). Jharkhand (4). Odisha Correct option : 1

******ebook converter DEMO Watermarks*******

Q24. Diego Garcia Island is located in Choose an option: (1). Pacific Ocean (2). Atlantic Ocean (3). Indian Ocean (4). Arctic Ocean Correct option : 3 Diego Garcia is an atoll just south of the equator in the central Indian Ocean.

******ebook converter DEMO Watermarks*******

Q25. Great Salt Lake is located in: Choose an option: (1). Australia (2). Tunisia (3). Canada (4). U.S.A. Correct option : 4 The Great Salt Lake is located in the northern part of the state of Utah, USA. It is the largest salt water lake in the Western Hemisphere. Its high salinity is caused by the accumulation of minerals with no removal and the accompanying water evaporation.

******ebook converter DEMO Watermarks*******

Q26. Tropic of cancer does not pass through Choose an option: (1). Tripura (2). Mizoram (3). Manipur (4). Chhattisgarh Correct option : 3

The Tropic of Cancer is the circle marking the latitude 23.5 degrees north, where the sun is directly overhead at noon on June 21, the beginning of summer in the northern hemisphere. The states of India that the Tropic of Cancer passes through are Gujarat, Rajasthan, Madhya Pradesh, Chhattisgarh, Jharkhand, West Bengal, Tripura and Mizoram.

******ebook converter DEMO Watermarks*******

Q27. Which is the second most spoken language in India? Choose an option: (1). Urdu (2). Hindi (3). Bengali (4). Telugu Correct option : 3 Bengali is an Indo-Aryan language spoken in Indian Subcontinent. With over 250 million speakers, Bengali is the seventh most spoken native language in the world. Today, Bengali is the primary language spoken in Bangladesh and the second most widely spoken language in India.

******ebook converter DEMO Watermarks*******

Q28. Who among the following is the author of Milind Panho? Choose an option: (1). Nagasen (2). Ashwaghosh (3). Dharmpal (4). Devardhi Correct option : 1 The Milinda Panha is a Buddhist text written by Nagasen which dates from approximately 100 BCE. It explains the Buddhist doctrines in the form of a dialogue between Milind and his teacher Nagsena (the great Buddhist philosopher ) is in Pali Language.

******ebook converter DEMO Watermarks*******

Q29. Find out among the following, the author of Muasir-i-Alamgiri? Choose an option: (1). Alberuni (2). Barni (3). Mustaid Khan (4). Sams-i-Siraj Correct option : 3 Mustaid Khan was in the service of Aurangazeb for forty years. He was an eyewitness to many of the events record by him. Ma’asir-i-Alamgiri was written by him after Aurangazeb’s death.

******ebook converter DEMO Watermarks*******

Q30. Polworth, Corriedole, Blackpace and Bingi are varieties of: Choose an option: (1). Vegetables (2). Spices (3). Sheep (4). Buffaloes Correct option : 3

Fig. A white sheep The sheep is a ruminant mammal typically kept as livestock. A sheep’s wool is the most widely used animal fiber and meat. Various varieties of sheep are Polworth, Corriedole, Blackpace and Bingi.

******ebook converter DEMO Watermarks*******

Q31. Who was the head of the boundary commission which demarcated the boundaries of India and Pakistan? Choose an option: (1). Redcliffe (2). McMahon (3). Simon (4). None of the above Correct option : 1

On 17th August 1947 the Radcliffe Line was declared as the boundary between India and Pakistan, following the Partition of India. The line is named after Sir Cyril Radcliffe who was commissioned to equitably divide 4,50,000 km sq of territory with 88 million people. The idea behind the Radcliffe Line was to create a boundary which would divide India along religious demographics, under which Muslim majority provinces would become part of the new nation of Pakistan and Hindu and Sikh majority provinces would remain in India. Today its western side still serves as the Indo-Pakistani border and the eastern side serves as the India-Bangladesh border.

******ebook converter DEMO Watermarks*******

Q32. What is the local name of shifting cultivation in Malaysia? Choose an option: (1). Jhum (2). Kumri (3). Podu (4). Ladang Correct option : 4 Shifting cultivation is an agricultural system in which plots of land are cultivated temporarily, then abandoned and allowed to revert to their natural vegetation while the cultivator moves on to another plot. Shifting cultivation is called as :- Jhumming in north-east India, Milpa in Mexico, Roca in Brazil and Ladang in Malaysia.

******ebook converter DEMO Watermarks*******

Q33. When was ‘Beti Hai Anmol’ Yojna launched in Himachal Pradesh? Choose an option: (1). January 2009 (2). July 2010 (3). October 2011 (4). December 2012 Correct option : 2

Fig. Beti Hai Anmol Beti Hai Anmol Yojna - The programme was launched in Himachal Pradesh 05 July, 2010. This scheme is for girls of BPL families up to two girl children. After their birth, the department deposits rupees 10,000 per girl child in the post office/Bank account. These girls get scholarship ranging from Rs. 300 to Rs.1200 from first to 12th Class for their books/dresses etc.

******ebook converter DEMO Watermarks*******

Q34. Disguised unemployment generally means: Choose an option: (1). Large number of people remains unemployed (2). Alternative employment is not available (3). Marginal productivity is zero (4). Productivity of worker is low Correct option : 3

Fig. Farmer in the field The Disguised Unemployment refers to the work area where surplus manpower is employed out of which some individuals have zero or almost zero marginal productivity such that if they are removed the total level of output remains unchanged. It is unemployment that does not affect aggregate output. The disguised unemployment is very much prevalent in the under developing and unorganized sectors of the economy, such as agriculture, where the population pressure is high, and the job opportunities are few. It is very difficult to track the disguised unemployment as it cannot be measured and shown in the official records of unemployment.

******ebook converter DEMO Watermarks*******

Q35. Which of the following amendments to the Indian Constitution has made the right of education a fundamental right? Choose an option: (1). 84th (2). 86th (3). 91st (4). 92nd Correct option : 2 The 86th Amendment Act, 2002 inserted a new article 21A below the Article 21 which made Right to Education a Fundamental Right for children in the range of 6-14 years.

******ebook converter DEMO Watermarks*******

Q36. Which of the following are the exclusive committees of Rajya Sabha? (i) Committee on Subordinate legislation (ii) Committee on government assurances (iii) Committee on welfare of scheduled castes and scheduled tribes (iv) Committee on public undertakings Select the correct answer from the codes given below: Choose an option: (1). (1) and (3) (2). (1) and (2) (3). (1), (2) and (3) (4). All of the above Correct option : 2 The Committee on the Welfare of Scheduled Castes and Scheduled Tribes selects such subjects pertaining to the welfare of Scheduled Castes and Scheduled Tribes as may deem fit to the Committee. It consists of 30 Members— 20 elected by Lok Sabha from amongst its Members. Committee on Public Undertakings examines the reports and accounts of the PSUs and the CAG audit reports related to PSUs. It has 22 members from both the houses of parliament {15 from Lok Sabha and 7 from Rajya Sabha}.

******ebook converter DEMO Watermarks*******

Q37. The elected members of the state legislative assembly participate in the elections of (1) The President of India (2) The Vice President of India (3) Members of Rajya Sabha (4) Member of Legislative Council Choose an option: (1). (1), (2) and (3) (2). (1), (3) and (4) (3). (1), (2) and (4) (4). (1), (2), (3) and (4) Correct option : 2 The Vice-President is elected by an Electoral College, which consists of the members of the Lok Sabha and Rajya Sabha (both elected and nominated members).

******ebook converter DEMO Watermarks*******

Q38. Which of the Commissions/Committees has recommended that there should be Minister of Panchayati Raj? Choose an option: (1). Ashok Mehta Committee (2). L M Singhavi Committee (3). G V K Rao Committee (4). Second Administrative Reforms commission Correct option : 1 In December 1977, the Janta Government appointed a committee on Panchayati Raj institutions under the chairmanship of Ashok Mehta. It submitted its report in August 1978 and made 132 recommendations to revive and strengthen the declining Panchayati Raj System in the country. Its main recommendations are: 1. The 3-tier system of Panchayati Raj should be replaced by the 2-tier system: Zilla Parishad at the district level, and below it,the Mandal Panchayat consisting of a group of villages covering a population of 15000 to 20000. 2. A district should be the first point for decentralisation under popular supervision below the state level. 3. Zila Parishad should be the executive body and made responsible for planning at the district level. 4. A minister for Panchayati Raj should be appointed in the state council of ministers to look after the affairs of the Panchayati Raj institutions.

******ebook converter DEMO Watermarks*******

Q39. Pradhan Mantri Jan Dhan Yojna was launched on Choose an option: (1). 15th August, 2014 (2). 1st January, 2015 (3). 26th January, 2015 (4). 28th August 2014 Correct option : 4 Pradhan Mantri Jan-Dhan Yojana - Pradhan Mantri Jan-Dhan Yojana(PMJDY) is India’s National Mission for Financial Inclusion to ensure access to financial services, namely Banking Savings & Deposit Accounts, Remittance, Credit, Insurance, Pension in an affordable manner. This financial inclusion campaign was launched by the Prime Minister of India Narendra Modi on 28 August 2014.

******ebook converter DEMO Watermarks*******

Q40. Which of the following has the lowest sex ratio in India? Choose an option: (1). Punjab (2). Haryana (3). Dadar and Nagar Haveli (4). Daman and Div Correct option : 4 Sex ratio is used to describe the number of females per 1000 of males. In the Population Census of 2011, it was revealed that the population ratio of India 2011 is 940 females per 1000 of males. Kerala had the highest sex ratio(1084), while Daman and Diu had least seax ratio(710).

******ebook converter DEMO Watermarks*******

Q41. Which of the following statements is/are correct ? (i) Recorded temperature decreases from North to South India (ii) Rainfall increases from West to East in Northern Plains of India Choose an option: (1). Only (i) (2). Only (ii) (3). Both (i) and (ii) (4). Neither (i) nor (ii) Correct option : 2 The cold weather season begins from mid-November in northern India and stays till February. December and January are the coldest months in the northern part of India. The temperature decreases from south to the north. However, rainfall increases from West to East in Northern Plains of India.

******ebook converter DEMO Watermarks*******

Q42. Which of the following is the largest coal producer? Choose an option: (1). China (2). U.S. A (3). India (4). Australia Correct option : 1

Coal has many important uses worldwide. The most significant uses of coal are in electricity generation, steel production, cement manufacturing and as a liquid fuel. Largest coal producer in the world is China.

******ebook converter DEMO Watermarks*******

Q43. Which of the following is the correct order of rivers from north to south? Choose an option: (1). Jhelum, Chenab, Ravi, Satluj (2). Jhelum, Ravi, Chenab, Satluj (3). Ravi, Jhelum, Chenab, Satluj (4). Ravi, Chenab, Jhelum, Satluj Correct option : 1 The order of rivers in India from north to south is Indus, Jhelum, Chenab , Ravi and Satluj.

******ebook converter DEMO Watermarks*******

Q44. Which Mughal ruler is said to have given the title of ‘Mian’ to the Rajput princes of Punjab Hill states who were held hostages in the royal court? Choose an option: (1). Akbar (2). Jahangir (3). Shah Jahan (4). Aurangzeb Correct option : 2 Mian Rajput are the ruling Rajput clans of Jammu & Kashmir and Himachal Pradesh State of India. Mian Rajput consider themselves to be of highest class. They do not engage in trade or agriculture. They are major land owners cultivated by other castes. Some mians are Jamwals, Katoch, Chandel and Dogras. The title Mian was originally given by Emperor Jahangir to the 22 Rajput Princes from the Punjab Hill States, who were hostages at the Mughal Court.

******ebook converter DEMO Watermarks*******

Q45. Ruler of which of the following hill states were from a common ancestor? Choose an option: (1). Mandi, Mangal and Bhajji (2). Suket, Mandi and Bhagal (3). Mandi, Suket and Keonthal (4). Keonthal, Suket and Jubbal Correct option : 3 The rulers and chiefs of Mandi, Suket, Keonthal and Kishtawar claim descent from a common ancestor of the Sena dynasty of Bengal, of Chandrabansi rajputs and from the Pandvas of the Mahabharta. The descendants of the common ancestors were three brothers, on of them named Vir or Bir sen, became the ruler of Suket. Among Bir sen’s descendants were two brothers who ruled in about 1000 AD and were named Sahu sen and Bahu sen.

******ebook converter DEMO Watermarks*******

Q46. Who started the Ice-Skating Rink at Shimla around 1920? Choose an option: (1). Peter Ta-Tung (2). Blessington (3). Daniel Wilson (4). William Hay Correct option : 2 The Ice skating Rink was established in 1920 by Mr. Blessington, an English gentle man. Shimla Ice Skating Club was the first to be established in India and the only of its kind in whole of South East Asia. During the British era the Club only permitted Europeans to be its members, later however with certain reservations, a few selected Indians were also included as members. After the dawn of independence it was rendered open to all sports loving persons, irrespective of caste, color and creed.

******ebook converter DEMO Watermarks*******

Q47. What was the annual growth rate in the economy of Himahcal Pradesh during the 2012-13 fiscal? Choose an option: (1). 5.7 (2). 5.9 (3). 6.1 (4). 6.6 Correct option : 3

Fig. Himachal Pradesh - growth rate in the economy The economy of Himachal Pradesh has transformed rapidly from the most backward State of India to one of the most advanced State. The pace of such transformation has emerged Himachal Pradesh as a leader in Hill Area Development. The annual growth rate in the economy of Himahcal Pradesh during the 2012-13 fiscal was 6.1.

******ebook converter DEMO Watermarks*******

Q48. Which one of the following is a Pagoda style temple in HP? Choose an option: (1). Hateshwari Devi in Jubbal Valley (2). Hidimba Temple in Manali Valley (3). Lakshna Devi in Bharmaur (4). Shakti Devi in Chhatrari Correct option : 2

Hadimba Temple is located in Manali, Distt Kullu. It is an ancient temple dedicated to Hadimba Devi, wife of the strongest of the Pandava brothers, Bhima. The temple is surrounded by a cedar forest at the foot of the Himalayas. This temple was constructed by Raja Bahadur Singh in Pagoda style. There are many wooden carving works in the walls and main entrance door of the temple.

******ebook converter DEMO Watermarks*******

Q49. Identify the correct order of density of population among the following districts of HP according to 2011 census data (in descending order): Choose an option: (1). Bilaspur, Una, Solan, Mandi (2). Una, Bilaspur, Solan, Mandi (3). Una, Mandi, Bilaspur, Solan (4). Bilaspur, Solan, Una, Mandi Correct option : 2 According to 2011 census data, the order of density of population among the districts of HP (in descending order) is as: 1. Hamirpur 2. Una 3. Bilaspur 4. Solan 5. Kangra 6. Mandi 7. Sirmaur 8. Shimla 9. Chamba 10. Kullu 11. Kinnaur 12. Lahaul and Spiti

******ebook converter DEMO Watermarks*******

Q50. Which district of HP occupies top position in the production of ginger? Choose an option: (1). Sirmaur (2). Bilaspur (3). Una (4). Mandi Correct option : 1

Ginger is the major spice crop of Himachal Pradesh. Sirmaur district of HP occupies top position in the production of ginger. Bella valley in Sirmaur region is famous for its different varieties of ginger. About 90% of the ginger produced in the state is exported to the adjoining states.

******ebook converter DEMO Watermarks*******

Q51. India signed an agreement on transfer of sentenced persons with the Hong-Kong special administrative region in 2014. Which among the following statements is/are correct ones in this regard? (i) The agreement will mostly apply to Hong-Kong prisoners. (ii) Most of the Indian Prisoners have been jailed for serious crimes like drug dealing. (iii) India has similar agreement with several other countries but none with Mainland China. The Right answer is: Choose an option: (1). (i) (2). (ii) (3). (iii) (4). All of the above Correct option : 3 India on 20 January 2015 signed an agreement on transfer of sentenced persons with Hong Kong Special Administrative Region. Under the agreement, the two sides will exchange sentenced prisoners in each other’s jurisdiction. The agreement will mostly apply to Indian prisoners as there are very few Hong Kong residents in Indian prisons. There are at present 93 Indians in Hong Kong jails. Only 17 of them are undergoing sentence for serious crimes like drug dealing while most of the remaining prisoners have been jailed for relatively minor offenses like immigration law violation. India has similar agreements on exchange of prisoners with several other countries but none with Mainland China.

******ebook converter DEMO Watermarks*******

Q52. Based upon the relations between India and Pakistan, Khurshid M. Kasuri, the Ex-foreign Minister of Pakistan has recently penned down a book. What is the title of the book? Choose an option: (1). India-Pakistan in War and Peace (2). Neither a Hawk nor a Dove (3). The Origins of War in South Asia: The Indo- Pakistan Conflicts Since 1947 (4). India and Pakistan: A Political Analysis Correct option : 2 Khurshid Mahmood Kasuri, Ex-Foreign Minister of Pakistan, authored a book titled "Neither a Hawk nor a Dove". The book is based upon the relations between India and Pakistan. He writes that the two countries have no option but to go for a just and peaceful solution to Jammu and Kashmir and other contentious issues between them.

******ebook converter DEMO Watermarks*******

Q53. Which of the following statements are correct about the National Human Right Commission? (i) The National Human Rights Commission is an independent body (ii) It is not a constitutional body (iii) It is a statutory body (iv) It is a multi-member body Select the correct answer from the code given below: Choose an option: (1). (i) and (ii) (2). (i), (ii) and (iii) (3). (i), (ii) and (iv) (4). All of the above Correct option : 4 National Human Rights Commission of India is an autonomous/independent public body constituted on 12 October 1993 under the Protection of Human Rights Ordinance of 28 September 1993. It is non-Constitutional (statutory) body established under Protection of Human Rights Act, 1993. It is a multi-member body with following members: 1. Chairman - Retired Chief Justice of India (CJI). 2. Members (4 in number) 3. Four ex-officio members

******ebook converter DEMO Watermarks*******

Q54. Which of the following agriculture practices is eco-friendly? Choose an option: (1). Organic farming (2). Shifting cultivation (3). Cultivation of high yielding (4). Growing plants in glass-houses Correct option : 1 Eco friendly agriculture practices describe landscapes that support both agricultural production and biodiversity conservation, working in harmony together to improve the livelihoods of rural communities. Organic farming is eco friendly agriculture practice. It is an integrated farming system that strives for sustainability, the enhancement of soil fertility and biological diversity whilst, with rare exceptions, prohibiting synthetic pesticides, antibiotics, synthetic fertilizers, genetically modified organisms, and growth hormones.

******ebook converter DEMO Watermarks*******

Q55. According to the Wealth Health Organization, the desease which causes the death of the largest number of people today is Choose an option: (1). AIDS (2). Tuberculosis (3). Malaria (4). Ebola Correct option : 1 Acquired immunodeficiency syndrome (AIDS) is the leading infectious cause of adult death in the world. Untreated disease caused by the human immunodeficiency virus (HIV) has a case fatality rate that approaches 100%.

******ebook converter DEMO Watermarks*******

Q56. The Raja of Bushahar princely state was assisted by three hereditary Wazir families. One of them was Kohal. Where did it come from? Choose an option: (1). Kullu (2). Garhwal (3). Kinnaur (4). Jubbal Correct option : 2 In the Bushahar state, next to Raja were three Kanait hereditary wazir families, named Pawari, Kohal ans Shua. Kohals were from Garhwal region.

******ebook converter DEMO Watermarks*******

Q57. When was the Department of Public Relations created in Himachal Pradesh? Choose an option: (1). 1951 (2). 1952 (3). 1954 (4). 1956 Correct option : 3 The Department of Information and Public Relations since its Inception in 1954 has passed through various stages and has done its best to equip itself with what was essentially required to meet the challenges of times and thus make its presence felt through the optimum use of its potential. The main function of department is to disseminate the policies and programmes of the Government in the right perspective and to gauge and assess the reactions of the quarter concerned and keep informed the Government about their impact amongst the masses.

******ebook converter DEMO Watermarks*******

Q58. In India, which of the following have the highest share in the disbursement of credit to agriculture and allied activities? Choose an option: (1). Commercial Banks (2). Micro Finance Institutions (3). Regional Rural Banks (4). Co-operative Banks Correct option : 1 Agriculture and allied activities constitute the single largest contributor to the Gross Domestic Product (GDP), accounting for almost 33% of the total. They, are vital to the national well-being as, besides providing the basic needs of the society and the raw materials for some of the important segments of Indian industry. Credit is a catalyst that lubricates the process to accelerate the farm and non-farm sector development including rural industrialization, business and service segments of the economy. Credit helps farmers invest in creating assets to generate output and income through deploying science, technology and modern methods of business management. In India, Commercial Banks have the highest share in the disbursement of credit to agriculture and allied activities.

******ebook converter DEMO Watermarks*******

Q59. The World Investment Report (WIR) is published by Choose an option: (1). International Monetary Fund (2). United Nations Conference on Trade and Development (UNCTAD) (3). General Agreement on Tariffs and Trade (GATT) (4). Organization of Economic Cooperation and Development (OECD) Correct option : 2 The World Investment Report has been published annually since 1991 by United Nations Conference on Trade and Development (UNCTAD). The Report focuses on trends in foreign direct investment (FDI) worldwide, at the regional and country levels and emerging measures to improve its contribution to development.

******ebook converter DEMO Watermarks*******

Q60. What is the correct sequence of the following four stages in the public policy formulation process? (i) Making policy decisions (ii) Putting public problems on policy agenda (iii) Identifying public problems (iv) Formulating policy proposals to deal with the problem Select the correct answer from the code given below: Choose an option: (1). (i), (ii), (iii) and (iv) (2). (iii), (ii), (iv) and (i) (3). (i), (iii), (ii) and (iv) (4). (ii), (iv), (iii) and (i) Correct option : 2 Public Policy is a document that contains the broad outline as well as the detailed description of formulation as well as implementation of various govt. programmes and plans that are taken out for the goal/objective of public benefit. It is implemented through the constitutional authorities, bureaucracy and government organisations/institutions in collaboration with civil society organisations. The sequence of various stages in the public policy formulation process are: (1) Identifying public problems (2) Putting public problems on policy agenda (3) Formulating policy proposals to deal with the problem (4) Making policy decisions

******ebook converter DEMO Watermarks*******

Q61. Annual Summit 2015 of BRICS nations was held in Choose an option: (1). New Delhi, India (2). Fortaleza, Brazil (3). Ufa in Bashkortostan, Russia (4). Durban, South Africa Correct option : 3 BRICS is the acronym for an association of five major emerging national economies: Brazil, Russia, India, China and South Africa. The 7th BRICS summit was held in the Russian city of Ufa in Bashkortostan on 8–9 July 2015.

******ebook converter DEMO Watermarks*******

Q62. The most widely used antacid is: Choose an option: (1). Cetrizine (2). Iproniazid (3). Chlorpheniramine (4). Zantac Correct option : 4 Antacids are a class of medicines that neutralize acid in the stomach. They contain ingredients such as aluminum, calcium, or magnesium which act as bases (alkalis) to counteract the stomach acid and lower pH. They work quickly and are used to relieve symptoms of acid reflux, heartburn or indigestion (dyspepsia). Antacids are available as liquids or tablets. Some mixtures contain sodium and may not be suitable for people on a sodium restricted diet. Some products combine antacids with alginates and are used in the treatment of Gastroesophageal Reflux Disease (GERD). Alginates are gum-like substances that float on top of the stomach contents, forming a raft that acts like a barrier, preventing gastric acid from escaping back up the esophagus. The most widely used antacid is Zantac.

******ebook converter DEMO Watermarks*******

Q63. In which district of HP is Saryanj Sarma watershed project? Choose an option: (1). Una (2). Sirmaur (3). Bilaspur (4). Solan Correct option : 4

Saryanj is a Village in Kunihar Tehsil, Solan, Himachal Pradesh. Saryanj Sarma watershed project is in district Solan. It is the one of the six watershed development projects on full grant basis funded through NABARD’s Watershed Development Fund. These projects have resulted in raising the water availability table. They have also proved to be an important mechanism for environment protection besides increasing productivity and income of the farmers and conserving the diminishing pastures, thus facilitating animal husbandry also.

******ebook converter DEMO Watermarks*******

Q64. A simple machine helps a person in doing: Choose an option: (1). Less Work (2). The same amount of work with lesser force (3). The same amount of work slowly (4). The same amount of work much faster Correct option : 1 Work is defined as force acting on an object in the direction of motion. A machine makes work easier to perform by accomplishing one or more of the following functions: 1. transferring a force from one place to another, 2. changing the direction of a force, 3. increasing the magnitude of a force, or 4. increasing the distance or speed of a force.

******ebook converter DEMO Watermarks*******

Q65. Which Raja of Spiti invaded Kullu and made it a tributary? Choose an option: (1). Samudra Sena (2). Chet Sena (3). Hemat Sena (4). Rajendra Sena Correct option : 4 Rajendra Sena of Spiti invaded Kullu and made it a tributary.

******ebook converter DEMO Watermarks*******

Q66. As per the IPCC 4th Assessment Report, the average sea level during the period 1961-2003 increased approximately at the rate of Choose an option: (1). 1.8 cm per year (2). 5.9mm per year (3). 4.1mm per year (4). 1.8mm per year Correct option : 4

******ebook converter DEMO Watermarks*******

Q67. Who abolished Dual Government in Bengal? Choose an option: (1). Sir John Shore (2). Lord Curzon (3). Robert Clive (4). Warren Hasting Correct option : 4

******ebook converter DEMO Watermarks*******

Q68. The temples located in the region between the Vindhyas and the Krishna river are known as: Choose an option: (1). Dravid style (2). Nagar Style (3). Vakataka Style (4). Hybrid style Correct option : 1

******ebook converter DEMO Watermarks*******

Q69. Which one of the following pairs is not correctly matched? Towns - Rivers Choose an option: (1). Ayodhya - Saryu (2). Surat - Tapti (3). Bareilly - Ganga (4). Kurnool - Tungbhadra Correct option : 3

******ebook converter DEMO Watermarks*******

Q70. According to Dr. B. R. Ambedkar, which of the following articles of the Indian Constitution was to remain a dead-letter? Choose an option: (1). Article 360 (2). Article 356 (3). Article 352 (4). Article 32 Correct option : 2

According to Dr. B. R. Ambedkar, Article 356 of the Indian Constitution was to remain a dead-letter.

******ebook converter DEMO Watermarks*******

Q71. The responsibility of law and order rests with the police commissioner in the cities of (i) Chandigarh (ii) Delhi (iii) Mumbai (iv) Kolkata Select the correct answer from the code given below Choose an option: (1). (i), (ii), (iii) (2). (i), (ii), and (iv) (3). (ii), (iii) and (iv) (4). (i), (ii), (iii) and (iv) Correct option : 1

******ebook converter DEMO Watermarks*******

Q72. Which one of the following functions does not figure in the 11th schedule of the constitution (73rd Amendment)? Choose an option: (1). Libraries (2). Cultural activities (3). Fire services (4). Family welfare Correct option : 3

******ebook converter DEMO Watermarks*******

Q73. Driving force for an eco-system is Choose an option: (1). Solar energy (2). Vegetation (3). Producers (4). Bio-Mass Correct option : 1 Solar energy is the most readily available source of energy. It is also the most important of the non-conventional sources of energy because it is non-polluting and, therefore, helps in lessening the greenhouse effect. It is the driving force for an eco-system.

******ebook converter DEMO Watermarks*******

Q74. Read the following statements regarding the recently released NTCA report on the status of tigers in India, 2014. Find out the incorrect ones: (i) The report showed a 30% increase in tiger population to 2226 from 1706 in 2010. (ii) The largest increase was recorded in Kerala, Karnataka, Goa and Tamil Nadu. (iii) The Mudumalai-Bandipur-Nagarhole-Wayarad Complex hold the world’s largest tiger population currently estimated at over 570 tigers. The correct answer is: Choose an option: (1). (i) and (ii) (2). (ii) and (iii) (3). (iii) only (4). None of the above Correct option : 4

******ebook converter DEMO Watermarks*******

Q75. At present, the primary energy source in the world is Choose an option: (1). Hydro power (2). Fossil fuels (3). Nuclear power (4). Solar power Correct option : 2

******ebook converter DEMO Watermarks*******

Q76. Which of the following has the maximum bio-diversity? Choose an option: (1). Mangroves (2). Coral reefs (3). Temperature regions (4). Tropical regions Correct option : 4

******ebook converter DEMO Watermarks*******

Q77. World Ozone day is celebrated on Choose an option: (1). June 19th (2). August 19th (3). September 16th (4). November 19th Correct option : 3 September 16 was designated by the United Nations General Assembly as the International Day for the Preservation of the Ozone Layer. This designation had been made on December 19, 2000, in commemoration of the date, in 1987, on which nations signed the Montreal Protocol on Substances that Deplete the Ozone Layer.

******ebook converter DEMO Watermarks*******

Q78. In India, the forest cover percentage of the total geographical area of the country is appox. Choose an option: (1). 18% (2). 29% (3). 21% (4). 31% Correct option : 3 According to the India State of Forest Report (ISFR) 2015 stats, the total forest and tree cover is 79.42 million hectare, which is 24.16 percent of the total geographical area. in 2010, forest cover was 22% of total area of country. India is one of the ten most forest-rich countries of the world along with Russia, Brazil, Canada, United States of America, China, Democratic Republic of the Congo, Australia, Indonesia and Sudan. http://pib.nic.in/newsite/PrintRelease.aspx?relid=132571 https://en.wikipedia.org/wiki/Forestry_in_India

******ebook converter DEMO Watermarks*******

Q79. A fuse is used in main electric supply as safety devices. Which one of the following statements about the fuse is correct? Choose an option: (1). It is connected in parallel with the main switch (2). It is made mainly from silver alloys (3). It must have a low melting point (4). It must have a very high resistance Correct option : 3

******ebook converter DEMO Watermarks*******

Q80. What is the script of the earliest epigraphical rock inscriptions at Pathyar and Kanhiyara in Kangra? Choose an option: (1). Brahmi and Tankri (2). Kharoshthi and Tankri (3). Brahmi and Kharoshthi (4). Kharoshthi and Persian Correct option : 3

******ebook converter DEMO Watermarks*******

Q81. How many biosphere reserves are there in India? Choose an option: (1). 12 (2). 18 (3). 22 (4). 30 Correct option : 2 The Indian government has established 18 Biosphere Reserves in India. These Biospheres protect larger areas of natural habitat essential for wild life including flora and fauna. Protection is granted not only to the flora and fauna of the protected region, but also to the human communities who inhabit these regions, and their ways of life. Himachal has 1 out of these 18 Biosphere covering 7770 Sq KM area. It is located in Pin Valley National Park and surroundings, Chandratal and Sarchu & Kibber Wildlife Sancturary.

******ebook converter DEMO Watermarks*******

Q82. Earth Summit 2012 for sustainable development was held in Choose an option: (1). New Delhi, India (2). Rio-Di-Janeiro, Brazil (3). Johannesburg, South Africa (4). New York, USA Correct option : 2 In 2012, the United Nations Conference on Sustainable Development was held in Rio, and is also commonly called Rio+20 or Rio Earth Summit 2012. It resulted in a focused political outcome document which contains clear and practical measures for implementing sustainable development.

******ebook converter DEMO Watermarks*******

Q83. The sensation of fatigue in the muscle after prolonged strenuous physical work is caused by: Choose an option: (1). A decrease in the supply of oxygen (2). Minor wear and tear of muscles fibres (3). The depletion of glucose (4). The accumulation of lactic acid Correct option : 4 Muscle fatigue is inability of muscles to generate force. It may be result of vigorous exercise. It may be due to multiple reasons like - shortage of fuel within muscle fiber, excess of potassium, low potassium or sodium, or due to Lactic Acid build up in muscles.

******ebook converter DEMO Watermarks*******

Q84. The UN Convention on Biodiversity was ratified by the Indian government in the year Choose an option: (1). 1992 (2). 1993 (3). 1994 (4). 1997 Correct option : 3 India has always been playing a proactive role in the development and implementation of several global, international, regional and national policies and programmes related to environment, biodiversity, trade and other relevant areas. Convention on Biological Diversity was adopted by the Intergovernmental Negotiating Committee for a Convention on Biological Diversity, during its Fifth session, held at Nairobi from 11 to 22 May 1992. Consequent to the ratification of UN Convention on Biological Diversity(CBD) by India on 18th February, 1994 and in pursuance of the COP decisions of CBD that followed, the Govt of India has taken several steps in implementing the CBD provisions.

******ebook converter DEMO Watermarks*******

Q85. National Sample Survey Office was established in the year : Choose an option: (1). 1950 (2). 1951 (3). 1947 (4). 1948 Correct option : 1 The National Sample Survey Office(NSSO) in India is a unique setup to carry out surveys on socio-economic, demographic, agricultural and industrial subjects for collecting data from house holds and from enterprises located in villages and in the towns. It is a focal agency of the Govt. of India for collection of statistical data in the areas which are vital for developmental planning. The National Sample Survey Directorate was first setup in the country in the ministry of finance in 1950.

******ebook converter DEMO Watermarks*******

Q86. Match the List I with List II List 1 A) Dadabhai Naoroji B) Dayanand Sarswati C) Bal Gangadhar Tilak D) Bankim Chandra Chatterjee List II 1) Satyarth Prakash 2) Anand Math 3) Poverty and Un-British Rule of India 4) Gita Rahasya Select the correct answer from the codes given below: Codes: Choose an option: (1). (a-2), (b-3), (c-4), (d-1) (2). (a-3), (b-1), (c-4), (d-2) (3). (a-4), (b-1), (c-3), (d-2) (4). (a-1), (b-3), (c-4), (d-2) Correct option : 2

******ebook converter DEMO Watermarks*******

Q87. Recently, the government of India has decided to revise the base year for estimating the GDP from Choose an option: (1). 2001-02 to 2010-11 (2). 2004-05 to 2013-14 (3). 2004-05 to 2011-12 (4). 2001-02 to 2011-12 Correct option : 3

******ebook converter DEMO Watermarks*******

Q88. Modi Government re-launched the Kisan Vikas Patra (KVP) investment scheme to top house hold saving for funding infrastructure development in the country. Consider the following statements about this and pick the correct statement: (i) The saving instrument will be available in the denomination of Rs. 1000, Rs 5000, Rs 10,000 and Rs. 50,000. (ii) There will be no upper ceiling on investment. (iii) No tax benefits would be available on this investment. Choose an option: (1). (i) (2). (ii) (3). (i) and (iii) only (4). All of the above Correct option : 4

******ebook converter DEMO Watermarks*******

Q89. Shanta Kumar Committee has submitted its report on restructuring of Food Corporation of India (FCI) to Mr. Modi. Read the following recommendations and find the incorrect ones: (i) FCI Procurement should focus on the Western belt, where farmers get the minimum support price. (ii) FCI should hand over the procurement of Wheat and Rice to four states which include Punjab, Haryana, Andhra Pradesh and Chhattisgarh. (iii) Proposed uniform tax of minimum 3% and maximum 4% on Wheat and Rice and the same to be included in the minimum support price. The answer is: Choose an option: (1). (i) and (ii) (2). (i) and (iii) (3). (ii) and (iii) (4). None of the above Correct option : 1

******ebook converter DEMO Watermarks*******

Q90. As per the population census 2011, what is the percentage of urban population in Himachal and India respectively Choose an option: (1). 15.05 and 35.60 (2). 10.03 and 31.16 (3). 10.03 and 35.60 (4). 12.65 and 31.16 Correct option : 2

******ebook converter DEMO Watermarks*******

Q91. Which one of the following does a TV remote control unit to use to operate a TV set? Choose an option: (1). Light Waves (2). Sound Waves (3). Micro Waves (4). Radio Waves Correct option : 4 TV Remote Control Unit uses Infrared (IR) Light range of Radio Waves to operate a TV Set. The signal between a remote control handset and the device it controls consists of pulses of infrared light that represents a specific binary code like Powen On/Off, which is invisible to the human eye, but can be seen through a digital camera, video camera or a phone camera. Same concept is used in all kind of Toy’s with a remote control. Radio waves are electromagnetic waves. Tip Switch on mobile phone camera. Point TV remote towards the camera. Try to see the TV Remote Infra Red LED through Camera while tapping on remote buttons. See what happens! Try it now.

******ebook converter DEMO Watermarks*******

Q92. Which Raja of Mandi gave the Shivratri Fair the form of a cultural festival? Choose an option: (1). Suraj Sen (2). Bahu Sen (3). Bir Sen (4). Ban Sen Correct option : 1

******ebook converter DEMO Watermarks*******

Q93. Which of the following peaks are in Himachal Pradesh? (i) Abi gamin (ii) Reo-Purjil (iii) Rangrik Range (iv) Gya Choose the correct answer: Choose an option: (1). Only (iv) (2). Only (i) (3). None of the above (4). All of the above Correct option : 4

******ebook converter DEMO Watermarks*******

Q94. Find the mismatch in the following pairs: Choose an option: (1). Tieun Dhar - Chamba (2). Jakh Dhar - Hamirpur (3). Haripur Dhar - Sirmaur (4). Sanjhi Dhar – Shimla Correct option : 1

******ebook converter DEMO Watermarks*******

Q95. When did the number of members in the HP Vidhan Sabha increase from 36 to 41? Choose an option: (1). 1971 (2). 1967 (3). 1963 (4). 1954 Correct option : 4

******ebook converter DEMO Watermarks*******

Q96. Almost all major oil produces including Saudi Arabian have become dependent on high prices, with break-even level usually over $90 per barrel to meet their fiscal obligations. Consider the following can be termed as explanation for the present decline in oil prices? (i) The world markets are flush with supplies for which there are not enough consumers. (ii) The global economic slowdown is the principal culprit: with China’s national growth projected at around 7% per year, increase in oil demand is negligible. (iii) Europe is also not expected to contribute to increased demand due to its sluggish growth. Which of the statements given above is/are correct? Choose an option: (1). (i) (2). (ii) (3). Both (i) and (ii) (4). All of the above Correct option : 4

******ebook converter DEMO Watermarks*******

Q97. The world’s largest ground based telescopic observatory is located in: Choose an option: (1). Colombia (2). India (3). Nepal (4). Switzerland Correct option : 2

******ebook converter DEMO Watermarks*******

Q98. Apart from Carbon-dioxide, which of the following gases also have a role in climate change? Choose an option: (1). Methane (2). Nitrogen Dioxide (3). Sulphur Dioxide (4). Helium Correct option : 1 Greenhouse effect. A gas that contributes to the greenhouse effect by absorbing infrared radiation, thereby warming up the planet’s surface. The primary greenhouse gases in Earth’s atmosphere are water vapor, carbon dioxide, methane, nitrous oxide, ozone and chlorofluorocarbons.

******ebook converter DEMO Watermarks*******

Q99. At present, which country is the maximum emitter of Carbon-dioxide? Choose an option: (1). USA (2). China (3). Russia (4). France Correct option : 2 China has maximum share (30% carbon emission) of Carbon Dioxide in world. China (30%), United States (15%), European Union (10%), India (7%), Russia (5%) and Japan (4%) roughly emit 70% of total Carbon Emission in world. Rest of the countries together make less than 30% carbon emission.

******ebook converter DEMO Watermarks*******

Q100. What is the tenure of Chief Election Commissioner of India? Choose an option: (1). Five Yaers (2). During the pleasure of the President (3). Six years or till the age 65 years whichever is earlier (4). Five Years or till the age of 65 years whichever is earlier Correct option : 3 The President of India appoints the Chief Election Commissioner. Conventionally, senior-most Election Commissioner is appointed as CEC. He has tenure of six years, or up to the age of 65 years, whichever is earlier.

******ebook converter DEMO Watermarks*******

2013 HPAS Solved Paper This PDF Booklet contains 2013 HPAS Solved Paper. Explanation is provided by Shunya Foundation Team. 2013 Solved Paper for HPAS General Studies

******ebook converter DEMO Watermarks*******

Q1. What is the name of Pakistani Bus that plies between Lahore and Delhi? Choose an option: (1). Sada-e-Sarhad (2). Samjhauta (3). Karvan-e-Aman (4). Dosti Correct option : 1

Delhi Lahore Bus The Delhi–Lahore Bus, officially known as Sada-e-Sarhad, is a passenger bus service connecting the Indian capital of Delhi with the city of Lahore, Pakistan via the border transit post at Wagah. It was inaugurated on 19th Feb 1999. Key points 1. It was officially launched on 16 March 1999. 2. In its inaugural run on 19 February 1999, the bus carried the then-Indian Prime Minister Atal Bihari Vajpayee, who was to attend a summit in Lahore and was received by his Pakistani counterpart, Nawaz Sharif at Wagah. 3. the bus service was not halted even after the outbreak of the Kargil War. 4. The bus service was halted in the aftermath of the 2001 Indian Parliament attack, which led to a serious confrontation between the two neighbours. 5. The bus service was resumed on 16 July 2003 when bilateral relations had improved. 6. The duration of the entire journey is 8 hours, covering a distance of 530 km.

******ebook converter DEMO Watermarks*******

Q2. Standard 18-carat gold sold in the market contains Choose an option: (1). 82 parts gold and 18 parts other metals (2). 18 parts gold and 82 parts other metals (3). 18 parts gold and 6 parts other metals (4). 9 parts gold and 15 parts other metals Correct option : 3 Carat is a fractional measure of purity for gold alloys, in parts fine per 24 parts whole. K = 24 (Mg/Mm) Where K = Karat rating Mg = mass of pure gold in alloy Mm = total mass of the material. For example, 24K = 99.99% purity (24/24) 22K = 91.66% purity (22/24) 18K = 75% purity (18/24) So 18 Carat gold means 18 parts pure gold out of total 24 parts (6 parts other alloy).

******ebook converter DEMO Watermarks*******

Q3. Which of the following is represented by the Harrapan Civilization? Choose an option: (1). Second urbanization (2). Third urbanization (3). First urbanization (4). Fourth urbanization Correct option : 3 Harrapan Civilization represents first urbanization. About Harrapa 1. Mohenjo-daro and Harappa belong to 2600 BCE. 2. Both these civilizations flourished along the banks of Indus River in Punjab and Sindh. 3. These two cities were first to have their own Sewerage System. 4. The city is believed to have had as many as 23,500 residents and occupied about 370 acres.

******ebook converter DEMO Watermarks*******

Q4. In which of the following year the Asiatic Society of Bengal was established? Choose an option: (1). 1757 (2). 1784 (3). 1835 (4). 1857 Correct option : 2 In 1783, William Jones arrived in Calcutta as a junior judge at the Supreme Court that the Company had set up. In addition to being an expert in law, Jones was a linguist English. Jones set up the Asiatic Society of Bengal on 15 January 1784,

******ebook converter DEMO Watermarks*******

Q5. Who is Shinzo Abe? Choose an option: (1). Prime Minister of Israel (2). Dissident artist of China (3). Chess player of Korea (4). Prime Minister of Japan Correct option : 4 Shinzō Abe is the current Prime Minister of Japan and the President of the Liberal Democratic Party (LDP). He is one of the longest serving PMs in Japanese history.

******ebook converter DEMO Watermarks*******

Q6. The major component of honey is Choose an option: (1). Glucose (31%) (2). Sucrose (1%) (3). Maltose (4). Fructose (38%) Correct option : 4 Honey is a source of Carbohydrates, mainly containing 1. 80% natural sugar (Fructose and Glucose) 2. 18% water 3. 2% minerals, vitamins, pollen and protein. Key Highlights of Honey 1. Most microorganisms do not grow in honey, so sealed honey does not spoil, even after thousands of years. 2. Honey gets its sweetness from the monosaccharides fructose and glucose. 3. In 2013, 1.7 million tonnes of honey were produced worldwide, with China accounting for 28% of the world total (table). The next four largest producers – Turkey, Argentina, Ukraine, and Russia – accounted collectively for less than 20% of the world total. 4. Honey contains about Fructose: 28% to 41% and Glucose: 22% to 35% 5. Honey has a density around 1.36 kg/l (36% denser than water)

******ebook converter DEMO Watermarks*******

Q7. With which district of Himachal Pradesh is Kayang folk dance mainly associated? Choose an option: (1). Una (2). Solan (3). Sirmaur (4). Kinnaur Correct option : 4

The Kayang Mala Dance is very popular in Himachal Pradesh. It is mainly associated with Kinnaur district. In this, the dancers weave their arms together, to form a sort of a criss-cross pattern, so that they appear like beads in an intricately woven garland.

******ebook converter DEMO Watermarks*******

Q8. Which lake is jointly owned by China and India? Choose an option: (1). Tsomoriri (2). Tsokar (3). Pongong Tso (4). Thanka Correct option : 3 Pangong Tso is situated at the Line of Actual Control between India and China. Its elevation from sea level is 4350 meters. Total length of lake is 134 km and maximum width is 5 km covering total area of 604 square km. Approximately 60% of the length of the lake is controlled by China. This lake is completely frozen in winters. The lake is in the process of being identified under the Ramsar Convention as a wetland of international importance.

******ebook converter DEMO Watermarks*******

Q9. "There is enough for everybody’s need but not enough for everybody’s greed." Who said it? Choose an option: (1). Nelson Mandela (2). Jawaharlal Nehru (3). M.K. Gandhi (4). Rabindranath Tagore Correct option : 3 India’s great moral leader Mohandas Gandhi famously said that there is enough on Earth for everybody’s need, but not enough for everybody’s greed. Gandhi was making the point that there is enough food and water, shelter, etc. (basic needs) for the needs of every person. There is not, however, enough natural resources to fill the want of people’s greed. It means that too many human beings hoard food and money and don’t share it with others. There is ample supply of everything for everybody but greed and selfishness makes people unwilling to share.

******ebook converter DEMO Watermarks*******

Q10. Which one of the following is not trade block? Choose an option: (1). SAFTA (2). EFTA (3). OPEC (4). UPEC Correct option : 4 A trade bloc is a type of intergovernmental agreement, often part of a regional intergovernmental organization, where regional barriers to trade, (tariffs and non-tariff barriers) are reduced or eliminated among the participating states. Organization of the Petroleum Exporting Countries (OPEC) - It is an intergovernmental organization of 13 nations as of 2017, founded in 1960 in Baghdad by the first five members (Iran, Iraq, Kuwait, Saudi Arabia, Venezuela), and headquartered since 1965 in Vienna. The South Asian Free Trade Area (SAFTA) - It is an agreement reached on 6 January 2004 at the 12th SAARC summit in Islamabad, Pakistan. It created a free trade area of 1.6 billion people in Afghanistan, Bangladesh, Bhutan, India, Maldives, Nepal, Pakistan and Sri Lanka. The European Free Trade Association (EFTA) It is a regional trade organisation and Free trade area consisting of four European states: Iceland, Liechtenstein, Norway, and Switzerland. The organisation operates in parallel with the European Union (EU), and all four member states participate in the EU’s single market.

******ebook converter DEMO Watermarks*******

Q11. Climatology is the science of: Choose an option: (1). Edaphic factors (2). Topographic (3). Climate factors (4). Biotic factors Correct option : 3 Climatology is the study of atmospheric changes (both long and short-term) that define average climates and their change over time, due to both natural and anthropogenic climate variability. In fact, climatology is the science of climate factors.

******ebook converter DEMO Watermarks*******

Q12. Zulus tribe is found in? Choose an option: (1). South America (2). Africa (3). Australia (4). Asia Correct option : 2

Fig. Zulu Tribal People The Zulu are a Bantu ethnic group of Southern Africa and the largest ethnic group in South Africa. They are considered traditional warriors.

******ebook converter DEMO Watermarks*******

Q13. Which one of the following is the largest producer of coffee in the world? Choose an option: (1). Sri Lanka (2). Brazil (3). America (4). Kenya Correct option : 2

Fig. Coffee Beans Coffee grows in deep, porous and water retentive soils with high humus content. It grows best at an altitude of 300 m to 800 m. Volcanic soil is very suitable for coffee. Frost is its enemy, hence its cultivation is confined to the tropics. The chief coffee-producing countries of the world are - Brazil, Colombia, Central America, Mexico and Indonesia. Brazil, being the largest producer of coffee, alone produces nearly half the world’s supply of coffee.

******ebook converter DEMO Watermarks*******

Q14. Which is the tallest dam of India? Choose an option: (1). Hirakund (2). Tehri (3). Bhakhra- Nangal (4). Sardar Sarovar Correct option : 2 Tehri Dam is located on the Bhagirathi River in Uttarakhand. Tehri Dam is the highest/tallest dam in India. With a height of 261 meters, it is the eighth tallest dam in the world.

******ebook converter DEMO Watermarks*******

Q15. Which of the following rivers is called ‘Nadittam’ in Rigveda? Choose an option: (1). Jhelum (2). Ravi (3). Beas (4). Saraswati Correct option : 4 Saraswati river is called ‘Nadittam’ in Rigveda.

******ebook converter DEMO Watermarks*******

Q16. The officer who enjoyed authority over the large pasture ground was known as during the Rigvedic period? Choose an option: (1). Gramini (2). Vrajapati (3). Kulapas (4). Senani Correct option : 2 During Rig-Vedic era, the basic unit of power lied within a patriarchal family (Kula). The Grama was not essentially a village but a group of families who kept on migrating. So, the concept of a settled village was absent in early Vedic era. The officer who enjoyed authority over the large pasture ground was known as Vrajapati. His duty was to manage pastoral land and to lead the battles.

******ebook converter DEMO Watermarks*******

Q17. Usually fuels on burning causes pollution. which one of the following fuel causes minimum environmental pollution ? Choose an option: (1). Diesel (2). Coal (3). Hydrogen (4). Kerosene Correct option : 3 Hydrogen Fuel Cell - One of the most important benefits of a hydrogen economy is that fuel cells are nonpolluting. No carbon emissions are produced when electricity is generated in a fuel cell. A hydrogen fuel cell produces two byproducts - heat and water.

******ebook converter DEMO Watermarks*******

Q18. What is the average fat content of "Buffalo milk"? Choose an option: (1). 7.2% (2). 4.2% (3). 9.0% (4). 10.0% Correct option : 1 The average fat content in buffalo milk is about 7 to 8% while protein content in buffalo milk ranges from 4.2 to 4.5%.

******ebook converter DEMO Watermarks*******

Q19. On the bank of which river is Nadaun town of Hamirpur District? Choose an option: (1). Satluj (2). Ravi (3). Beas (4). Uhl Correct option : 3 River Beas passes through Nadaun situated in Hamirpur district of Himachal Pradesh. Satluj touches Kinnaur, Shimla and Bilaspur district of Himachal Pradesh. Ravi flow through district Chamba of Himachal Pradesh.

******ebook converter DEMO Watermarks*******

Q20. Which of the following is the author of Tamil Ramayana? Choose an option: (1). Pampa (2). Ponna (3). Kamban (4). Ranna Correct option : 3 Kambar (Kamban) was a medieval Tamil poet and the author of the Ramavataram, popularly known as Kambaramayanam, the Tamil version of Ramayana. Kambar also authored other literary works in Tamil, such as 'Thirukkai Vazakkam',Erezhupathu, Silaiezhupathu, Kangai Puranam, sadagopar anthathi, and Sarasvati Anthati.

******ebook converter DEMO Watermarks*******

Q21. Jhum is a term used for: Choose an option: (1). Terrace farming (2). Tribal farming (3). Shifting farming (4). Farming on a slope Correct option : 3 Shifting cultivation is an agricultural system in which plots of land are cultivated temporarily, then abandoned and allowed to revert to their natural vegetation while the cultivator moves on to another plot. Shifting cultivation is called as :-Jhum or Jhumming in north-east India, Milpa in Mexico, Roca in Brazil and Ladang in Malaysia.

******ebook converter DEMO Watermarks*******

Q22. When ants bite, they inject: Choose an option: (1). Glacial acetic acid (2). Methanol (3). Formic acid (4). Stearic acid Correct option : 3 Ant bites and bee stings are acidic in nature. Generally, ant bites cause irritation, swelling, pain and itching. Most ants inject a venom, the main constituent of which is formic acid.

Fig. Ants

******ebook converter DEMO Watermarks*******

Q23. The 'Stones' formed in human kidney consist mostly of: Choose an option: (1). Calcium oxalate (2). Sodium acetate (3). Magnesium sulphate (4). Calcium alone Correct option : 1

Kidney stones are solid masses that form in the kidney when there are high levels of calcium, oxalate, cystine, or phosphate and too little liquid. Calcium oxalate stones are the most common type of kidney stone.

******ebook converter DEMO Watermarks*******

Q24. Which tehsil of Mahasu district was carved out to form Kinnaur district? Choose an option: (1). Rohru (2). Chini (3). Kotkhai (4). Rampur Correct option : 2 Kinnaur district a erstwhile known as Chini tehsil of the former Mahasu district, came into being as an independent district w.e.f. 1st May, 1960. Prior to merger of State on the eve of independence, Kinnaur valley was a part of erstwhile Bushahr State which had its headquarters at Rampur.

Fig. Kinnaur District

******ebook converter DEMO Watermarks*******

Q25. For which work did Mridula Garg get the 2013 Sahitya Academy Award? Choose an option: (1). Miljulaman (2). Uske Hisse Ki Dhoop (3). Chitkobra (4). Kathgulab Correct option : 1 Miljul Mann (novel) was awarded the Sahitya Akademi Award in 2013.

******ebook converter DEMO Watermarks*******

Q26. According to Transparency Intentional which are the most honest countries in the world in 2012-13? Choose an option: (1). France and Belgium (2). Holland and Australia (3). New Zealand and Denmark (4). Sweden and Germany Correct option : 3 New Zealand, Denmark and Finland are most honest countries in 2012-13

******ebook converter DEMO Watermarks*******

Q27. Who was crowned as Miss World in 2013 beauty contest? Choose an option: (1). Marialin of France (2). Navaneet Kaur of India (3). Oaklin of Ghana (4). Megan Young of Philippines Correct option : 4 Megan Young of Philippines was announced the winner of Miss World 2013.

******ebook converter DEMO Watermarks*******

Q28. The UN Human Rights Council (HRC) was set up in: Choose an option: (1). 2006 (2). 2008 (3). 2010 (4). 2012 Correct option : 1 The Human Rights Council is an inter-governmental body within the United Nations system made up of 47 States responsible for the promotion and protection of all human rights around the globe. The Council was created by the United Nations General Assembly on 15 March 2006.

******ebook converter DEMO Watermarks*******

Q29. Which day is observed as Anti-Leprosy Day? Choose an option: (1). October 02 (2). November 14 (3). January 30 (4). October 31 Correct option : 3 ANTI LEPROSY DAY 2017. Anti Leprosy Day (Martyrdom Day of Mahatma Gandhi) 2017 will be celebrated all over India at Monday, on 30th of January.

******ebook converter DEMO Watermarks*******

Q30. Arteries supplying blood to the: Choose an option: (1). Carotid arteries (2). Hepatic arteries (3). Coronary arteries (4). Pulmonary arteries Correct option : 3

Heart and Arteries Arteries supplies Oxygen Rich blood to Coronary Arteries. Coronary arteries supply blood to the heart muscle. Like all other tissues, Heart muscles need Oxygen Rich blood for its functioning. Since coronary arteries deliver blood to the heart muscle, any coronary artery disorder or disease can have serious implications by reducing the flow of oxygen and nutrients to the heart muscle. This can lead to a heart attack and possibly death.

******ebook converter DEMO Watermarks*******

Q31. Cryogenic engines find applications in: Choose an option: (1). Submarine propulsion (2). Frost-free refrigerators (3). Rocket technology (4). Researches in super-conductivity Correct option : 3 A cryogenic engine is a rocket engine that uses a cryogenic fuel or oxidizer, that is, its fuel or oxidizer (like liquid hydrogen and liquid oxygen) are gases liquefied and stored at very low temperatures. The word originates from the Greek words 'kryos' meaning "frost" and 'genic' meaning "to produce." Under such a definition it could be used to include all temperatures below the freezing point of water (0 C).

******ebook converter DEMO Watermarks*******

Q32. Which of the following is not in the vicinity of Kullu? Choose an option: (1). Sultanpur Palace (2). Bijli Mahadev (3). Dev Tibba (4). Kalatope Correct option : 4 Kalatop Khajjiar Sanctuary is located in Chamba District of Himachal Pradesh. Rest all are located in vicinity of Kullu.

******ebook converter DEMO Watermarks*******

Q33. In which town is Panchvaktra temple? Choose an option: (1). Chamba (2). Mandi (3). Hamirpur (4). Una Correct option : 2 Panchvaktra temple is located in Bhiuli, Mandi, Himachal Pradesh Pin - 175001. Panchvaktra temple is dedicated ti Lord Shiva.

******ebook converter DEMO Watermarks*******

Q34. Ultimate source of energy for living being is: Choose an option: (1). Carbohydrates (2). Fats (3). Sunlight (4). ATB Correct option : 3

Fig. The Sun All changes in the weather are caused by the sun. The sun is a huge sphere of hot gases at a very high temperature. The distance of the sun from us is very large. Even then the energy sent out by the sun is so huge that it is the source of all heat and light on the earth. So, the sun is the primary source of energy that causes changes in the weather. Energy absorbed and reflected by the earth’s surface, oceans and the atmosphere play important roles in determining the weather at any place. If you live near the sea, you would have realised that the weather at your place is different from that of a place in a desert, or near a mountain.

******ebook converter DEMO Watermarks*******

Q35. Soil erosion can be prevented by: Choose an option: (1). Deforestation (2). Forestation (3). Overgrazing (4). Vegetation removal Correct option : 2 Soil erosion is a naturally occurring process that affects all landforms. In agriculture, soil erosion refers to the wearing away of a field’s topsoil by the natural physical forces of water and wind or through forces associated with farming activities such as tillage. Forestation, Planting Grass, etc can be used to limit the Soil Erosion.

******ebook converter DEMO Watermarks*******

Q36. With which game/sport is Deepika Kumari associated? Choose an option: (1). Wrestling (2). Archery (3). Badminton (4). Boxing Correct option : 2 Deepika Kumari is an Indian athlete who competes in the event of Archery, is currently ranked World No. 5, and is a former world number one. She won a gold medal in the 2010 Commonwealth games in the women’s individual recurve event.

******ebook converter DEMO Watermarks*******

Q37. When was Telegraph/Telegram service closed in India? Choose an option: (1). July, 2013 (2). September, 2012 (3). June, 2013 (4). October, 2013 Correct option : 1 Indian Telegram Service was started in 1850 on an experimental basis between Koklata and Diamond Harbour, it was opened for use by the British East India Company the following year. In 1854, the service was made available to the public. The Service is eventually replaced by SMS, emails, mobile phones, thereby reducing revenue generation. By July 2013, this service generated about Rs 75 lakh annually, compared with the cost of over Rs. 100 core to run and manage it. Finally it was shutdown in July 2013.

******ebook converter DEMO Watermarks*******

Q38. Fat present below the skin surface in our body, acts as a barrier against: Choose an option: (1). Loss of heat from our body (2). Loss of essential body fluids (3). Loss of salts from the body (4). Entry of harmful micro-organisms form the environments Correct option : 1 Fat present below the skin surface of human body acts as a barrier against the Loss of heat from our body. That’s the reason people with very less fat need a warm surrounding to avoid loss of body heat.

******ebook converter DEMO Watermarks*******

Q39. In which year publication of Hindi daily Divya Himachal begin? Choose an option: (1). 1997 (2). 2001 (3). 2003 (4). 2007 Correct option : 1 Divya Himachal is a Hindi Newspaper of Himachal Pradesh, having circulation in Himachal Pradesh, Punjab, Haryana, Uttranchal and Chandigarh (UT). The newspaper was launched on 29 December 1997 and now has three editions namely Dharamshala, Shimla and Chandigarh with the fourth Baddi edition being launched on April 1, 2017.

******ebook converter DEMO Watermarks*******

Q40. Who was the first person to vote in the First General Elections in India in 1951-52? Choose an option: (1). Shyam Saran Negi of Kalpa (2). Satya Dev Bushehri of Rohru (3). Bhagmal Sautha of Jubbal (4). Anand Chand of Bilaspur Correct option : 1 1. born in 1917 at Kalpa, Himachal Pradesh is the first person to cast the first vote in 1951-52 general election. 2. He is a retired school teacher by profession. 3. 1951-52 was nation’s first general election since the end of British Raj in 1947. 4. Negi has voted in every general election since 1951, and is believed to be India’s oldest voter as well its first. 5. In 2010, then-chief election commissioner Navin Chawla visited Negi’s village to honour him as part of the Election Commission’s diamond jubilee celebrations. 6. Negi has voted in every general election since 1951, and is believed to be India’s oldest voter as well its first.

******ebook converter DEMO Watermarks*******

Q41. Which of the following has the highest sex ratio as per 2011 census? Choose an option: (1). Mizoram (975) (2). Meghalaya (986) (3). Tamil Nadu (995) (4). Andhra Pradesh (992) Correct option : 3 As per 2011 census, Kerala has sex ratio of 1084, Puducherry has 1037 and Tamil Nadu has 996 females per 1000 males. In most sexually reproducing species, sex ratio tends to be 1:1 i.e. 1000 females per 1000 males. This tendency is explained by Fisher’s Principle. See Also. https://en.wikipedia.org/wiki/Fisher%27s_principle https://en.wikipedia.org/wiki/Sex_ratio

******ebook converter DEMO Watermarks*******

Q42. Which Indian State has the lowest death rate according to census 2011? Choose an option: (1). Orissa (2). Kerala (3). West Bengal (4). Himachal Pradesh Correct option : 3 According to 2011 census, West Bengal has crude death rate of 6.2 per thousand population. Odisha has highest rate of 8.5 while Delhi has 4.3. Himachal Pradesh’s crude death rate stands at 6.7 per thousand of population.

******ebook converter DEMO Watermarks*******

Q43. Which one of the following is located in Atlantic Ocean? Choose an option: (1). Okinawa (2). Bermuda (3). Timor (4). Male Correct option : 2 Bermuda is a British Overseas Territory in the North Atlantic Ocean. The first person known to have reached Bermuda was the Spanish sea captain Juan de Bermúdez in 1503, after whom the islands are named.

******ebook converter DEMO Watermarks*******

Q44. Diffusion of light in the atmosphere takes place due to: Choose an option: (1). Carbon dioxide (2). Dust particles (3). Helium (4). Water vapors Correct option : 2 Diffusion of Light in atmosphere takes place due to dust particles.

******ebook converter DEMO Watermarks*******

Q45. The characteristics odour of garlic is due to: Choose an option: (1). A chloro compound (2). A sulphur compound (3). Acetic acid (4). A fluorine compound Correct option : 2 The sulfur compounds are responsible both for garlic’s pungent odor and many of its medicinal effects.

******ebook converter DEMO Watermarks*******

Q46. Which lake at Mandi is known for floating island? Choose an option: (1). Kuntbhyog (2). Parashar (3). Kalasar (4). Sukhsar Correct option : 2 Parashar Lake in district Mandi of Himachal Pradesh is known for its floating island.

******ebook converter DEMO Watermarks*******

Q47. Which of the following are the ways of acquiring citizenship of India? 1) Birth 2) Descent 3) Incorporational of Territory 4) Naturalization Choose an option: (1). 1 only (2). 1 and 2 (3). 1, 2 and 4 (4). 1, 2, 3 and 4 Correct option : 4

Naturalization Citizenship of India by naturalisation can be acquired by a foreigner (not illegal migrant) who is ordinarily resident in India for 12 years.

******ebook converter DEMO Watermarks*******

Q48. The Right to Freedom under Article 19 of the Constitution of India may be suspended by President of India? Choose an option: (1). Under Article 360 (2). Under Article 356 (3). Under Article 352 (4). At any time he so desires Correct option : 3

******ebook converter DEMO Watermarks*******

National emergency under article 352 National emergency can be declared on the basis of external aggression or armed rebellion in the whole of India or a part of its territory. Such an emergency was declared in, 1. 1962 China War 2. 1971 Pakistan War 3. 1975 declared by then Prime Minister Indira Gandhi During a national emergency, many Fundamental Rights of Indian citizens can be suspended. The six freedoms under Right to Freedom are automatically suspended. By contrast, the Right to Life and Personal Liberty cannot be suspended according to the original Constitution.

******ebook converter DEMO Watermarks*******

Q49. Which of the following languages have been declared by the Constitution to be the languages for conducting business in Parliament? Choose an option: (1). English (2). Hindi (3). English and Hindi (4). English, Hindi and mother tongue Correct option : 3 The Constitution of India designates the official language of the Government of India as Hindi written in the Devanagari script, as well as English. There is no national language as declared by the Constitution of India. Hindi and English are used for official purposes such as parliamentary proceedings, judiciary, communications between the Central Government and a State Government. States within India have the liberty and powers to specify their own official language(s) through legislation and therefore there are 22 officially recognized languages in India. Around 45% of Indian population can speak some or other dialect of Hindi Language.

******ebook converter DEMO Watermarks*******

Q50. Which is the correct measure of Infant Mortality Rate (IMR)? Choose an option: (1). No. of infant deaths during a year/ No. of live births during the year x 1000 (2). No. of live births during the year/ No. of infant deaths during a year x 1000 (3). No. of total deaths during the year/ No. of infant deaths during a year x 1000 (4). No. of infant deaths during a year/ No. of total deaths during the year x 1000 Correct option : 1

infant mortality rate The infant mortality rate (IMR) is the number of deaths of infants under one year old per 1,000 live births. This rate is often used as an indicator of the level of health in a country. The infant mortality rate of the world is 49.4 according to the United Nations . India has infant mortality rate of 40, lowest being in kerala which is 12. Sweden, Norway, Iceland and Finland has IMR lower than 3.

******ebook converter DEMO Watermarks*******

Q51. After which American scientist is ‘h-index’ named? Choose an option: (1). Jorge E. Hirsch (2). Frederick Sainger (3). Linus Pauling (4). Percy Julian Correct option : 1

h-index 1. h-index is an index to quantify a scientist’s publication productivity and the basis of several scholar indices. 2. It was invented by professor Jorge Eduardo Hirsch in 2005. 3. Jorge Eduardo Hirsch (born 1953) is an Argentine American professor of physics at the University of California, San Diego.

******ebook converter DEMO Watermarks*******

Q52. NABARD came into existence in the year: Choose an option: (1). 1980 (2). 1982 (3). 1984 (4). 1986 Correct option : 2 National Bank for Agriculture and Rural Development is an apex development bank in India, headquartered at Mumbai with branches all over India. It was founded in 1982.

******ebook converter DEMO Watermarks*******

Q53. Who is George Alexander Louis? Choose an option: (1). A French Novelist (2). An American film star (3). Third in the line as heir to the British throne (4). A German football player Correct option : 3 George Alexander Louis is the only son of Prince William, Duke of Cambridge, and Catherine, Duchess of Cambridge. He is third in line to succeed his paternal great-grandmother, Queen Elizabeth II, after his paternal grandfather, Charles, Prince of Wales, and his father.

******ebook converter DEMO Watermarks*******

Q54. Which of the following is believed to be the oldest among Shimla Hill States? Choose an option: (1). Jubbal (2). Keonthal (3). Bushahr (4). Baghat Correct option : 3 Bashahr, also spelt as 'Bushahr' and 'Bussahir', was a princely state in India during the British Raj. The territory of this former state is now part of Kinnaur and Shimla district of the Indian republic’s state of Himachal Pradesh.

******ebook converter DEMO Watermarks*******

Q55. Which one of the following is taken as the Base Year for estimating GDP at constant prices in India during 2012-13? Choose an option: (1). 1993-94 (2). 1999-2000 (3). 2004-2005 (4). 2009-2010 Correct option : 3 2014-05 was taken as the base year for estimating GDP at constant prices during 2012-13. Important Update. As of 2017, The Central Statistics Office (CSO) has already changed the base year of India’s national accounts including the GDP and the gross value addition, to 2011-12.

******ebook converter DEMO Watermarks*******

Q56. "What was the average annual growth rate of Agricultural and Allied Sectors in India during 11th five year plan? Choose an option: (1). 3 (2). 3.6 (3). 4.2 (4). 2.8 Correct option : 2 India has recorded an average annual economic growth rate of 8 per cent during the 11th Five-Year Plan (2007-12) compared to a targeted 9 per cent, as per Govt. data. Agricultural and Allied Sectors contribution is 3.7% compared to 4% in 10th five year plan.

******ebook converter DEMO Watermarks*******

Q57. Which of the statements given below is/are correct? 1. Bihar has the highest decadal (2001-2011) growth rate population, while Kerala has the lowest. 2. The best performing state in terms of growth rate during 2011-12 in Bihar. Select the correct answer from the codes given below: Choose an option: (1). 1 only (2). 2 only (3). Both 1 and 2 (4). Neither 1 nor 2 Correct option : 3 Population Growth. Bihar’s population growth rate has dipped by 3.45 per cent to 25.07 per cent in the past one decade till 2011 compared to 28.62 per cent growth rate in a decade earlier, according to the decadal census report. Kerala had 4.9% growth rate of population during the same time period.

******ebook converter DEMO Watermarks*******

Q58. The Targeted Public Distribution System (TDPS) was adopted in the year: Choose an option: (1). 1991 (2). 1995 (3). 1997 (4). 2002 Correct option : 3 Targeted Public Distribution System (TPDS) was introduced with effect from June 1997. The focus of the Targeted Public Distribution System (TPDS) is on "poor in all areas" and TPDS involves issue of 10 Kg of food grains per family per month for the population Below Poverty Line (BPL) at specially subsidized prices.

******ebook converter DEMO Watermarks*******

Q59. What were the main findings of the N.N. Vohra Committee? Choose an option: (1). Internal Security Threat (2). Electoral Malpractices (3). Politicization of Bureaucracy (4). Nexus between Politicians, Bureaucrats and Criminals Correct option : 4 Vohra Report was submitted by former Indian Union Home Secretary, N.N. Vohra, in October 1993. It studied the nexus among criminals, politicians and bureaucrats in India.

******ebook converter DEMO Watermarks*******

Q60. Who among the following increased the Mansab from 5000/- to 7000/-? Choose an option: (1). Akbar (2). Jahangir (3). Shah Jahan (4). Aurangzeb Correct option : 1 The Mansabdari system was borrowed from the system followed in Mongolia.The Mughal officers whether Hindus or Muslims were granted territorial commands in return for the military service. They had to bring in some fixed number of men-at-arms, horses and elephants to the field and were rated as per the numbers which was known as Zats. So they were called Mansabdars of 10, 20, 100, and 1000 and so on. Akbar increased the Mansab from 5000/to 7000/-.

******ebook converter DEMO Watermarks*******

Q61. Which of the following political parties was called ‘Rajadroha Ka Karkhana’ by the British? Choose an option: (1). Muslim League (2). Indian National Congress (3). Communist Party of India (4). Unionist Party Correct option : 2 Indian National Congress was called ‘Rajadroha Ka Karkhana’ by the British.

******ebook converter DEMO Watermarks*******

Q62. Who among the following established the Central Hindu School at Banaras? Choose an option: (1). Madan Mohan Malviya (2). Annie Besant (3). Raja Ram Mohan Roy (4). Swami Dayanand Correct option : 2 Central Hindu School, formerly known as Central Hindu College, is one of India’s largest schools which is situated at Kamachha in the heart of the holy city Varanasi. It provides education to all society of people and is well equipped with highly qualified faculties and laboratory. It was founded by noted freedom-fighter Annie Besant in July 1898, with Dr. Arthur Richardson.

******ebook converter DEMO Watermarks*******

Q63. Rani Gidalu was one of the freedom fighter from the north-east. She hailed from one of the following states Choose an option: (1). Nagaland (2). Manipur (3). Assam (4). Meghalaya Correct option : 2 Rani Gidalu was one of the freedom fighter from the north-east. She was born on 26 January 1915 in Nungkao (a Rongmei village) in Manipur. She had participated in the freedom struggle at a very young age of 13 after she came under the influence Heraka religious movement. In 1937, Pandit Jawaharlal Nehru had met her at Shillong Jail and promised to pursue her release. During this time, Nehru gave her title of Rani (Queen) and since then she gained local popularity as Rani Gaidinliu.

******ebook converter DEMO Watermarks*******

Q64. Which of the following states did not return Congress in power after the first provincial elections in 1937? Choose an option: (1). Punjab (2). Gujarat (3). Sindh (4). Madras Correct option : 1 Provincial elections were held in British India in the winter of 1936-37 as mandated by the Government of India Act 1935. Elections were held in eleven provinces - Madras, Central Provinces, Bihar, Orissa, United Provinces, Bombay Presidency, Assam, NWFP, Bengal, Punjab and Sindh. The final results of the elections were declared in February 1937. The Indian National Congress emerged in power in eight of the provinces - the three exceptions being Bengal, Punjab, and Sindh.

******ebook converter DEMO Watermarks*******

Q65. Match names of List I with that of List II and select the correct answer from the codes given below: Choose an option: (1). Kailashanath Temple - Kanchipuram (2). Brihadeshwar Temple – Tanjavur (3). Hoyasaleshwar - Halebid (4). Gomteshwar - Shravanabelogola Correct option : 3 Hoysaleswara temple is a temple dedicated to the Hindu god Shiva. It was built in Halebidu during the rule of King Vishnuvardhana of the Hoysala Empire in the 12th century. The construction was started around 1120 CE and completed in 1150 CE.

******ebook converter DEMO Watermarks*******

Q66. Which one of the following is incorrect? Choose an option: (1). No scheduled caste population is found in Karnataka (2). Majority of Scheduled tribe population is found in hilly areas (3). Main occupation of scheduled caste population is agriculture based activities (4). No caste is notified as scheduled tribe in Delhi Correct option : 1 The total population of Karnataka, as per 2001 Census is 52,850,562. Of this, 8,563,930 are Scheduled Castes (SCs).

******ebook converter DEMO Watermarks*******

Q67. Which is not temperate grassland? Choose an option: (1). Prairie (2). Steppes (3). Compos (4). Downs Correct option : 3

******ebook converter DEMO Watermarks*******

Q68. In which of the following rivers ‘Swargadwari’ is located at? Choose an option: (1). Indus (2). Ganga (3). Godavari (4). Brahmaputra Correct option : 2 The camp 'Swargadwari' in which Mohammad-bin-Tughlaq spent two and a half years was located on the banks of Ganga river.

******ebook converter DEMO Watermarks*******

Q69. Which one of the following is incorrect? Choose an option: (1). India’s north-south extent is 3124 km (2). India’s east-west extent is 2933 km (3). India’s land frontier is 15,200 km (4). India’s total geographical area is 4.2% of total land area of the world Correct option : 4 The total area of Indian land mass is around 3.28 million Km². This implies that the total area of India accounts for about 2.4 per cent of the total geographical area of the word.

******ebook converter DEMO Watermarks*******

Q70. Which is the correct descending order in terms of length (km) of river? Choose an option: (1). Ganga, Godavari, Krishna, Narmada (2). Ganga, Krishna, Godavari, Narmada (3). Ganga, Godavari, Narmada, Krishna (4). Ganga, Narmada, Godavari, Krishna Correct option : 1 Ganga (2525 km), Godavari (1465 km), Krishna (1400 km), Narmada (1312 km)

******ebook converter DEMO Watermarks*******

Q71. Which of the following pairs is not correctly matched? Choose an option: (1). Pittsburg of India – Jamshedpur (2). Boston of India – Ahmadabad (3). Silicon Valley of India – Hyderabad (4). Deccan Queen of India – Pune Correct option : 3 The Silicon Valley of India is a nickname of the Indian city of Bangalore. It was named so because Bangalore is on the Mysore Plateau, the area is also sometimes referred to as "Silicon Plateau".

******ebook converter DEMO Watermarks*******

Q72. What is India’s Global Position out of 187 countries as per UNDP’s Human Development Report 2011? Choose an option: (1). 119 (2). 134 (3). 143 (4). 137 Correct option : 2 As per UNDP’s Human Development Report 2011, India’s Global Position is 134 out of 187 countries.

******ebook converter DEMO Watermarks*******

Q73. Who is the Chairman of Commission for Agricultural Costs and Prices (CACP) at present? Choose an option: (1). Abhijit Sen (2). Arjun Sengupta (3). Ashok Gulati (4). G.K. Chaddha Correct option : 3 Padma Shri Ashok Gulati served as chairman of the Commission for Agricultural Costs and Prices (CACP) from 2011 to 2014. Prof. Vijaya Paul Sharma is currently working as the new Chairman.

******ebook converter DEMO Watermarks*******

Q74. An air bubble in water will act like a: Choose an option: (1). Convex mirror (2). Convex lens (3). Concave mirror (4). Concave lens Correct option : 4 The refractive index of air is less than that of water as water is a thicker medium. As per the law of physics, ray of light diverges away from the normal as it enters a thicker medium(water) making it a diverging lens i.e. Concave Lens. In the case of an air bubble in water, the refractive index of water is more than that of air, hence it acts as Concave lens.

******ebook converter DEMO Watermarks*******

Q75. Which one of the following is not an essential micronutrient for plants? Choose an option: (1). Boron (2). Zinc (3). Sodium (4). Copper Correct option : 3 Micronutrients are essential for plant growth and play an important role in balanced crop nutrition. They include boron (B), copper (Cu), iron (Fe), manganese (Mn), molybdenum (Mo), zinc (Zn), nickel (Ni) and chloride (Cl). A lack of any one of the micronutrients in the soil can limit growth, even when all other nutrients are present in adequate amounts.

******ebook converter DEMO Watermarks*******

Q76. Identify the correct chronological order in which the following Europeans travelled in Punjab and Shimla Hill states? Select the correct answer from the code given below: 1. Vigne 2. Moorcraft 3. Forster 4. Thomas Coryat Codes: Choose an option: (1). 4, 3, 2,1 (2). 1, 3, 2, 4 (3). 2, 1, 3, 4 (4). 4, 2, 1, 3 Correct option : 4 Godfrey Thomas Vigne was an English amateur cricketer and traveller. In 1831 Vigne left England for Persia, and then travelled to India. William Moorcroft, a veterinary surgeon, was the first European to make botanical collections in western Tibet and Kashmir. He travelled to India in 1808. Edward Morgan Forster known as E. M. Forster, was an English novelist, short story writer, essayist and librettist. In 1911 he came to India with G. Lowes Dickinson. Thomas Coryat was an English traveller. He travelled India in 1612-1617 under the ruler Jahangir.

******ebook converter DEMO Watermarks*******

Q77. Who negotiated the arrangement between the states of Kashmir and Chamba by which the later became independent of the former? Choose an option: (1). Col. Lawrence (2). George Russel (3). David Ochterlony (4). Captain G. Birch Correct option : 1 Jawahar Singh and Moti Singh quarrelled in 1852 and Poonch was awarded to Moti Singh as Jagir. Jawahar Singh personally went to Colonel Lawrence and asked that he should be made independent. As per the treaty, Raja of Chamba became independent.

******ebook converter DEMO Watermarks*******

Q78. Raja of which state was treated by the wife of Dara Shikoh as her son? Choose an option: (1). Kangra (2). Sirmaur (3). Nurpur (4). Guler Correct option : 3

******ebook converter DEMO Watermarks*******

Q79. When did the consultative body of ruler of princely states known as Narendra Mandal came into existence? Choose an option: (1). 1911 (2). 1921 (3). 1931 (4). 1941 Correct option : 2 The Chamber of Princes (Narender Mandal or Narendra Mandal) was an institution established in 1920 by a royal proclamation of King-Emperor George V to provide a forum in which the rulers of the princely states of India could voice their needs and aspirations to the colonial government of British India. It survived until the end of the British Raj in 1947.

******ebook converter DEMO Watermarks*******

Q80. Who led the famous Mandi conspiracy of 1914-15? Choose an option: (1). Swami Krishna Nand (2). Pandit Gauri Prasad (3). Sobha Singh (4). Mian Jawahar Singh Correct option : 4 The famous Mandi conspiracy took place in 1914-15 A.D. under the influence of the Ghadhar Party, some member of which had returned from America and spread out in Mandi and Suket to win adherents to their cause. Extracts from Ghadhar - ki - Gunj were read by them to influence the people. Main jawahar Singh and the Rani Khairgarhi of Mandi came under their influence. The Rani helped the movement financially.

******ebook converter DEMO Watermarks*******

Q81. According to Annual Status Education Report (ASER) 2012-13 what percentage of class V students of Himachal Pradesh Government and private schools can read only class II text? Choose an option: (1). 44.6 (2). 57.5 (3). 72.8 (4). 70.0 Correct option : 1 Annual Status of Education Report (ASER) is an annual, nationwide survey of children’s ability to read simple text and do basic arithmetic that would engage ordinary citizens in finding out whether their children were learning. According to Annual Status Education Report (ASER) 2012-13 44.6 percent of class V students of Himachal Pradesh Government and private schools can read only class II text.

******ebook converter DEMO Watermarks*******

Q82. What was the exact constitutional status of the Indian Republic on 26th January 1950? Choose an option: (1). A Democratic Republic (2). A Sovereign Democratic Republic (3). A Sovereign Secular Democratic Republic (4). A Sovereign Socialist Secular Democratic Republic Correct option : 2 The exact constitutional status of the Indian Republic on 26th January 1950 was - a Sovereign Democratic Republic.

******ebook converter DEMO Watermarks*******

Q83. Disagreement between the two Houses of Parliament on the following bills can be resolved by the Houses in a joint sitting 1) An Ordinary Bill 2) Money Bill 3) Constitutional Amendment Bill 4) Private Member Bill Select the correct answer from the codes given below:- Choose an option: (1). 1 and 3 (2). 1, 3 and 4 (3). 1, 2 and 3 (4). 1 and 4 Correct option : 4 Disagreement between the two Houses of Parliament can be resolved by the Houses in a joint sitting. Not all bills can be refereed to a joint sitting of Parliament. There are two exception: 1. Money Bill Under the Constitution of India, money bills require approval of the Lok Sabha only. Rajya Sabha can make recommendations to Lok Sabha, which it is not required to accept. Even if Rajya Sabha doesn’t pass a money bill within 14 days, it is deemed to have been passed by both the Houses of Parliament after expiry of the above period. Therefore, a requirement to summon a joint session can never arise in the case of money bill. 2. Constitution Amendment Bill Article 368 of Indian constitution require that constitution of India can be amended by both houses of parliament by 2/3 majority. In case of disagreement between both houses, there is no provision to summon joint session of parliament.

******ebook converter DEMO Watermarks*******

Q84. In a National Park, protection is provided to: Choose an option: (1). Entire ecosystem (2). Flora and Fauna (3). Fauna only (4). Flora only Correct option : 2

******ebook converter DEMO Watermarks*******

Q85. Which of the following union territories do not have any representative in the Rajya Sabha? 1) Delhi 2) Chandigarh 3) Dadra and Nagar Haveli 4) Lakshadweep Select the correct answer from the codes given below: Choose an option: (1). 4 only (2). 3 and 4 (3). 2, 3 and 4 (4). 1, 3 and 4 Correct option : 3 Article 80 of the Constitution lays down the maximum strength of Rajya Sabha as 250, out of which 12 members are nominated by the President and 238 are representatives of the States and of the two Union Territories. The present strength of Rajya Sabha, however, is 245, out of which 233 are representatives of the States and Union territories of Delhi and Puducherry and 12 are nominated by the President. The members nominated by the President are persons having special knowledge or practical experience in respect of such matters as literature, science, art and social service.

******ebook converter DEMO Watermarks*******

Q86. Which among the following committees was the first to demand constitutional recognition for Panchayats? Choose an option: (1). Balwantrai Mehta Committee (2). Ashok Mehta Committee (3). G.V.K. Rao Committee (4). L.M. Singhvi Committee Correct option : 2 In December 1977, the Janta Government appointed a committee on Panchayati Raj institutions under the chairmanship of Ashok Mehta. It submitted its report in August 1978 and made 132 recommendations to revive and strengthen the declining Panchayati Raj System in the country. Its main recommendations are: 1. The 3-tier system of Panchayati Raj should be replaced by the 2-tier system: Zilla Parishad at the district level, and below it,the Mandal Panchayat consisting of a group of villages covering a population of 15000 to 20000. 2. A district should be the first point for decentralisation under popular supervision below the state level. 3. Zila Parishad should be the executive body and made responsible for planning at the district level. 4. A minister for Panchayati Raj should be appointed in the state council of ministers to look after the affairs of the Panchayati Raj institutions.

******ebook converter DEMO Watermarks*******

Q87. In which of the following states have the Panchayati Raj institutions been set up in conformity with the traditions and customs of the local people? 1) Assam 2) Goa 3) Nagaland 4) Arunachal Pradesh Select the correct answer from the codes given below: Codes:- Choose an option: (1). 1 and 3 (2). 1 and 4 (3). 3 and 4 (4). 1, 3 and 4 Correct option : 3 Following states have set the Panchayati Raj institutions in conformity with the traditions and customs of the local people: 1. Nagaland 2. Arunachal Pradesh

******ebook converter DEMO Watermarks*******

Q88. Who among the following former Presidents contested the election to the office of the President of India without resigning from the office of VicePresident of India? (1) Dr. Zakir Husain (2) V.V. Giri (3) R. Venkataraman (4) Dr. S.D. Sharma Select the correct answers from the codes given below: Codes:- Choose an option: (1). 1, 3 and 4 (2). 2, 3 and 4 (3). 1, 2 and 3 (4). 1 and 3 Correct option : 1 Following former Presidents contested the election to the office of the President of India without resigning from the office of Vice-President of India: (1) Dr. Zakir Husain (2) R. Venkataraman (3) Dr. S.D. Sharma

******ebook converter DEMO Watermarks*******

Q89. Which one of the following models/theories views public policy as a continuation of previous government activities with some modifications? Choose an option: (1). Rational Model (2). Incremental Model (3). Group Theory (4). Game Theory Correct option : 2 Incremental Model –It views public policy as a continuation of past government activities with only incremental modifications. This is done as policy makers do not have the time, information or money to investigate all the alternatives to existing policy. They do not have predictive capacities to know what all the consequences of each alternative will be, nor are they able to calculate cost-benefit ratios for alternative polices when many diverse factors are at stake.

******ebook converter DEMO Watermarks*******

Q90. The Khadi and Village Industries Commission was established during which of the Five Year Plans? Choose an option: (1). 4th (2). 2nd (3). 3rd (4). 6th Correct option : 2

******ebook converter DEMO Watermarks*******

Q91. In which field did Frederick Sainger win Nobel Prize twice? Choose an option: (1). Literature (2). Physics (3). Chemistry (4). Peace Correct option : 3

Frederick Sainger Frederick Sainger, a British Biochemist, is a double Nobel Prize winner. He won Nobel Prize 1. for developing a technique in 1958 to work out precise chemical structure of proteins. 2. for developing "Sanger Sequencing" technique in 1980

******ebook converter DEMO Watermarks*******

Q92. Which of the following types of glasses can cut-off ultraviolet rays? Choose an option: (1). Soda glass (2). Pyrex glass (3). Jena glass (4). Crookes glass Correct option : 4 Crookes glass In this glass mainly cerium oxide (CiO2) is present which sharply absorbs the ultraviolet rays from the sunlight so utilised in making lenses of sunglasses.

******ebook converter DEMO Watermarks*******

Q93. Which one of the following is an active component of clove oil? Choose an option: (1). Menthol (2). Eugenol (3). Methanol (4). Benzaldehyde Correct option : 2

******ebook converter DEMO Watermarks*******

Q94. In eye donation, which one of the following parts of donor’s eye is utilized? Choose an option: (1). Iris (2). Lens (3). Cornea (4). Retina Correct option : 3

Cross section through the eye and eyelids, including the lacrimal gland and third eyelid Whole Eye can not be utilized in eye donation, both the cornea and sclera can be utilised separately for ocular surgery. However, the entire eye may be used for research and educational purposes.

******ebook converter DEMO Watermarks*******

******ebook converter DEMO Watermarks*******

Cornea Cornea ensures an unobstructed passage of light rays into the eye and onto the retina, so that you can see. Unlike most tissues in the body, the cornea contains no blood vessels to nourish or protect it against infection. Instead, the cornea receives its nourishment from tears and the aqueous humor.

******ebook converter DEMO Watermarks*******

Q95. Which action of Raja Sansar Chand prompted the Hill Chiefs to invite Gurkhas to invade Kangra? Choose an option: (1). His failure to retrieve Kangra fort (2). His attack on Kahlur state and capture some territory (3). His alliance with Sikh Chief Jassa Singh Ramgarhia (4). His defeat at the hands of Sikh Chief Jai Singh of Kanhaiya Misl Correct option : 2

******ebook converter DEMO Watermarks*******

Q96. When was a Sanad granted to the Raja of Bilaspur by the British government confirming his possession on the right bank of Satluj? Choose an option: (1). February, 1846 (2). March, 1846 (3). October, 1847 (4). April, 1848 Correct option : 3

******ebook converter DEMO Watermarks*******

Q97. Court poet of which princely state of Shimla Hills is the author of Shashi Vansh Vinod? Choose an option: (1). Jubbal (2). Nurpur (3). Guler (4). Bilaspur Correct option : 4

******ebook converter DEMO Watermarks*******

Q98. Which mountain pass connects Spiti to Ladakh? Choose an option: (1). Kangla (2). Shipki (3). Rohtang (4). Manirang Correct option : 1 Kangla pass connects Lahual & Spiti of Himachal Pradesh and Zanskar Range in Ladakh

******ebook converter DEMO Watermarks*******

Q99. With which of the following is NGO named Vishakha is associated? Choose an option: (1). Dalits (2). Women (3). Physically challenged (4). Senior citizens Correct option : 2 Vishakha is a NGO for Women’s Education and Research. Vishakha aspires to create a participatory society where women, children and marginalized communities have opportunities to build their capacities and live a life of dignity and equity at all levels of self, community and the macro- context of development and governance. Reference. http://www.vishakhawe.org/pages.aspx?pname=VISION

******ebook converter DEMO Watermarks*******

2012 HPAS Solved Paper This PDF Booklet contains 2012 HPAS Solved Paper. Explanation is provided by Shunya Foundation Team. 2012 Solved Paper for HPAS General Studies

******ebook converter DEMO Watermarks*******

Q1. At which place in Shimla District is Herbal garden? Choose an option: (1). Dumreda (2). Jharag (3). Purag (4). Sainj Correct option : 1 The Department of Ayurveda has set up four herbal gardens and 33 ayurvedic hospitals in the state. The gardens are in Jogindernagar, Neri (Hamirpur), Dhumrera (Rohru) in Shimla district and Jungal Thalera in Bilaspur district.

******ebook converter DEMO Watermarks*******

Q2. Koderma, a place in Jharkhand , is famous for Choose an option: (1). Mica Mining (2). Photo goods (3). Cement factory (4). Leather goods Correct option : 1 India is a major producer of Mica in the world. It is the largest producer of sheet mica. According to British Geological Survey, the world’s largest deposit of mica is at Koderma district in the state of Jharkhand (India). About 95% of India’s mica is distributed in just three states of Jharkhand, Andhra Pradesh and Rajasthan.

******ebook converter DEMO Watermarks*******

Q3. What is Curiosity Rover? Choose an option: (1). Nasa’s reserach vechical sent to Mars (2). An intelligence agency of the USA (3). A medical reserach group in Europe (4). An associations of scientists of India Correct option : 1

Curiosity Mars Rover Curiosity Rover is a car-sized robotic rover exploring Gale Crater on Mars as part of NASA’s Mars Science Laboratory mission (MSL). Curiosity was launched from Cape Canaveral on November 26, 2011.

******ebook converter DEMO Watermarks*******

Q4. The continental drift theory was propounded by Choose an option: (1). F.B Taylor (2). Lowthin Green (3). Alfred wegner (4). J.A. Steers Correct option : 3 Continental drift was a theory that explained how continents shift position on Earth’s surface. It was propounded by Alfred Wegener in 1912.

******ebook converter DEMO Watermarks*******

Q5. What are stalagmites? Choose an option: (1). Erosional landforms (2). Depositional landsform (3). Volacanic landsforms (4). Solution holes Correct option : 1

Fig. Stalagmites Stalactites and stalagmites are types of mineral deposits found in caves that accrue through the processes of solution and deposition. Stalactites hang downwards from the ceiling of caves, whereas stalagmites rise upwards from the floor of a cave.

******ebook converter DEMO Watermarks*******

Q6. With which of the following is Losar festival associated? Choose an option: (1). The harvesting of crop (2). Tibetan new year (3). Birth of Gautam Budha (4). Birth of Padamsambhava Correct option : 2 The Tibetan New year, also known as Losar Festival, is the most important festival on the Tibetan calendar. In Ladakh, people celebrate Losar on the first day of the eleventh lunar month.

******ebook converter DEMO Watermarks*******

Q7. According to legend what is the vehicle of Kamdev? Choose an option: (1). Parrot (2). Crow (3). Pigeon (4). Sparrow Correct option : 1 Kamadeva is the Hindu deity of love. The god Kamadeva, is the incarnation of Pradyumna and son of the goddess Sri. He rides a parrot which is His vehicle. He is accompanied by Vasanta, the god of spring and Apsaras, Gandharvas and Kinnaras.

******ebook converter DEMO Watermarks*******

Q8. The drugs used to get relief from pain are called: Choose an option: (1). Antipyretics (2). Analgesics (3). Amtibiotics (4). Antiseptics Correct option : 2 Analgesics are drugs designed specifically to relieve pain. Tip Paracetamol and aspirin are common example of analgesic drugs.

******ebook converter DEMO Watermarks*******

Q9. Metals are good conductors of electricity because Choose an option: (1). They contain free electrons (2). The atoms are lightly packed (3). They have high melting point (4). All of the above Correct option : 1 Metals have free electrons which helps them in the conduction of electricity whereas non metals have no free electrons which makes them a poor conductor of electricity.

******ebook converter DEMO Watermarks*******

Q10. With which game is Ravi Rampaul associated? Choose an option: (1). Cricket (2). Badminton (3). Hockey (4). Kabaddi Correct option : 1 Ravi Rampaul is a West Indian cricketer. He is the first quick bowler of Indian descent to represent West Indies at international level

******ebook converter DEMO Watermarks*******

Q11. Against whom did Aurangzeb fight the battle at Samugarh? Choose an option: (1). Humayun (2). Dara (3). Murad Baksh (4). Akbar II Correct option : 2

Battle of Samugarh The battle of Samugarh was fought between Dara Shikoh and his two younger brothers Aurangzeb and Murad Baksh. It was a decisive battle in the struggle for the throne during the Mughal war of succession (1658–1659) between the sons of the Mughal Emperor Shah Jahan.

******ebook converter DEMO Watermarks*******

Q12. Who was Amrita Shergil? Choose an option: (1). Dancer (2). Poet (3). Painter (4). Politician Correct option : 3 Amrita Shergil was an eminent Indian painter. She is sometimes known as India’s Frida Kahlo.

******ebook converter DEMO Watermarks*******

Q13. In which year Shimla was declared as the summer capital of British India? Choose an option: (1). 1862 (2). 1863 (3). 1864 (4). 1865 Correct option : 3 The British called Shimla the "Queen of Hill Stations. In 1864 Lord Sir John Lawrence declared Shimla the summer capital of India.

******ebook converter DEMO Watermarks*******

Q14. Which of the following states Nathpa Jhakri Hydel Project under execution Choose an option: (1). Sikkim (2). Jammu and Kashmir (3). Himachal Pardesh (4). Uttarakhand Correct option : 3 The Nathpa Jhakri Hydroelectric Station of 1500 MW capacity is the country’s largest hydropower plant. The Nathpa Jhakri Dam is a concrete gravity dam on the Sutlej river in Himachal Pradesh.

******ebook converter DEMO Watermarks*******

Q15. Which country is called the 'Sugar Bowl' of the world? Choose an option: (1). India (2). Cuba (3). USA (4). Indonesia Correct option : 2 'Cuba' is commonly known as the "sugar bowl of the world". It is called by this name because at one time it was the largest producer of Sugar, but Brazil is currently the largest producer of Sugar.

******ebook converter DEMO Watermarks*******

Q16. Who was the first Caliph of Islam ? Choose an option: (1). Umar (2). Usman (3). Ali (4). Abu Bakr Correct option : 4 Abu Bakr the first Caliph of Islam. Abu bakr was the son of abu qahafa, and made his living as a merchant in Makkah.

******ebook converter DEMO Watermarks*******

Q17. What is INS Chakra? Choose an option: (1). Aircraft carrier (2). Submarine (3). Missile ship (4). Naval helicopter Correct option : 2 INS Chakra is a nuclear-powered attack submarine.

******ebook converter DEMO Watermarks*******

Q18. In coming years , skin related disorders will become more common due to : Choose an option: (1). Excessive use of detergents (2). Water pollution (3). Air pollution (4). Depletion of ozone layer Correct option : 4 The ozone layer protects us from the harmful effects of certain wavelengths of ultraviolet (UV) light from the Sun. Any significant decrease of ozone in the stratosphere would result in an increase of UV-B radiation reaching the Earth’s surface, and of skin cancers.

******ebook converter DEMO Watermarks*******

Q19. In the process of galvanization of an iron sheet is coated with: Choose an option: (1). Zinc (2). Aluminium (3). Tin (4). Magnesium Correct option : 1 Galvanization is the process of applying a protective zinc coating to steel or iron, to prevent rusting. The most common method is hot-dip galvanizing, in which parts are submerged in a bath of molten zinc.

******ebook converter DEMO Watermarks*******

Q20. One liter of water is equivalent to: Choose an option: (1). 1.5 kg (2). 1.25 kg (3). 0.25 kg (4). 0.90 kg Correct option : 4 One liter of water is equivalent to 0.90 kg.

******ebook converter DEMO Watermarks*******

Q21. A transformer is used for: Choose an option: (1). Increasing DC voltage (2). Decreasing DC voltage (3). Converting DC into AC (4). Increasing or decreasing AC voltage Correct option : 4 A transformer is an electrical device that is used to change the voltage in alternating current (AC) electrical circuits. It is used to either raise or lower voltages and currents in an electrical circuit.

******ebook converter DEMO Watermarks*******

Q22. Where is KEE monastry? Choose an option: (1). Spiti (2). Kinnaur (3). Lahaul (4). Naggar Correct option : 1

Key Gompa or Kee Monastery is a Tibetan Buddhist monastery located on top of a hill at an altitude of 4,166 metres above sea level in Spiti valley, Himachal Pradesh. It is the highest monastery in the world. It is also known as "Little Tibet".

******ebook converter DEMO Watermarks*******

Q23. Which type of pollution is caused by two - wheelers? Choose an option: (1). Air pollution (2). Soil pollution (3). Air and soil pollution (4). Water pollution Correct option : 1 Two-wheelers cause air pollution.

They contribute the biggest chunk of air pollution. They contribute 41% of particulate and 67% of nitrogen oxide levels in the air. Vehicles are more threatening than other sources of pollution as they discharge emissions directly into the air.

******ebook converter DEMO Watermarks*******

Q24. Chernobyl disaster was caused by a : Choose an option: (1). Nuclear weapon accident (2). Nuclear waste leak (3). Nuclear reactor accident (4). Nucelear test Correct option : 3 The Chernobyl disaster was a catastrophic Nuclear reactor accident. It occurred on 25–26 April 1986 in a town in the Soviet Union (USSR).

******ebook converter DEMO Watermarks*******

Q25. First share market in India was established in : Choose an option: (1). Delhi (2). Kolkata (3). Chenni (4). Mumbai Correct option : 4 The Bombay Stock Exchange (BSE) is an Indian stock exchange located at Dalal Street, Mumbai (formerly Bombay). Established in 1875, the BSE (formerly known as Bombay Stock Exchange Ltd.) is Asia’s first stock exchange.

******ebook converter DEMO Watermarks*******

Q26. Who founded the revolutionary organization named 'Abhinava Bharat' ? Choose an option: (1). Hardayal (2). Bhai Parmanand (3). Hadgewar (4). Savankar Brothers Correct option : 4 Abhinav Bharat was a secret society founded by Vinayak Damodar Savarkar and his brother Ganesh Damodar Savarkar in 1903. It carried out a few assassinations of British officials, after which the Savarkar brothers were convicted and imprisoned. The society was formally disbanded in 1952.

******ebook converter DEMO Watermarks*******

Q27. At what interval is Bhunda festival of Nirmand held? Choose an option: (1). 2 years (2). 6 years (3). 12 years (4). 15 years Correct option : 3

Bhunda festival is celebrated in Nirmand region of Himachal Pradesh. Famous by the name of Old Narmedh Yagya, Bhunda is celebrated every after 12 years. This festival mainly revolves around Beda tribes of Himachal Pradesh, who have a major role to play in the whole Bhunda ceremony.

******ebook converter DEMO Watermarks*******

Q28. Who is the author of 'Abode of God'? Choose an option: (1). B.N. Datar (2). Andrew Wilson (3). Ram Rahul (4). William Moorcraft Correct option : 2

******ebook converter DEMO Watermarks*******

Q29. Which of the following is not a tributary of the Beas? Choose an option: (1). Suketi (2). Banganga (3). Ali (4). Uhl Correct option : 3

******ebook converter DEMO Watermarks*******

Q30. With which region of Himachal Pardesh is Sih ballet associated? Choose an option: (1). Kullu-Manali (2). Pangi- Bharmaur (3). Nurpur-Kangra (4). Rohuru-Jubbal Correct option : 4

******ebook converter DEMO Watermarks*******

Q31. Which Sultan of Delhi helped Raja Megh Chand of Bilaspur to regain his lost kingdom? Choose an option: (1). Alauddin Khalji (2). Altamash (3). Balban (4). Ferozshah Tughlak Correct option : 2

******ebook converter DEMO Watermarks*******

Q32. Which Raja of Mandi built the Darbar Hall in 1906? Choose an option: (1). Bhawani Sen (2). Shamsher Sen (3). Ishwar Sen (4). Prakash Chand Correct option : 1

******ebook converter DEMO Watermarks*******

Q33. Who won the 2012 Himachal Pardesh Vidhan Sabha election by the highest margin? Choose an option: (1). Virbhardra Singh (2). Mohan Lal Brakta (3). Rohit Thakur (4). Sanjay Rattan Correct option : 2

******ebook converter DEMO Watermarks*******

Q34. Youth of which age group are eligible to avail the skill development allowance announced by the Himachal Pradesh Government in 2013-14 budget? Choose an option: (1). 18-25 Years (2). 20-25 Years (3). 20-30 Years (4). 25-35 Years Correct option : 4

******ebook converter DEMO Watermarks*******

Q35. Match the following proposed rope-ways correctly and choose the correct answer from the codes given below: Column-I Column-II (1) Dharmshala (a) Bijli Mahadev (2) Bhunter (b) Rohtnag (3) Palchan (c) Triund (4) Village Jia (d) Adi Himani Chamunda Choose an option: (1). 1-b, 2-a, 3-c , 4-d (2). 1-a, 2-b, 3-c, 4-d (3). 1-c, 2-b, 3-d, 4-a (4). 1-c, 2-a, 3-b, 4-d Correct option : 4 Dharmsala – Triund, Bhunter – Bijli Mahadev, Palchan – Rohtang, Village Jia – Adi Himani Chamunda

******ebook converter DEMO Watermarks*******

Q36. According to 2008-09 data, which district of Himachal Pradesh has recorded the highest percentage in the production of pulses? Choose an option: (1). Kangra (2). Mandi (3). Shimla (4). Chamba Correct option : 3

******ebook converter DEMO Watermarks*******

Q37. Which one of the following is not a precondition for implementing national programme for education of girls at the elementary level? Choose an option: (1). It is educationally population backward block (2). The SC/ST population is more than 50% (3). The ruarl female literacy rate is below national average (4). Gender gap is above the national level Correct option : 2

******ebook converter DEMO Watermarks*******

Q38. Which is scheme has been introduced in place of Kishori Shakti Yojana in some district of Himachal Pradesh? Choose an option: (1). Beti Hai Anmol (2). Rajiv Gandhi Schme for Empowerment of Adolescent Girls (3). Self - Employment Scheme for Women (4). Mukhya Mantri Kanyadan Yojana Correct option : 2

******ebook converter DEMO Watermarks*******

Q39. Which sector had highest outlay (in % terms ) in the annual plan of 2011-2012? Choose an option: (1). Agricultural (2). Transport (3). Energy (4). Social Services Correct option : 4

******ebook converter DEMO Watermarks*******

Q40. Beside Kinnaur and Lahaul Spiti, which district of Himachal Pradesh has recorded the lowest percentage of urban population in the 2011 census? Choose an option: (1). Kangra (2). Mandi (3). Chamba (4). Hamirpur Correct option : 1

******ebook converter DEMO Watermarks*******

Q41. What was the percentage of female literacy in Himachal Pradesh in 1951? Choose an option: (1). 2% (2). 6% (3). 8% (4). 10% Correct option : 1

******ebook converter DEMO Watermarks*******

Q42. Match the following: a) Indica – 1) Ibn Batuta, b) Historica – 2) Herodotus, c) Safarnama – 3) Alberuni d) Tarikh-al-Hind – 4) Magasthenes a b c d Choose an option: (1). 1 2 3 4 (2). 4 3 1 2 (3). 4 2 1 3 (4). 2 3 4 1 Correct option : 3 Indica – Magasthenes, Historica – Herodotus, Safarnama – Ibn Batuta, Tarikh-al-Hind – Alberuni

******ebook converter DEMO Watermarks*******

Q43. Whom did emperor Ashoka send to Lanka to promote Buddhism? Choose an option: (1). Mahindera and Sona (2). Sona and Uttra (3). Uttra and Sanghamitra (4). Mahindera and Sanghamitra Correct option : 4

******ebook converter DEMO Watermarks*******

Q44. There are four Upvedas. Three of them are Ayurveda, Dhanaurveda and Sthapatya veda. Which is the fourth? Choose an option: (1). Yoga Veda (2). Nyaya Vada (3). Raja veda (4). Gandharva Veda Correct option : 4

******ebook converter DEMO Watermarks*******

Q45. Which among the following scholar belong to the later Vedic period? Choose an option: (1). Gagri (2). Lopmudra (3). Shraddha (4). Yami Correct option : 1

******ebook converter DEMO Watermarks*******

Q46. Whom did the Huns defeat to establish their in India? Choose an option: (1). Samudragupta (2). Harshavardhana (3). Kanishka (4). Skandagupta Correct option : 4

******ebook converter DEMO Watermarks*******

Q47. The first Rani of Mewar who committed johar was Padmini. Who was the second? Choose an option: (1). Hasnabai (2). Raj Kumari (3). Karnavati (4). Karma Khanti Correct option : 3

******ebook converter DEMO Watermarks*******

Q48. At which place did the battle between Babur and Rana Sanga take place 1527 ? Choose an option: (1). Kannauj (2). Khanua (3). Mewat (4). Sirohi Correct option : 2

******ebook converter DEMO Watermarks*******

Q49. Who organized the Red Shirts Movement ? Choose an option: (1). Khan Abdual Gaffar Khan (2). Abdul Rahim (3). Fazal-ul-Haq (4). Shaukat Ali Correct option : 1

******ebook converter DEMO Watermarks*******

Q50. Who is associated with the moderate strand of the Khilafat movement of 1919-20? Choose an option: (1). Chotani (2). Ali Brothers (3). Hasrat Mohani (4). Journalists and Ulema Correct option : 1

******ebook converter DEMO Watermarks*******

Q51. The marked of Moshad the spy agency of Israel is that it has as its member a large number of Choose an option: (1). Artists (2). Convicts (3). Woman (4). Non-Israelis Correct option : 3

******ebook converter DEMO Watermarks*******

Q52. What is the major religion of Myanmar? Choose an option: (1). Hinduism (2). Islam (3). Rohingya (4). Buddhism Correct option : 4 Myanmar (Burma) is a multi-religious country. There is no official state religion, but the government shows preference for Buddhism, which is the majority religion. According to the 2014 census of the Burmese government, Buddhism is the dominant religion practiced by 88% of the population.

******ebook converter DEMO Watermarks*******

Q53. With which field is Malala Yousafzai associated? Choose an option: (1). Girl education (2). Protection of environment (3). Welfare of the widows (4). Empowerment of the poor Correct option : 1

******ebook converter DEMO Watermarks*******

Q54. Identify the country whose seat of legislature, executive and judiciary is at three different cities? Choose an option: (1). Canada (2). Russia (3). South Affrica (4). Somalia Correct option : 3

******ebook converter DEMO Watermarks*******

Q55. Which one of the following refused the Nobal Prize awarded to him? Choose an option: (1). Henry Kissinger (2). Jimmy Carter (3). Wooddrow Wilson (4). Theodre Roosevelt Correct option : 1

******ebook converter DEMO Watermarks*******

Q56. When was the World Anti-Doping Agency(WADA) set up? Choose an option: (1). 1999 (2). 2003 (3). 2006 (4). 2009 Correct option : 1

******ebook converter DEMO Watermarks*******

Q57. Which of the following states records the highest proportion of scheduled castes population ? Choose an option: (1). Parts of Uttar Pardesh (2). Punjab (3). Haryana (4). Gujarat Correct option : 2

******ebook converter DEMO Watermarks*******

Q58. The most ideal region for the cultivation of cotton in India is Choose an option: (1). The Brahmaputra Valley (2). Indo-Gangetic plain (3). The Deccan plateau (4). The Rann of Kutch Correct option : 3 The Deccan Plateau is a large plateau in India which covers most of the southern part of the country. It is between three mountain ranges and extends over eight Indian states; in particular the states of Telangana, Maharashtra, Andhra Pradesh, Karnataka, Kerala and Tamil Nadu are on this plateau. Cotton cultivation is ideal in Deccan plateau, because: 1. Presence of black cotton soil 2. Temperature is between 20°C-32°C which is good for cotton 3. Rainfall (50-200 cms) is well distributed and if not enough irrigation is possible by use of canals.

******ebook converter DEMO Watermarks*******

Q59. Which of the following factors is responsible for the rapid growth of sugar production in south India as compared to North India? (1) Higher per acre field or sugarcane (2) Higher sucrose content of sugarcane (3) Lower labour cost (4) Longer crushing period Choose an option: (1). 1 only 2 (2). 1 2 and 3 (3). 1 and 3 , 4 (4). 1, 2 and 4 Correct option : 4

******ebook converter DEMO Watermarks*******

Q60. Which of the following state groups is the largest producer of iron-ore in india? Choose an option: (1). Orissa, M.P. and Jharkhand (2). M.P. , Maharastra and Punjab (3). Maharashtra , Orissa and A.P. (4). Bihar, West Bengal and U.P. Correct option : 1

******ebook converter DEMO Watermarks*******

Q61. Whose government was defeated for the first time on a vote of confidence? Choose an option: (1). Chandra Shekhar government (2). V.P. Singh government (3). Charan Singh government (4). Deve Gowda Government Correct option : 2

******ebook converter DEMO Watermarks*******

Q62. The Question Hour in the House of the people is followed by Choose an option: (1). Supplementary question (2). Starred question (3). Unsterred question (4). Short notice question Correct option : 4

******ebook converter DEMO Watermarks*******

Q63. CENVAT is assoicated with : Choose an option: (1). Direct Tax (2). Income Tax (3). Indirect Tax (4). Service Tax Correct option : 3

******ebook converter DEMO Watermarks*******

Q64. The 73rd and 74th constitutional Amendments were during the primeministership of: Choose an option: (1). Rajiv Gandhi (2). Narasimha Rao (3). V.P. Singh (4). Atal Bihari Vajpayee Correct option : 2

******ebook converter DEMO Watermarks*******

Q65. The Indian Constitution refers to minorities based on Choose an option: (1). Religion or language (2). Religion or region (3). Region or language (4). Language or culture Correct option : 1

******ebook converter DEMO Watermarks*******

Q66. Which Indian company has been included for the first time in USA’s index NASDAQ-100? Choose an option: (1). Infosys (2). Tata Motors (3). ICICI Bank (4). Videsh Sanchar Nigam LTD Correct option : 1

******ebook converter DEMO Watermarks*******

Q67. UNDP has introduced a new poverty index known as: Choose an option: (1). Human Poverty Index (2). Poorest among Poor Index (3). Social Poverty Index (4). Multi- dimensional Poverty index Correct option : 4

******ebook converter DEMO Watermarks*******

Q68. The excess discharge of fertilizers into water bodies result in: Choose an option: (1). Silt (2). Death of hydrophytes (3). Eutrophication (4). Growth of fish Correct option : 3

******ebook converter DEMO Watermarks*******

Q69. To which community/organisation maximum goods are exported from India? Choose an option: (1). EEC (2). Commonwealth Nation (3). SAFTA (4). OPEC Correct option : 4

******ebook converter DEMO Watermarks*******

Q70. The antiknock agent added to unleaded petrol is: Choose an option: (1). Tetraethyl lead (2). Tetramethyl lead (3). Dibromoethane (4). Methyl tertiary butyl ether Correct option : 1

******ebook converter DEMO Watermarks*******

Q71. 3-4 benzopyrene causes Choose an option: (1). Leukemia (2). Cytosilicosis (3). Lung cancer (4). Tuberculosis Correct option : 3

******ebook converter DEMO Watermarks*******

Q72. Natural silk is a : Choose an option: (1). Polypeptide (2). Polysaccharide (3). Polyvinyl Chloride (4). Polyacrylonitrite Correct option : 1

******ebook converter DEMO Watermarks*******

Q73. Which of the following states have recently (2013) got separate high courts? (1) Sikkim (2) Manipur (3) Meghalaya (4) Tripura select the correct answer from the codes given below: codes Choose an option: (1). 1 and 2 (2). 1, 2 and 3 (3). 1, 2 and 4 (4). 2 and 3, 4 Correct option : 4

******ebook converter DEMO Watermarks*******

Q74. Most hazardous metal pollution of automobile exhaust is: Choose an option: (1). Cd (2). Hg (3). Copper (4). Lead Correct option : 4 Lead used to be added to fuel to create leaded petrol (hence why we have unleaded petrol now). It interferes with the production of red blood cells, causes high blood pressure and inhibits development in children. Lead no longer appears in exhaust emissions except in a handful of countries that still use leaded petrol. Leaded petrol is not sold in India anymore. India banned leaded petrol in March 2000.

******ebook converter DEMO Watermarks*******

Q75. If BOD of a water is found very high, this means water: Choose an option: (1). Is clean (2). Is highly polluted (3). Is having algae (4). Contains dissolved minerals Correct option : 2 Biochemical oxygen demand (a.k.a BOD) is the amount of dissolved oxygen demanded by aerobic biological organisms to break down organic material present in a given water sample at certain temperature over a specific time period.

BOD Value

BOD Reference Value Meaning

Inference

1-2 ppm

very good

There will not be much organic waste present in the water supply

3-5 ppm

moderately clean

A water supply with a BOD level of 3-5 ppm is considered moderately clean.

6-9 ppm

somewhat polluted

there is usually organic matter present and bacteria are decomposing this waste

ppm = parts per million

******ebook converter DEMO Watermarks*******

Q76. Minamata disease affects the : Choose an option: (1). Circulatory system (2). Skeletal system (3). Nervous system (4). Respiratory system Correct option : 3

******ebook converter DEMO Watermarks*******

Q77. Chemical which cause bone cancer and degeneration of tissue is : Choose an option: (1). I-131 (2). Ca-40 (3). I-127 (4). Str-90 Correct option : 4

******ebook converter DEMO Watermarks*******

Q78. Who was the Chairman of the committee on pricing and taxation of petroleum products(2013)? Choose an option: (1). Raja J. Chelliah (2). C. Rangarajan (3). Y.V Reddy (4). Abid Husssain Correct option : 2

******ebook converter DEMO Watermarks*******

Q79. When was World Trade Organization set up by the member countries of the United Nations to promote trade among countries? Choose an option: (1). 1995 (2). 1991 (3). 1985 (4). 1999 Correct option : 1

******ebook converter DEMO Watermarks*******

Q80. Which of the following commissions/committees have recommended legal protection to whistle blowers? (1) Law Commission (2) Second Administrative Reforms Commission (3) Election Commission (4) Vohra Committee Select the correct answer from the codes given below: codes Choose an option: (1). 1 only 2 (2). 1 and 3 (3). 2, 3 and 4 (4). 1, 3 and 4 Correct option : 1

******ebook converter DEMO Watermarks*******

Q81. For which of the following posts has the Supreme court directed the central government that not only civil servants but other persons with impeccable integrity should also be considered? Choose an option: (1). Chief Election Commissioner (2). Central Vigilance Commissioner (3). Director of Central Bureau of Investigation (4). Comptroller and Auditor General of India Correct option : 2

******ebook converter DEMO Watermarks*******

Q82. Which of the following occupied the highest place in growth rate during the 11th Plan period 2007-2012? Choose an option: (1). Agricultural sector (2). Service sector (3). Industrial sector (4). Mining Correct option : 2

******ebook converter DEMO Watermarks*******

Q83. Which of the following can be remolded time and again without producing any change? Choose an option: (1). PVC (2). Bakelite (3). Melmac (4). Urea-formaldehyde resin Correct option : 1

******ebook converter DEMO Watermarks*******

Q84. Most commonly used tranquillizers are derivatives of: Choose an option: (1). Acetic acid (2). Salicylic acid (3). Barbituric acid (4). Sulphanilamide Correct option : 3

******ebook converter DEMO Watermarks*******

Q85. At what capacity was P.C. Mahalanobis associated with the formulation of the economic policy of independent India ? Choose an option: (1). As an applied statistician (2). As Deputy Chairman of the Planing commission (3). As a leading industrialist (4). As a minister of the Union cabinet Correct option : 1

******ebook converter DEMO Watermarks*******

Q86. The range of normal human hearing lies between: Choose an option: (1). 500 Hz to 10000 Hz (2). 20 Hz to 20000 Hz (3). 5000 Hz to 6000 Hz (4). 50 Hz to 15000 Hz Correct option : 2 The range of human hearing is generally considered to be 20 Hz to 20 kHz, but it is far more sensitive to sounds between 1 kHz and 4 kHz. The human hearing range depends on both the pitch (i.e. whether it is high or low) of the sound and the loudness (also called as amplitude) of the sound. Pitch is measured in Hertz (Hz) and loudness is measured in decibels (dB). Caution Sounds that are more than 85dB can be dangerous for your hearing in the case of prolonged exposure.

******ebook converter DEMO Watermarks*******

2011 HPAS Solved Paper This PDF Booklet contains 2011 HPAS Solved Paper. Explanation is provided by Shunya Foundation Team. 2011 Solved Paper for General Studies Paper 1

******ebook converter DEMO Watermarks*******

Q1. Who was the first Indian to declare that "Freedom is my birth right"? Choose an option: (1). Mahatma Gandhi (2). Bal Gangadhar Tilak (3). Lala Lajpat Rai (4). Chandra Shekhar Azad Correct option : 2 Bal Gangadhar Tilak was the first Indian to declare that "Freedom is my birth right".

******ebook converter DEMO Watermarks*******

Q2. At which place was Vardhman Mahavir born? Choose an option: (1). Gaya (2). Kundgram (3). Kapilvastu (4). Patliputra Correct option : 2 About Mahavir 1. Mahavir is 24th Tirthankara of Jainism. 2. According to the Kalpasutra, Mahavira was born at Kundagrama in the state of Bihar. 3. According to Digambara Jains, Mahavira was born in 582 BC. 4. Jainism 5 main teachings are Ahimsa, Satya, Asteya, Brahmacharya and Aparigraha.

******ebook converter DEMO Watermarks*******

Q3. Zawar mines are important for: Choose an option: (1). Bauxite (2). Tungsten (3). Zinc (4). Lead Correct option : 3 Zawar is located in Udaipur District, Rajasthan. It is one of the oldest zinc mines of the world.

******ebook converter DEMO Watermarks*******

Q4. Which Mughal ruler imprisoned his stepmother after the death of his father? Choose an option: (1). Aurangzeb (2). Shahjahan (3). Jahangir (4). Humayun Correct option : 2 Shah Jahan (grandson of Akbar) was the fifth Mughal emperor, who reigned from 1628 to 1658. His first act as ruler was to execute his chief rivals and imprison his step mother Nur Jahan.

******ebook converter DEMO Watermarks*******

Q5. Who founded the All India Depressed Classes Federation? Choose an option: (1). B.R. Ambedkar (2). Narayan Gurru (3). Jyotiba Phule (4). Lala Lajpat Rai Correct option : 1

Dr. B. R. Ambedkar, who belonged to one of the scheduled castes, devoted his entire life to fighting against caste tyranny. He founded All India Depressed Classes Federation for this purpose.

******ebook converter DEMO Watermarks*******

Q6. From which place did Mahatma Gandhi ji started Dandi March to the sea? Choose an option: (1). Wardha (2). Sabarmati (3). Kheda (4). Ahmedabad Correct option : 2 The Salt March, also known as the Dandi March and the Dandi Satyagraha, was an act of nonviolent civil disobedience in colonial India initiated by Mohandas Karamchand Gandhi to produce salt from the seawater in the coastal village of Dandi, as was the practice of the local populace until British officials introduced taxation on salt production. Gandhi led the Dandi March from his base, Sabarmati Ashram, near the city of Ahmedabad till the Village Dandi around 390 km from there.

******ebook converter DEMO Watermarks*******

Q7. Which of the following is not a measures of Human Development Index? Choose an option: (1). Life Expectancy (2). Sex Ratio (3). Litracy Rate (4). Gross Enrolment Correct option : 1 The Human Development Index (HDI) is a summary measure of average achievement in key dimensions of human development: a long and healthy life, being knowledgeable and have a decent standard of living. The health dimension is assessed by life expectancy at birth, the education dimension is measured by mean of years of schooling for adults aged 25 years and more and expected years of schooling for children of school entering age. The standard of living dimension is measured by gross national income per capita.

******ebook converter DEMO Watermarks*******

Q8. At which place in Mandi District has the Department of Ayurveda set-up herbal garden? Choose an option: (1). Sarkaghat (2). Sunder Nagar (3). Karsog (4). Joginder Nagar Correct option : 4 Department of Ayurveda has setup Herbal Garden at Joginder Nagar, Mandi. There are total four Herbal Garden Setup in Himachal Pradesh at below locations 1. Jungal Thalera in Bilaspur 2. Joginder Nagar, Mandi 3. Neri, Hamirpur 4. Dhumrera (Rohru) in Shimla The purpose pf these Herbal Gardens is conservation and promotion of herbal wealth of Himalayan State.

******ebook converter DEMO Watermarks*******

Q9. In which district of Himachal Pradesh is Mulgun valley? Choose an option: (1). Kinnaur (2). Lahul Spiti (3). Kullu (4). Chamba Correct option : 1

Mulgun valley is located in Kinnaur district.

******ebook converter DEMO Watermarks*******

Q10. Which statement is true about Ellora caves? Choose an option: (1). All of them are Buddhist caves (2). Majority of them are Buddhist caves (3). Majority of them are Hindu caves (4). All of them are Hindu caves Correct option : 3

Fig. Ellora caves Ellora is one of the largest rock-cut monastery-temple caves complexes in the world, and a UNESCO World Heritage Site in Maharashtra. The site presents monuments and artwork of Buddhism, Hinduism and Jainism from the 600-1000 CE period. Most of Ellora caves are Hindu caves. The crowning achievement of Ellora is the great Kailasanatha temple. The great trimurti figure of Shiva is situated in Elephanta caves.

******ebook converter DEMO Watermarks*******

Q11. Who was the founder of Arya Samaj? Choose an option: (1). Swami Vivekananda (2). Swami Agnivesh (3). Swami Dayanand (4). Pandit Vishnu Shashtri Correct option : 3 Maharashi Dayanand Saraswati founded Arya Samaj system in 1875 at Bombay. He propounded the slogan 'Back to the Vedas’ to help people get rid of the false beliefs and adopt an objective, rational and progressive way of life. He openly stood up against all the social evils such as discrimination on grounds of caste, creed, sex, economic status and social bigotry, male chauvinism and religious dominance of higher castes over the backward castes and classes. Arya Samaj is an Indian Hindu reform movement that promotes values and practices based on the infallibility of the Vedas.

******ebook converter DEMO Watermarks*******

Q12. Under the constitution of India, the power to issue a writ of Habeas Corpus is vested in: Choose an option: (1). The District Courts (2). The High Courts only (3). The Supreme Court only (4). Both the High Courts and the Supreme Court Correct option : 4 Habeas Corpus literally means ‘to have the body of’. Via this writ, the court can cause any person who has been detained or imprisoned to be physically brought before the court. The court then examines the reason of his detention and if there is no legal justification of his detention, he can be set free. Under the constitution of India, the power to issue a writ of Habeas Corpus is vested in both the High Courts and the Supreme Court.

******ebook converter DEMO Watermarks*******

Q13. On which river is Chamba Pattan Bridge which being without pillars is first of this kind in Himachal Pradesh? Choose an option: (1). Ravi (2). Satluj (3). Yamuna (4). Beas Correct option : 4 Beas River rises at an elevation of 14,308 feet from Beas Kund near Rohtang Pass in the western Himalayas. From there it flows south through the Kullu Valley, receiving tributaries from the flanking mountains, and then turns west to flow past Mandi into the Kangra Valley. After crossing the valley, the Beas enters Punjab state. Chamba Pattan Bridge is situated on Beas river. It is first of this kind in Himachal Pradesh, as it is built without pillars.

The Beas River was the approximate eastern limit of Alexander the Great’s invasion of India in 326 BCE.

******ebook converter DEMO Watermarks*******

******ebook converter DEMO Watermarks*******

Q14. Participatory Notes (PNs) are associated with which one of the following? Choose an option: (1). Consolidated Fund of India (2). Foreign Institutional Investors (3). United Nations Development Programme (4). Kyoto Protocol Correct option : 2 Participatory Notes commonly known as P-Notes or PNs are instruments issued by registered foreign institutional investors (FII) to overseas investors, who wish to invest in the Indian stock markets without registering themselves with the market regulator, the Securities and Exchange Board of India - SEBI. SEBI permitted foreign institutional investors to register and participate in the Indian stock market in 1992. Investing through P-Notes is very simple and hence very popular amongst foreign institutional investors.

******ebook converter DEMO Watermarks*******

Q15. Silicon carbide(SiC) is known as: Choose an option: (1). Quartz (2). Tridynite (3). Corundum (4). Carborundum Correct option : 4 Silicon carbide (SiC), also known as carborundum, is a compound of silicon and carbon with chemical formula SiC. Silicon Carbide is the only chemical compound of carbon and silicon. Silicon carbide is an excellent abrasive and has been produced and made into grinding wheels and other abrasive products. It is used in abrasives, refractories, ceramics, and numerous high-performance applications. The material can also be made an electrical conductor and has applications in resistance heating, flame igniters and electronic components.

******ebook converter DEMO Watermarks*******

Q16. Where was the capital of Kullu in the earlier times? Choose an option: (1). Brahmpur (2). Jagatsukh (3). Murgul (4). Trilokinath Correct option : 2 Kullu was probably the most ancient state next to Kashmir and Kangra. According to known history, it was founded in the first century of Christian era by one Behangamani Pal whose forefatheres origianlly came from Tripura and had migrated from there Allahabad and then to Mayapuri near Hardwar. In earlier times, the capital of Kullu was Jagatsukh.

******ebook converter DEMO Watermarks*******

Q17. In the coast of which state is Wheeler Island? Choose an option: (1). Goa (2). Gujrat (3). Orissa (4). Tamilnadu Correct option : 3 Dr. Abdul Kalam Island, formerly known as Wheeler Island, is an island off the coast of Odisha. Abdul Kalam Island is located close to the Gahirmatha Marine Sanctuary, the world’s largest rookery of the endangered Olive Ridley sea turtles.

******ebook converter DEMO Watermarks*******

Q18. Five rings linked together constitute the symbol of Olympic Games. Each ring is of a different color. Two colors are blue and black. Which are the other three colors? Choose an option: (1). Green, Red, Violet (2). Red, Violet, Yellow (3). Green, Pink, Yellow (4). Green, Yellow and Red Correct option : 4 There are five Olympic rings in five colours (plus the white background). The rings are five interlocking rings, colored blue, yellow, black, green and red on a white field, known as the "Olympic rings". The symbol was originally designed in 1912 by Baron Pierre de Coubertin, co-founder of the modern Olympic Games. He appears to have intended the rings to represent the five participating continents: Africa, Asia, America, Australia and Europe.

******ebook converter DEMO Watermarks*******

Q19. Who translated the collection of South Indian tales into Sanskrit and titled the work as ‘Kathasaritasager’? Choose an option: (1). Somdev (2). Kalhana (3). Jaideva (4). Shankaracharya Correct option : 1

******ebook converter DEMO Watermarks*******

Kathasaritasager (कथास र

ागर)

The Kathasaritasager is a famous 11th-century collection of Indian legends, fairy tales and folk tales as retold in Sanskrit by a Shaiva named Somadeva.

******ebook converter DEMO Watermarks*******

Q20. Arrange the following ruling dynasties of ancient India in correct chronological order: (i) Mauryas (ii) Guptas (iii) Nandas (iv) Kushans Choose an option: (1). (iv), (ii), (i), (iii) (2). (i), (iii), (iv), (i) (3). (iii), (i), (iv), (ii) (4). (iii), (ii), (iv), (i) Correct option : 3 1. Nanda Dynasity (345 BCE) 2. Mauryas Dynasity (322 BCE) - Chandragupta Maurya founded the Mauryan Empire by overthrowing the Nanda Dynasty. 3. Kushan Empire (1st centuary BCE) 4. Gupta Empire (320 to 550 CE) - The Gupta Empire was an ancient Indian empire founded by Sri Gupta. The empire existed at its zenith from approximately 320 to 550 CE and covered much of the Indian subcontinent. The peace and prosperity created under the leadership of the Guptas enabled the pursuit of scientific and artistic endeavors. The definition of chronological is arranged in the order it happened.

******ebook converter DEMO Watermarks*******

Q21. In which year was the Partition of Bengal annulled? Choose an option: (1). 1909 A.D. (2). 1910 A.D. (3). 1911 A.D. (4). 1912 A.D. Correct option : 3

******ebook converter DEMO Watermarks*******

Partition of Bengal The decision to effect the Partition of Bengal (Bengali: ব ভ ) was announced in July 1905, by the Viceroy of India, Lord Curzon. The partition animated the Hindus and led the Muslims to form their own national organization on communal lines. Bengal was reunited in 1911, in an effort to both appease the Bengali sentiment and have easier administration.

******ebook converter DEMO Watermarks*******

Meaning of Annulled Declare invalid, "The contract was annulled".

******ebook converter DEMO Watermarks*******

Q22. Haem is the important biologically in the myoglobin which is used to store: Choose an option: (1). Nitrogen in muscles (2). Chlorine in muscles (3). Oxygen in muscles (4). Carbon dioxide in muscles Correct option : 3

******ebook converter DEMO Watermarks*******

Haem 1. The component of haemoglobin responsible for binding oxygen, consists of an iron ion that binds oxygen and a porphyrin ring that binds the globin molecules; one molecule binds one molecule of oxygen. 2. A complex red organic pigment containing iron and other atoms to which oxygen binds

******ebook converter DEMO Watermarks*******

Q23. Which treaty signaled the departure of the Gurkhas and arrival of the British on the political canvas of Shimla Hill states? Choose an option: (1). Treaty of Sagauli (2). Treaty of Lahore (3). Treaty of Malsun (4). Treaty of Ramgarh Correct option : 1

******ebook converter DEMO Watermarks*******

Treaty of Sagauli Treaty of Sagauli established the boundary line of Nepal, was signed on 2 December 1815 between the East India Company and King of Nepal following the Anglo-Nepalese War of 1814-16.

******ebook converter DEMO Watermarks*******

Q24. Mediterranean agriculture is: Choose an option: (1). Dependent upon large quantities of summer rainfall (2). Known for grapes, olives, oranges and figs (3). One of the least productive of the agricultural regions (4). Found only in Southern Europe and Northern Africa Correct option : 2

Mediterranean agriculture Mediterranean agriculture is an agro-ecological strategy, an adjustment to particular climatic conditions in Mediterranean zones: mild, humid winter with no or very little frost, and a warm, dry summer.

Crops in Mediterranean agriculture Subsistence agriculture occurs side by side with commercial farming. Many crops such as wheat, barley and vegetables are raised for domestic consumption, while others like citrus fruits, olives, and grapes are mainly for export. The Mediterranean lands are also known as 'orchard lands of the world'.

******ebook converter DEMO Watermarks*******

Q25. Which city in India is called the Garden city? Choose an option: (1). Chandigarh (2). Bengaluru (3). Srinagar (4). Thiruvananthapuram Correct option : 2 Bangolre was known as the Garden City due to two beautiful gardens namely, Lalbhagh Gardens and Cubbon Park. Development comes with a price and the greenery of Bangalore has, in recent times, given way to make room for a concrete jungle, re-naming it ‘The Silicon Valley of India’.

******ebook converter DEMO Watermarks*******

Q26. Koyali is: Choose an option: (1). Hydroelectric plant in Assam (2). Thermal Electric plant in Maharashtra (3). Oil Refinery in Gujarat (4). Biogas plant in Andhra Pradesh Correct option : 3

Gujarat Refinery (Located at Koyali in Gujarat) The Gujarat Refinery is an oil refinery located at Koyali (Near Vadodara) in Gujarat, Western India. It is the Second largest refinery owned by Indian Oil Corporation after Panipat Refinery. The refinery is currently under projected expansion to 18 MMTPA. MMTPA million metric tonnes per annum

******ebook converter DEMO Watermarks*******

Q27. Which group of rivers originates from the Himachal mountains? Choose an option: (1). Beas, Ravi and Chenab (2). Ravi, Chenab and Jhelum (3). Satluj, Beas and Ravi (4). Satluj, Ravi and Jhelum Correct option : 1 Beas, Ravi and Chenab originates from Himahcal Pradesh. Satluj originates from China. Jhelum originates from Jammu and Kashmir.

******ebook converter DEMO Watermarks*******

Q28. The Public Accounts Committee presents its report to: Choose an option: (1). The President of India (2). The Finance Minister (3). The Parliamentary Affairs Minister (4). The Parliament Correct option : 4

******ebook converter DEMO Watermarks*******

Public Accounts Committe The Public Accounts Committee (PAC) is a committee of selected members of Parliament, constituted by the Parliament of India, for the auditing of the revenue and the expenditure of the Government of India. The PAC is formed every year with a strength of not more than 22 members of which 15 are from Lok Sabha, the lower house of the Parliament, and 7 from Rajya Sabha, the upper house of the Parliament. The term of office of the members is one year.

******ebook converter DEMO Watermarks*******

Q29. The Governor has the power to impose Governor’s rule in the State of: Choose an option: (1). Tripura (2). Nagaland (3). Mizoram (4). Jammu & Kashmir Correct option : 4 In the state of Jammu and Kashmir, failure of constitutional machinery results in Governor’s rule, imposed by invoking Section 92 of the Constitution of Jammu and Kashmir. The proclamation is issued by the state’s Governor after obtaining the consent of the President of India. If it is not possible to revoke Governor’s rule within six months of imposition then the President’s Rule under Article 356 of the Indian Constitution is imposed.

******ebook converter DEMO Watermarks*******

Q30. Which Indian city has the highest population according to 2011 Census? Choose an option: (1). Kolkata (2). New Delhi (3). Chennai (4). Mumbai Correct option : 4 According to 2011 census, top 5 cities by population are 1. Mumbai has around 12.44 million people 2. Delhi has 11 million people 3. Bangalore has 8.44 million people 4. Hyderabad has 6.7 million people 5. Ahmadabad has 5.5 million people

******ebook converter DEMO Watermarks*******

Q31. ‘Nirmal Bharat Abhiyan Yojna’ is associated with: Choose an option: (1). The Development of Bihar (2). Community toilets in slum areas (3). Construction of Houses for low income groups (4). None of the above Correct option : 2

Nirmal Bharat Abhiyan Nirmal Bharat Abhiyan, a sanitation program started by Govt of India in 1999 with a motive to create awareness about the importance of sanitation particularly among rural in Rural India.

******ebook converter DEMO Watermarks*******

Q32. Fly ash is environment pollutant produced by: Choose an option: (1). Thermal power plant (2). Oil Refinery (3). Fertiliser Plant (4). Strip mining Correct option : 1

Fly ash Fly ash is a fine powder which is a byproduct from burning pulverized coal in electric generation power plants Fly ash is a pozzolan, a substance containing aluminous and siliceous material that forms cement in the presence of water. When mixed with lime and water it forms a compound similar to Portland cement.

******ebook converter DEMO Watermarks*******

Q33. What is the capital of Republic of South Sudan? Choose an option: (1). Khartoum (2). Juba (3). Omdurman (4). Bengui Correct option : 2

Juba Juba is the capital and largest city of the Republic of South Sudan. It also serves as the capital of Jubek State, one of the 28 states of South Sudan. The city is situated on the White Nile and functions as the seat and metropolis of Juba County.

******ebook converter DEMO Watermarks*******

Q34. What is the name of first Indian Research Station in the Arctic? Choose an option: (1). Himadri (2). Maitri (3). Bharati (4). Dakshin Gangotri Correct option : 1

Himadri Station Himadri Station is India’s first Arctic research station located at Spitsbergen, Svalbard, Norway. It is located at the International Arctic Research base, Ny-Ålesund. It was inaugurated on the 1st of July, 2008 by the Minister of Earth Sciences. It was set up during India’s second Arctic expedition in June 2008. It is located at a distance of 1,200 kilometres (750 mi) from the North Pole.

******ebook converter DEMO Watermarks*******

Q35. Who has been given the 2011 Sahitya Academy Award? Choose an option: (1). Namavar Singh (2). Doodhnath Singh (3). Kashinath Singh (4). Rajendra Yadav Correct option : 3

******ebook converter DEMO Watermarks*******

Sahitya Academy Award Sahitya Akademi Award is given each year, since 1955, by Sahitya Akademi (India’s National Academy of Letters), to writers and their works, for their outstanding contribution to the upliftment of Indian literature and Hindi literature in particular. No Award was conferred in 1962. in 2011, Kashinath Singh won the Sahitya Academy Award for his work "Rehan Par Ragghu"

******ebook converter DEMO Watermarks*******

Q36. With which activity is Magsaysay awardees Rajendra Singh associated? Choose an option: (1). Human Rights (2). Child Welfare (3). Environmental issues (4). Water Conservation Correct option : 4

******ebook converter DEMO Watermarks*******

Dr. Rajendra Singh Born on 9 August 1959, is a well-known water conservationist from Alwar district, Rajasthan in India. Also known as "waterman of India", he won the Stockholm Water Prize, an award known as "the Nobel Prize for water", in 2015. He had also won Ramon Magsaysay Award for community leadership in 2001 for his pioneering work in community-based efforts in water harvesting and water management.

******ebook converter DEMO Watermarks*******

Q37. After Subhash Chandra Bose resigned from the President ship of Congress in 1939 who became the party president? Choose an option: (1). Pattabhi Sitaramayya (2). Maulana Azad (3). Rajendra Prasad (4). Jaya Prakash Narayan Correct option : 3 Rajendra Prasad replaced Bose after the session on Subash Chandra Bose’s resignation.

******ebook converter DEMO Watermarks*******

Q38. Who were the two prominent leaders of Home Rule League? Choose an option: (1). Tilak and Annie Besant (2). Tilak and Feroz Shah Mehta (3). Annie Besant and B.C. Pal (4). B.C. Pal and Tilak Correct option : 1

Home Rule Movement (1916 - 1918) The Indian Home Rule movement was a movement in British India on the lines of Irish Home Rule movement and other home rule movements. The movement lasted around two years between 1916–1918 and is believed to have set the stage for the independence movement under the leadership of Mahatma Gandhi.

Major Leaders 1. Annie Besant (who was of Irish origin and a firm supporter of Irish home rule movement) 2. Bal Gangadhar Tilak 3. Sir S. Subramania Iyer

******ebook converter DEMO Watermarks*******

Q39. Environment planning organization is: Choose an option: (1). CECRI (2). NEERI (3). ICAR (4). CEPHERI Correct option : 2

******ebook converter DEMO Watermarks*******

NEERI The National Environmental Engineering Research Institute (NEERI) is a research institute created and funded by Government of India. It was established in Nagpur in 1958 with focus on water supply, sewage disposal, communicable diseases and to some extent on industrial pollution and occupational diseases found common in postindependent India. NEERI is a pioneer laboratory in the field of environmental science and engineering and part of Council of Scientific and Industrial Research (CSIR). NEERI falls under the Ministry of Science and Technology of central Government.

******ebook converter DEMO Watermarks*******

Q40. Shallow lake with rich organic products are called: Choose an option: (1). Heterotrophic (2). Eutrophic (3). Mesotrophic (4). Oligotrophic Correct option : 2

Eutrophic A Eutrophic lake is characterized by an abundant accumulation of nutrients that support a dense growth of algae and other organisms, the decay of which depletes the shallow waters of oxygen in summer. In simple words, a lake which is rich in nutrients but low in oxygen.

******ebook converter DEMO Watermarks*******

Q41. Which of the following plans is meant for constructing houses for rural people? Choose an option: (1). Indira Awas Yojna (2). Ambedkar Awas Yojna (3). PURA (4). None of the above Correct option : 1 Pradhan Mantri Gramin Awaas Yojana (PMGAY), previously Indira Awaas Yojana (IAY), is a social welfare flagship programme, created by the Indian Government, to provide housing for the rural poor in India. Indira Awaas Yojana was launched by Rajiv Gandhi, the then Prime Minister of India, as one of the major flagship programs of the Ministry of Rural Development to construct houses for BPL population in the villages. A similar scheme for urban poor was launched in 2015 as Housing for All by 2022.

******ebook converter DEMO Watermarks*******

Q42. In 1943, Muslim League ministers were installed in four provinces. Two of them were Sind and NWFP. Which were the other two? Choose an option: (1). Punjab and Bengal (2). Assam and Punab (3). Bengal and Assam (4). Bengal and U.P. Correct option : 3 In 1943, Muslim League ministries had been installed in Assam (August 1942), Sind (October 1942), Bengal (March 1943) and N.W.F.P. (May 1943).

******ebook converter DEMO Watermarks*******

Q43. At which venue did the 1907 A.D. split in Congress Party take place? Choose an option: (1). Benaras (2). Calcutta (3). Nagpur (4). Surat Correct option : 4 At the Surat Split 1907, congress leaders split into two groups – moderates and extremists. The Rift between these two sections became apparent at the Banaras Session (1905) when some nationalist-led by Tilak denounced the method of the moderates and suggested passive resistance. They also advocated the boycott of British goods and government institutions. At the end of this session, Lokmanya Tilak, and his followers held a separate conference and announced the formation of the Extremist Party. However, they decided to work as a part of the Indian National Congress.

******ebook converter DEMO Watermarks*******

Q44. Tarapore Committee was associated with which one of the following? Choose an option: (1). Special Economic Zones (2). Full Capital Account convertibility (3). Foreign exchange reserves (4). Effect of oil prices on the Indian Economy Correct option : 2 The Committee on Capital Account Convertibility (CAC) or Tarapore Committee was constituted by the Reserve Bank of India for suggesting a roadmap on full convertibility of Rupee on Capital Account. The committee submitted its report in May 1997. The Tarapore committee observed that the Capital controls can be useful in insulating the economy of the country from the volatile capital flows during the transitional periods and also in providing time to the authorities, so that they can pursue discretionary domestic policies to strengthen the initial conditions.

******ebook converter DEMO Watermarks*******

Q45. The Western Ghats and the Eastern Ghats meet at: Choose an option: (1). Anaimalai Hills (2). Anaimudi Hills (3). Nilgiri Hills (4). Palni Hills Correct option : 3 The meeting point of the western and Eastern Ghats in the Nilgiri Hills has the highest point at Doda Betta Peak.

******ebook converter DEMO Watermarks*******

Q46. Rusting of iron is: Choose an option: (1). An electrochemical process (2). A photochemical process (3). An electroanalytical process (4). All of the above Correct option : 1 Rusting of any metal(corrosion) takes place on the basis of electrochemical reactions(oxidation and reduction) on the surface of metal. Usually corrosion takes place in presence of moisture and it is called wet corrosion.Environment of the metal place very important role in corrosion, especially oxygen, water vapour, acidic and basic medium.

******ebook converter DEMO Watermarks*******

Q47. The urine sample of a diabetic patient contains Choose an option: (1). Sucrose (2). Fructose (3). Glucose (4). All of the above Correct option : 3 Diabetes is a disease that occurs when your blood glucose, also called blood sugar, is too high. The urine sample of a diabetic patient contains Glucose.

******ebook converter DEMO Watermarks*******

Q48. The quality of diesel oil for use in diesel engines is determined by: Choose an option: (1). The composition of the oil (2). The octane number (3). Cetane number (4). The amount of additive added Correct option : 3 Cetane is a colorless, liquid hydrocarbon (a molecule from the alkane series) that ignites easily under compression. For this reason, it was given a base rating of 100 and is used as the standard measure of the performance of compression ignition fuels, such as diesel fuel and biodiesel.

******ebook converter DEMO Watermarks*******

Q49. Which of the following is used as moderators in nuclear reactors? Choose an option: (1). Graphite (2). Paraffin (3). Heavy water (4). All of the above Correct option : 4 The moderator of a nuclear reactor is a substance that slows neutrons down. In traditional nuclear reactors, water is used as moderato. When fast neutrons strike the hydrogen atoms in H2O, they slow down a lot. There are other good moderators like graphite, Paraffin, beryllium, and more.

******ebook converter DEMO Watermarks*******

Q50. The 40 years war between ancient Aryan king Divodas and Shamber resulted in the defeat of: Choose an option: (1). Sakas (2). Khasas (3). Kirats (4). Kinnars Correct option : 3 The Kirata’s powerful king was "Shambar", who fought against the Aryan King ‘Divodas’. Rishi Bharadawaj was the advisor of the Aryan King’ Divodas’ . The war between the two lasted for 40 years and has been discussed in the Rig Veda by Rishi Vashishta and Vamdeva. Shambar and his ally, Verchi were killed at a place named ‘Udubraj’. Divodas was succeeded by his son ‘Sudas’ and Bharadwaj by Rishi Vashishta.

******ebook converter DEMO Watermarks*******

Q51. Who built the Lakshmi Narayan Temple at Chamba? Choose an option: (1). Meru Varman (2). Sahil Varman (3). Lakshmi Varman (4). Lalit Varman Correct option : 2 The Lakshmi Narayan temple complex, situated in Chamba district, was built in the 10th century by Raja Sahil Verman. It has been built to suit the local climatic conditions with wooden chatries and has a shikara, and a sanctum sanctorum (Garbhagriha), with an antarala and a mantapa.

******ebook converter DEMO Watermarks*******

Q52. Which of the following is not a natural polymer? Choose an option: (1). Wool (2). Silk (3). Cotton (4). Teflon Correct option : 4 Teflon is a synthetic polymer (fluoropolymer of tetrafluoroethylene), while Wool, Silk and Cotton are natural polymers.

******ebook converter DEMO Watermarks*******

Q53. When mild steel is heated to a high temprature and suddenly cooled in water, it becomes hard and brittle. The process is called: Choose an option: (1). Hardening (2). Annealing (3). Quenching (4). Tempering Correct option : 3 In materials science, quenching is the rapid cooling of a workpiece to obtain certain material properties. This method is commonly used to harden steel.

******ebook converter DEMO Watermarks*******

Q54. Which Raja’s wooing of Nokhu Gaddan is depicted in the "RajaGaddan" love lyric? Choose an option: (1). Man Singh of Jaswan (2). Sri Singh of Chamba (3). Bir Singh of Nurpur (4). Sansar Chand of Kangra Correct option : 4 Sansar Chand was romantically inclined, and apart from his three queens from Suket, Sirmaur and Bara Banghal, he also married Nokhu, a Gaddan beauty. The romance of Sansar Chand and Nokhu, the Gaddan beauty, later on ennobled and named Gulab Dassi, is celebrated in Kangra folk songs.

******ebook converter DEMO Watermarks*******

Q55. For distribution of essential commodities all families in Himachal Pradesh have been divided into four categories. Two of them are: Below Poverty Line (BPL), and Above Poverty Line (APL) families. Which are the other two are? Choose an option: (1). Antodaya and Scheduled Caste families (2). Scheduled Caste and Scheduled Tribe families (3). Annapurna and Scheduled Caste families (4). Annapurna and Antodaya families Correct option : 4 One of the main constituents of the Govt. strategy for poverty alleviation is Targeted Public Distribution System (T.P.D.S.) which ensures availability of essential commodities like Wheat, Rice, Levy Sugar and Kerosene through a net work of 4,131 Fair Price Shops for distribution of the essential commodities. The total families have been divided in four categories viz. (i) Above Poverty Line (APL), (ii) Below Poverty Line (BPL), (iii) Antyodaya (Poorest) and (iv) Annapurna (Indigient).

******ebook converter DEMO Watermarks*******

Q56. Dr. Piyush Guleri’s award winning work, "Chhaunte" is a ……………………. Choose an option: (1). Novel (2). Natak(Drama) (3). Kavita (Poem) (4). Essay Correct option : 3 Dr. Piyush Guleri’s award winning work, "Chhaunte" is a Poem.

******ebook converter DEMO Watermarks*******

Q57. According to 2011 census which district of Himachal Pradesh has the lowest female literacy rate? Choose an option: (1). Kullu (2). Kinnaur (3). Chamba (4). Lahul Spiti Correct option : 3 According to 2011 census Chamba district of Himachal Pradesh has the lowest female literacy rate.

******ebook converter DEMO Watermarks*******

Q58. On which issue did several Panchayats of Nichar Sub-division boycotted the December 2010-January 2011 Panchayat election? Choose an option: (1). Opposition of cement factory (2). Demand for better rooad connectivity (3). Demand for better marketing facilities for apple and other fruits (4). Opposition to Karcham Wangtoo Hydel Project Correct option : 4 Several Panchayats of Nichar Sub-division boycotted the December 2010-January 2011 Panchayat election to oppose the Karcham Wangtoo Hydel Project. They are saying the 1,000 MW Karcham Wangtoo Hydroelectric Project has not only compromised with their traditional water and forest rights but also affected their livelihood.

******ebook converter DEMO Watermarks*******

Q59. Almost 90% of the world’s annual fish supply comes form: Choose an option: (1). Inland waters (2). The coast of Peru (3). The Continental shelf (4). The open seas Correct option : 2

******ebook converter DEMO Watermarks*******

Q60. A polymer which is used for making ropes and carpet fibres is: Choose an option: (1). Polyacetylene (2). Polypropylene (3). Polyacrylonitrile (4). PVC Correct option : 2 Polypropylene (PP), also known as polypropene, is a thermoplastic polymer used in a wide variety of applications including packaging and labeling, textiles (e.g., ropes, thermal underwear and carpets), stationery, plastic parts and reusable containers of various types, laboratory equipment, loudspeakers, automotive components.

******ebook converter DEMO Watermarks*******

Q61. The bleaching action of chlorine occurs in the presence of: Choose an option: (1). Moisture (2). Sunlight (3). Pure oxygen (4). Pure sulphur dioxide Correct option : 1 Chlorine bleaches by the process of oxidation. It needs moisture for its bleaching action. Chlorine reacts with water to form hydrochloric and hypochlorous acids. Hypochlorous acid is unstable, and it easily dissociates to form nascent oxygen.

******ebook converter DEMO Watermarks*******

Q62. The maximum limit of Foreign Direct Investment (FDI) in public sector banking is Choose an option: (1). 51% (2). 50% (3). 33% (4). 49% Correct option : 4 maximum limit of FDI in public sector banking is 49%

******ebook converter DEMO Watermarks*******

Q63. The Pennines (Europe), Appalachians (America) and the Aravallis (India) are examples of: Choose an option: (1). Old Mountains (2). Young Mountains (3). Fold Mountains (4). Block Mountains Correct option : 1 The major difference between young and old mountains is the level of erosion within an entire mountain range. Pennines, Appalachians and Aravallis are example of Old Mountains, while Himalayas are relatively young and still growing.

******ebook converter DEMO Watermarks*******

Q64. Three important rivers of the Indian Subcontinent have their sources near Mansarovar Lake in the Great Himalayas. These rivers are: Choose an option: (1). Indus, Jhelum and Satluj (2). Brahmaputra, Satluj and Yamuna (3). Brahmaputra, Indus and Satluj (4). Jhelum, Satluj and Yamuna Correct option : 3 Brahmaputra, Indus and Satluj have their origin near to Mansarovar Lake in the Great Himalayas.

******ebook converter DEMO Watermarks*******

Q65. Which one of the following is no longer a source of Income of urban local bodies in Punjab, Haryana and Himachal Pradesh? Choose an option: (1). House Tax (2). Toll Tax (3). Fee (4). Octroi Correct option : 4 Octroi has been abolished in Punjab, Haryana and Himachal Pradesh

******ebook converter DEMO Watermarks*******

Q66. As per 13th Finance Commission Recommendations during 2010-15, transfers to the states in the form of ‘grant-in-aid’ are expected to be: Choose an option: (1). Rs. 44,000 crore (2). Rs. 1,64,832 crore (3). Rs. 3,18,581 crore (4). Rs. 1,07,552 crore Correct option : 3

******ebook converter DEMO Watermarks*******

Q67. The Production of most metallic minerals is affected by each of the following, except: Choose an option: (1). Distance to the market (2). Quantity available (3). Richness of the ore (4). Weight of the ore Correct option : 4

******ebook converter DEMO Watermarks*******

2010 HPAS Solved Paper This PDF Booklet contains 2010 HPAS Solved Paper. Explanation is provided by Shunya Foundation Team. 2010 Solved Paper in English for General Studies Paper 1

******ebook converter DEMO Watermarks*******

Q1. The definition of Biosphere is: Choose an option: (1). The earth and its atmosphere which is inhabited by living organisms (2). All living organisms on earth (3). All the plants on earth (4). All the animals on earth Correct option : 1

Fig. The biosphere is made up of the parts of Earth where life exists. The biosphere surrounds the Earth along with the lithosphere (rock), hydrosphere (water) and atmosphere (air) . In fact, it is the place on Earth’s surface where life dwells. Hence, the earth and its atmosphere which is inhabited by living organisms is called as Biosphere.

******ebook converter DEMO Watermarks*******

Q2. Rhododendron is the characteristic vegetation of: Choose an option: (1). Tropical zone (2). Alpine zone (3). Mangrove belt (4). Gangetic plane Correct option : 2

_Fig. Rhododendron Rhododendron is a keystone species of sub-alpine and alpine regions of the Himalaya of Northern India. These species have considerable ecological and economic importance. In western Himalaya, the wood of the species is used for fuel, flowers used to made into herbal drink. The drink is considered medicinal for heart diseases, bark used for digestive disorders. Due to anthropogenic interference associated with habitat destruction, climate change and over-exploitation have collectively put pressure on its natural population.

******ebook converter DEMO Watermarks*******

******ebook converter DEMO Watermarks*******

Q3. Some reliable indicators of pollutions are: Choose an option: (1). Ferns and Cycas (2). Algae and liverworts (3). Lichens and mosses (4). Neem tree and Eichhornia Correct option : 3

Lichens

Mosses Lichens are composite organisms in which a single species of fungus lives symbiotically with one or more algal species. Mosses are non-vascular plants. They produce spores for reproduction instead of seeds and don’t grow flowers, wood or true roots. Since mosses and lichens lack roots, surface absorption of rainfall is the only means of obtaining vital nutrients which are dissolved in rainwater. Pollution content accumulates within the lichens and mosses reach a level where it breaks down the chlorophyll molecules which are responsible for photosynthesis in the algae. Hence, the algae die and this leads to the death of the fungus. Mosses are reliable indicators of air pollution risks to ecosystems. Hence, Lichens and mosses as reliable indicators ******ebook converter DEMO Watermarks*******

of air pollution and heavy metals pollution.

******ebook converter DEMO Watermarks*******

Q4. Gas released during Bhopal gas tragedy was: Choose an option: (1). Sodium isocynate (2). Ethyl isocynate (3). Potassium isocynate (4). Methyl Isocynate Correct option : 4 The Bhopal disaster or Bhopal gas tragedy was an industrial accident. It happened at a Union Carbide subsidiary pesticide plant in the city of Bhopal, India. On 3 December 1984, the plant released 42 tonnes of toxic methyl isocyanate (MIC) gas, exposing more than 500,000 people to toxic gases.

******ebook converter DEMO Watermarks*******

Q5. Human eye is sensitive to the wavelength in the range of: Choose an option: (1). 300-390 nm (2). 300-360 nm (3). 380-760 nm (4). 790-800 nm Correct option : 3

Fig. Human Eye The human eye is sensible to light wave which wavelength is roughly between 400 nm (violet) and 700 nm (red). That range reflects the wavelengths of sunlight which reach the Earth’s surface with sufficient intensity to excite the cells on our retinae. Wavelengths shorter than 400 nm (ultraviolet, UV) or longer than 700 nm (infrared, IR) are not visible.

******ebook converter DEMO Watermarks*******

Q6. Who built the old fort (Purana Qila) in Delhi? Choose an option: (1). Sher Shah (2). Jahangir (3). Shah Jahan (4). Akbar Correct option : 1 Purana Qila (Old Fort) is one of the oldest forts in Delhi. Its current form was built by Sher Shah Suri, the founder of the Sur Empire during 1500-1545.

******ebook converter DEMO Watermarks*******

Q7. Under the Constitution of India who is the guardian of fundamental rights? Choose an option: (1). Parliament (2). President (3). Judiciary (4). Cabinet Correct option : 3 There are three Pillars of Democracy 1. Legislature: One who enacts the law. 2. Executive: One who implements the law. 3. Judiciary: One who upholds the law. Judiciary is responsible for guarding the fundamental rights of Indian Citizens.

******ebook converter DEMO Watermarks*******

Q8. MNREGA programme is related to: Choose an option: (1). Rural employment (2). Urban employment (3). Education for all (4). Public health Correct option : 1 MNREGA stands for "Mahatma Gandhi National Rural Employment Guarantee Act" Key Points 1. Starting from 200 districts on 2 February.. 2006, the NREGA covered all the districts of India from 1 April 2008 2. It provided minimum 100 days of wage employment to unskilled manual work every financial year. 3. It is applicable to Rural Areas.

******ebook converter DEMO Watermarks*******

Q9. Which one among the following countries is not a member of SAARC? Choose an option: (1). Afghanistan (2). Bhutan (3). Maldives (4). Myanmar Correct option : 4 SAARC stands for South Asian Association for Regional Cooperation Countries included in SAARC are 1. Afghanistan 2. Bangladesh 3. Bhutan 4. India 5. Nepal 6. Maldives 7. Pakistan 8. Sri Lanka SAARC comprises 3% of the world’s area, 21% of the world’s population and 3.8% (US$ 2.9 trillion) of the global economy, as of 2015.

******ebook converter DEMO Watermarks*******

Q10. What is the roughly ratio between the population density of Himachal Pradesh and India as per the provisional figures of 2011 Census of India? Choose an option: (1). 1:2 (2). 1:3 (3). 1:4 (4). 1:5 Correct option : 2 As per Niti Aayog, Population Density of India as of 2011 is 382, while that of Himachal is 123, so national population density if 3 times more than that of Himachal Pradesh.

******ebook converter DEMO Watermarks*******

Q11. Asbestos is no longer preferred for commercial use because: Choose an option: (1). It causes lung cancer (2). Its structures are very weak (3). It is not sound proof (4). It catches fire Correct option : 1 When Asbestos is inhaled, it does get deposited in lung tissue or in the linings of lungs and can cause cancer in both locations. Asbestos lodged in lung tissue cause lung cancer and in pleural lining causes pleural mesothelioma.

******ebook converter DEMO Watermarks*******

Q12. Hemorrhagic dengue fever is spread by: Choose an option: (1). Aedes fluviatilis (2). Aedes albimenus (3). Aedes aegyptii (4). Aedes meristica Correct option : 3 Aedes aegypti is a mosquito that can spread various viral diseases like Dengue fever, chikungunya, zika virus and yellow fever. This mosquito has white marking on its legs as the identification sign. It mostly bites in early morning or late afternoon. Male mosquitos do not bite or spread disease.

******ebook converter DEMO Watermarks*******

Q13. In which district of Himachal Pradesh is Uhal Hydroelectric Project located? Choose an option: (1). Mandi (2). Chamba (3). Shimla (4). Kullu Correct option : 1

Uhal Hydroelectric Project is Located in Mandi district of Himachal Pradesh.

******ebook converter DEMO Watermarks*******

Q14. Who was the founder of Satyashodhak Samaj? Choose an option: (1). B.G. Tilak (2). Jyotiba Phule (3). Narayan Guru (4). B.R. Ambedkar Correct option : 2 Satyashodhak Samaj was a social reform society established by Jyotirao Phule in Pune, India, on 24 September 1873. Its purpose was to liberate the Shudra and Untouchable castes from exploitation and oppression.

******ebook converter DEMO Watermarks*******

Q15. Where was the Headquarters of Ghadar Party? Choose an option: (1). New York (2). Los Angles (3). San Francisco, USA (4). London Correct option : 3 On April 21,1913, the Indians of San Francisco, USA assembled and formed the Ghadar Party (Revolution Party). The aim of the Ghadar Party was to get rid of the slavery caused by the British for the people of India. The British had been ruling India as a colony of the British Empire for over 100 years.

******ebook converter DEMO Watermarks*******

Q16. Which part of the world has a high density of organisms? Choose an option: (1). Grasslands (2). Savannah (3). Deciduos forests (4). Tropical rain forests Correct option : 4 Tropical rain forests have the highest density of organisms.

******ebook converter DEMO Watermarks*******

Q17. Carbon Dioxide absorbs strongly in Infrared region and its presence in atmosphere decreases the loss of heat from Earth by radiations. This is called? Choose an option: (1). Carbon Dioxide burning (2). Carbon Dioxide recycling (3). Carbon recycling (4). Greenhouse effect Correct option : 4 Greenhouse effect is the process by which radiations from a planet’s atmosphere warms the planet’s surface to a temperature above than what it would have been without its atmosphere. Carbon Dioxide is produced by fossil fuel burning and other activities like Cement production and Tropical deforestation. Its presence in atmosphere has increased from 313 parts per million in 1960 to 400 ppm in 2013. Carbon recycling is the process of using Carbon Dioxide, water and electricity to create Methanol Fuel.

******ebook converter DEMO Watermarks*******

Q18. During whose viceroyalty was the Vernacular Press Act enacted? Choose an option: (1). Lord Curzon (2). Lord Morley (3). Lord Lytton (4). Lord Ripon Correct option : 3 Proposed by Lord Lytton, then viceroy of India (governed 1876–80), the Vernacular Press Act was intended to prevent the vernacular press from expressing criticism of British policies.

******ebook converter DEMO Watermarks*******

Q19. In which district of Himachal Pradesh is Surajtal Lake ? Choose an option: (1). Kullu (2). Lahul and Spiti (3). Chamba (4). Sirmaur Correct option : 2

Suraj Tal lies just below the Bara-lacha-la pass (4,890m) in the Lahaul and Spiti district of Himachal Pradesh. It is the third highest lake in India. Suraj Tal Lake is the source of Bhaga River which joins the Chandra River downstream at Tandi to form the Chenab River.

******ebook converter DEMO Watermarks*******

Q20. Freon causes great environmental damage. They are : Choose an option: (1). Mixed Carbon gases (2). Mixed Chlorofluoro hydro-carbons (3). Mixed sulphur gases (4). Free gases Correct option : 2 Mixed chlorofluoro hydrocarbons (CFCs) such as CFCl3, CF2Cl2 and CF3Cl are known as freons. They were introduced as refrigerants in the 1930s. They are used as propellants in pressurized aerosol cans also. In the stratosphere they release chlorine atoms under the influence of intense short-wave ultraviolet radiation.

******ebook converter DEMO Watermarks*******

Q21. What is an archipelago? Choose an option: (1). Ring-shaped coral reef (2). A type of iceberg (3). A geological age (4). A group of large number of Islands Correct option : 4 An archipelago is a group of islands closely scattered in a body of water. Usually, this body of water is the ocean, but it can also be a lake or river. Most archipelagoes are made of oceanic islands. This means the islands were formed by volcanoes erupting from the ocean floor. Volcanoes do not form all archipelagoes. Many archipelagoes are continental islands formed only after the last ice age. As glaciers retreated, sea levels rose and low-lying valleys were flooded. Coastal mountain ranges became archipelagoes just off the mainland.

******ebook converter DEMO Watermarks*******

Q22. Siul stream is a tributary of river? Choose an option: (1). Chenab (2). Yamuna (3). Ravi (4). Beas Correct option : 3 Siul stream is a tributary of Ravi river.

******ebook converter DEMO Watermarks*******

Q23. Balwant Rai Mehta Committee is related to: Choose an option: (1). Panchayati Raj System (2). Centre-State Relations (3). Rural Development (4). Electoral Reforms Correct option : 1 Balwantrai Mehta Committee was the first Committee set up in 1957 to look into the problems of democratic decentralization in independent India. In 1957, the committee recommended for the introduction of three-tier Panchayati Raj System in India. These three-tires are 1. the Gram-Panchayats at the village level or at the bottom, 2. the Panchayat Samiti at the block level or in the middle and 3. the Zilla Parishad at the district leve

******ebook converter DEMO Watermarks*******

Q24. Which term is used for imaginary lines joining the places of equal rainfall? Choose an option: (1). Isochrone (2). Isohyet (3). Isoseismal (4). Isoneph Correct option : 2 isohyet is a line drawn on a weather map connecting points that receive equal amounts of precipitation during a given period of time.

******ebook converter DEMO Watermarks*******

Q25. Freons cause great environmental damage. They are: Choose an option: (1). Mixed Carbon gases (2). Mixed chlorofluoro hydrocarbons (3). Mixed sulphur gases (4). Free gases Correct option : 2 CFC (Chlorofluorocarbon) is an organic compound that contains Carbon, Chlorine and Fluoride and produced as volatile derivatives of methane, ethane and propane. Most common is RF-12 or Freon-12. As CFCs contribute to Ozone depletion in upper atmosphere, these are slowly replaced by Hydrofluorocarbons (HFCs) e.g. R-134a.

******ebook converter DEMO Watermarks*******

Q26. What is relevant to the Golden Quadrilateral in India? Choose an option: (1). Airways (2). Railways (3). Roadways (4). National electricity grid Correct option : 3 Golden Quadrilateral is a network of highways connecting India’s four top metropolitan cities, namely Delhi, Mumbai, Chennai and Kolkata, thereby, forming a quadrilateral. The overall length of the quadrilateral is 5,846 km consisting of four/six lane express highways.

******ebook converter DEMO Watermarks*******

Q27. Which of the following is not a plant hormone? Choose an option: (1). Cytokinin (2). Auxin (3). Calcium (4). Gibberellin Correct option : 3 Calcium is a chemical element and atomic number 20. Its fifth most abundant element by mass in Earth’s crust. It’s an important constituent of bone, teeth and shells. Calcium carbonate and calcium citrate are often taken as dietary supplements and is on list of World Health Organisation’s (WHO) list of Essential Medicines. Cytokinin is a class of plant growth substance that promotes cell division or cytokinesis, in plant roots and shoots. They are involved primarily in cell growth and differentiation but also affect apical dominance. Auxin are plant hormones with some morphogen-like characteristics. Auxins have a cardinal role in coordination of many growth and behavioural processes in plant’s life cycle. Gibberellins are plant hormones that regulates growth and influence various developmental process like stem elongation, germination, dormancy, flowering, enzyme induction and leaf and fruit senescence.

******ebook converter DEMO Watermarks*******

Q28. What is the silicon valley of United States of America famous for? Choose an option: (1). Apple cultivation (2). Petrochemicals (3). Electronics (4). Tourism Correct option : 3 Silicon Valley is the southern part of the San Francisco Bay Area in California in the United States. Its is famous for electronics.

******ebook converter DEMO Watermarks*******

Q29. What is the density of population at national level in India according to 2011 census? Choose an option: (1). 326 persons per square kilometer (2). 367 persons per square kilometer (3). 382 persons per square kilometer (4). 398 persons per square kilometer Correct option : 3 As per 2011 census, India has a population density of 382 person per square kilometer. 1. Himachal Pradesh has 123 person per square km. 2. Delhi has highest of 11, 297 person per square km. 3. Chandigarh has 9252 person per square km. 4. Arunachal Pradesh has least population density of 17 person per square km.

******ebook converter DEMO Watermarks*******

Q30. Which mountain range separates Sirmaur from Shimla? Choose an option: (1). Choor Chandani (2). Chanshal (3). Hattoo (4). Shali Correct option : 1 Choor Chandni or Churdhar range of mountains separate district Shimla from district Sirmour in Himachal Pradesh.

******ebook converter DEMO Watermarks*******

Q31. Who amongst the following can be the chairman of National Human Rights Commission? Choose an option: (1). Any serving judge of the Supreme Court (2). Any serving judge of High Court (3). A retired Chief Justice of Supreme Court (4). A retired Chief Justice of High Court Correct option : 3

NHRC (National Human Rights Commission) The Rights Commission (NHRC) of India is an autonomous public body constituted on 12 October 1993 under the Protection of Human Rights Ordinance. The NHRC is the National Human Rights Commission of India, responsible for the protection and promotion of human rights, defined by the Act as "rights relating to life, liberty, equality and dignity of the individual guaranteed by the Constitution or embodied in the International Covenants".

Chairman of NHRC A Chairperson, should be retired Chief Justice of India

2016 Chairperson Justice H.L. Dattu was appointed as the chair person on 29 Feb-2016

******ebook converter DEMO Watermarks*******

Q32. Antibodies are found in: Choose an option: (1). Platelets (2). RBC (3). Lymphocytes (4). Eosinophils Correct option : 3

******ebook converter DEMO Watermarks*******

B Lymphocytes B lymphocytes produce antibodies - proteins (gamma globulins) that recognize foreign substances (antigen) and attach themselves to them. B lymphocytes (or B cells) are each programmed to make one specific antibody.

******ebook converter DEMO Watermarks*******

Q33. Night Blindness is caused by the deficiency of: Choose an option: (1). Vitamin A (2). Vitamin B6 (3). Vitamin B12 (4). Vitamin C Correct option : 1

******ebook converter DEMO Watermarks*******

Night blindness (nyctalopia) Night blindness (nyctalopia) is the inability to see well at night or in poor light. It is not a disease in itself, but rather a symptom of an underlying problem, usually a retina problem.

Causes of Night Blindless Night blindness has many causes, including: 1. Vitamin A deficiency 2. Myopia 3. Glucoma

******ebook converter DEMO Watermarks*******

Q34. India is at which state of the following stages of Demographic transition? Choose an option: (1). Population level stabilizes with number of births and deaths being low and equal (2). Birth rates and death rates are both low (3). Reduction in birth rate but the reduction in death rate is higher than reduction in birth rate (4). Reduction in morality rate while birth rate continues to be high Correct option : 3 Birth rate in India is reducing but the reduction in death rate is higher than reduction in birth rate. Birth Rate per women has decreased from 6 (births per women) in 1960 to 2.43 in 2014.

******ebook converter DEMO Watermarks*******

Q35. Consider the following related to Sarva Shiksha Abhiyan(SSA) being implemented in partnership with the States for addressing the needs of children: (1) Enrollment of all children in school (2) Addressing the need of child in the age group of 6-10 (3) Setting up Education Guarantee Centres(EGC), Alternate Schools etc. (4) Bridging of gender and social category gaps in enrollment with retention and learning Which of the above is not correct? Choose an option: (1). (1) and (3) (2). (2) alone (3). (3) and (4) (4). (3) alone Correct option : 2 Sarva Shiksha Abhiyan is an Indian Government programme aimed at the universalisation of elementary education. It is working on free and compulsory education to children between the ages of 6 to 14. The programme was pioneered by former Indian Prime Minister Atal Bihari Vajpayee.

******ebook converter DEMO Watermarks*******

Q36. Which one of the following led to the Malogaon Committee to recommend for interest rate cap for micto-finance institutions? Choose an option: (1). Promoters of micro-finance institutions growing rich (2). High interest rate leading to suicides (3). Small loans offered to micro-enterprises (4). Borrowers stopped repaying loans Correct option : 2 Aimed at reviving the crisis- ridden micro finance sector, a Reserve Bank of India Committee on Wednesday suggested that micro finance institutions (MFIs) be allowed to charge a maximum interest of 24 per cent on small loans which cannot exceed Rs.25,000. Suicides due to high interest rate led to the Malogaon Committee to recommend for interest rate cap for mictofinance institutions.

******ebook converter DEMO Watermarks*******

Q37. Consider the following in relation with Corporate tax: (1) Total turnover of the company (2) Profit after distribution of dividend (3) Profit before distribution of dividend (4) Capital employed in the company Which one of the above is the basis of corporate tax? Choose an option: (1). (1) alone (2). (2) alone (3). (3) alone (4). (4) alone Correct option : 3 A dividend is the distribution of ‘after tax profits’ to the shareholders of a limited company. Most systems that tax corporations also impose income tax on shareholders of corporations when earnings are distributed. Such distribution of earnings is generally referred to as a dividend. The tax may be at reduced rates.

******ebook converter DEMO Watermarks*******

Q38. Consider the following taxes: (1) Sales tax (2) Income tax (3) Corporate tax (4) Entertainment tax Which of the above taxes are levied by the State government and not shared by Central Government? Choose an option: (1). (1) and (2) only (2). (2) and (3) only (3). (3) and (4) only (4). (1) and (4) only Correct option : 4 Taxes Levied by the State are Sales Tax and VAT, Professional Tax, Stamp Duty, Luxury Tax, Entertainment Tax, Motor Vehicles Tax, Tax on Vehicles Entering State, Transportation of Passengers and Goods Tax, Tax on Agricultural Income, Tax on Land and Buildings, Capitation Taxes and Tax on Mineral Rights.

******ebook converter DEMO Watermarks*******

Q39. Which of the following is the highest export-oriented handicrafts (in value terms) in India exports? Choose an option: (1). Gems and Ornaments (2). Leather goods (3). Brass goods (4). Ivory goods Correct option : 1 Gems and Ornaments are the highest export oriented handicrafts.

******ebook converter DEMO Watermarks*******

Q40. Which one of the following represents at least in part investment in human capital? Choose an option: (1). Formal education (2). On the job training (3). Better health care and nutrition (4). All of the above Correct option : 4 Human capital is a collection of traits – all the knowledge, talents, skills, abilities, experience, intelligence, training, judgment, and wisdom possessed individually and collectively by individuals in a population. These resources are the total capacity of the people that represents a form of wealth which can be directed to accomplish the goals of the nation or state or a portion thereof. The concept of human capital recognizes that not all labor is equal and that the quality of employees can be improved by investing in them;

******ebook converter DEMO Watermarks*******

Q41. Which one of the following treaties was not covered by Uruguay Round Negotiations, which established WTO? Choose an option: (1). General agreement on trade in Services (2). Trade related intellectual property rights (3). Trade related investment measures (4). General agreement on Environment Protction Correct option : 4 The Uruguay Round was the 8th round of multilateral trade negotiations (MTN) conducted within the framework of the General Agreement on Tariffs and Trade (GATT), spanning from 1986 to 1994 and embracing 123 countries as "contracting parties". The Round led to the creation of the World Trade Organization, with GATT remaining as an integral part of the WTO agreements. Uruguay Round Negotiations covered following treaties: 1. The Agreement Establishing the WTO 2. The Multilateral Agreements on Trade in Goods including the GATT 1994 and the Trade Related Investment Measures (TRIMS) 3. General Agreement on Trade in Services (GATS)) 4. Agreement on Trade-Related Aspects of Intellectual Property Rights (TRIPS) 5. Dispute settlement (DSU) 6. Reviews of governments' trade policies (TPRM)

******ebook converter DEMO Watermarks*******

Q42. Who founded the Guler state? Choose an option: (1). Karam Chand (2). Hari Chand (3). Rup Chand (4). Prithvi Chand Correct option : 2 Guler was a small state in the Lower Himalayas. Its capital was the town of Haripur Guler. The kingdom was founded in 1415 by Raja Hari Chand.

******ebook converter DEMO Watermarks*******

Q43. On the basin of which river is Dodra Kawar situated? Choose an option: (1). Pabbar (2). Tons (3). Yamuna (4). Giri Correct option : 2 Dodra and Kwar are two villages on the opposite sides of the valley separated by Rupin river that merges into Tons river. It borders with Uttarakhand.

******ebook converter DEMO Watermarks*******

Q44. In which year was ‘Swarna Jayanti Gram Swarozgar Yojna’ launched in Himachal Pradesh? Choose an option: (1). 1994-1995 (2). 1999-2000 (3). 2004-2005 (4). 2007-2008 Correct option : 2 Swarna Jayanti Gram Swarozgar Yojna was launched in Himachal Pradesh fom the year 1999-2000. This Yojana is a holistic package covering all aspect of self-employment such as organisation of poor into Self Help Groups, Training, Credit, Technology, Infrastructure and Marketing. The beneficiaries under this scheme are called "Swarozgaris. The objective of SGSY is to bring the assisted poor families above the poverty line, by providing them income-generating assets.

******ebook converter DEMO Watermarks*******

Q45. 'Mukhya Mantri Bal Udhar Yojna’ of Himachal Pradesh Government is restricted to the children belonging to: Choose an option: (1). Scheduled caste families (2). Scheduled tribe families (3). BPL families (4). Scheduled caste and Scheduled tribe families Correct option : 3 This scheme is for children in need of care and protection. Under the scheme destitute children are provided free housing, food, clothing, health, and educational facilities through Bal Balika Ashrams.

******ebook converter DEMO Watermarks*******

Q46. What is the architectural style of Jawalamukhi temple in Kangra District? Choose an option: (1). Flat - roofed style (2). Pagoda Style (3). Pent-roofed style (4). Domed style Correct option : 4 This style is a new addition to the old architecture of temples in the hills. In this style domes are rounded, elongated or flat. Jwalamukhi and Brajeshwari Devi in Kangra district, Chintpurni in Una district and Naina Devi in Bilaspur district stand in this category.

******ebook converter DEMO Watermarks*******

Q47. Male literacy in the recent Census 2011 has increased to: Choose an option: (1). 77.14% (2). 79.14% (3). 81.14% (4). 83.14% Correct option : 1 As per 2011 Census, male literacy has increased to 77.14%.

******ebook converter DEMO Watermarks*******

Q48. Indian agriculture has the following features: (1) Dependence of agriculture on the monsoons (2) Abolition of Zamindari (3) Absence of trade unions in the agriculture sector (4) Heavy pressure of population on Land Which of the above is responsible for disguised unemployment in the agriculture? Choose an option: (1). (1) alone (2). (1) and (2) only (3). (1), (2), (3) only (4). (4) only Correct option : 4

Fig. Farmer in the field The Disguised Unemployment refers to the work area where surplus manpower is employed out of which some individuals have zero or almost zero marginal productivity such that if they are removed the total level of output remains unchanged. Disguised unemployment exists frequently in developing countries whose large populations create a surplus in the labor force. Since, India population increasing rapidly, therefore, the land is not sufficient for the growing population. As a result, there is heavy pressure on the land. In rural areas, most of the people depend directly on land for their livelihood. Land is very limited in comparison to population. It creates the unemployment situation for a large number of persons who depend on agriculture in rural areas.

******ebook converter DEMO Watermarks*******

Q49. Which one of the following is non-toxic? Choose an option: (1). Cadmium (2). Mercury (3). Lead (4). Sodium Correct option : 4 Cadmium, Mercury and Lead are extremely toxic, heavy metals. Sodium (Na) is non-toxic and human body requires a minimum level of sodium in their body for its proper functioning. We consume sodium in the form of Salt.

******ebook converter DEMO Watermarks*******

Q50. Carbohydrates are not used in: Choose an option: (1). Food industry (2). Textile industry (3). Cosmetic industry (4). Paint industry Correct option : 4 Carbohydrate are extensively used in food Industry, textile industry and cosmetic industry.

******ebook converter DEMO Watermarks*******

Q51. Nanotechnology is where particles of the following size are used: Choose an option: (1). 10 -9 m (2). 10 9 m (3). 10 -9 cm (4). 10 9 m Correct option : 1 Nanotechnology starts at 10-9m (usually 1-100 nanometer) Nanotechnology is the branch of technology that deals with dimensions and tolerances of less than 100 nanometres, especially the manipulation of individual atoms and molecules.

******ebook converter DEMO Watermarks*******

Q52. Carbon dioxide absorbs the infrared radiations and its presence in the atmosphere decreases the loss of heat from earth by radiations. This is called : Choose an option: (1). Carbon dioxide burning (2). Carbon dioxide recycling (3). Carbon recycling (4). Greenhouse Effect Correct option : 4 The greenhouse effect increases the temperature of the Earth by trapping heat in our atmosphere. This keeps the temperature of the Earth higher than it would be if direct heating by the Sun was the only source of warming. When sunlight reaches the surface of the Earth, some of it is absorbed which warms the ground and some bounces back to space as heat. Greenhouse gases like Carbon dioxide that are in the atmosphere absorb and then redirect some of this heat back towards the Earth. The greenhouse effect is a major factor in keeping the Earth warm because it keeps some of the planet’s heat that would otherwise escape from the atmosphere out to space. In fact, without the greenhouse effect the Earth’s average global temperature would be much colder and life on Earth as we know it would not be possible.

******ebook converter DEMO Watermarks*******

Q53. Which trench is located in the Pacific Ocean? Choose an option: (1). Mariana (2). Romanche (3). Nicobar (4). Sunda Correct option : 1 Mariana Trench is the deepest part of the world’s oceans. It is located in the western Pacific Ocean.

******ebook converter DEMO Watermarks*******

Q54. In which Indian State is the Tawang Buddhist Monastery located? Choose an option: (1). Sikkim (2). Arunachal Pradesh (3). Himachal Pradesh (4). Bihar Correct option : 2 Tawang Monastery is located in Arunachal Pradesh. It is the largest monastery in India and the second largest in Asia. It was founded by Merak Lama Lodre Gyamtso in the year 1860-61 AD.

******ebook converter DEMO Watermarks*******

Q55. For which of the following functions was V.K. Shunglu Committee constituted? Choose an option: (1). To suggest measure to control inflation (2). To recommend administrative reforms (3). To draft lokpal bill (4). To probe financial irregularities in Commonwealth Games Correct option : 4 The Prime Minister-appointed Shunglu Committee probe the "weaknesses" in management, alleged misappropriation, irregularities and wrongdoing in the conduct of Commonwealth Games.

******ebook converter DEMO Watermarks*******

Q56. Who among the following was the speaker of first Lok Sabha? Choose an option: (1). Sardar Gurmukh Nihal Singh (2). G.V. Malvankar (3). A.S. Ayangar (4). Balram Jakhar Correct option : 2 Shri G.V. Mavalankar was the first Speaker of Lok Sabha (15 May 1952- 27 February 1956) and Shri M. Ananthasayanam Ayyangar was the first Deputy Speaker of Lok Sabha (30 May 1952-7 March 1956).

******ebook converter DEMO Watermarks*******

Q57. Which ancient Indian king wrote the three dramas, i.e. Ratnavali, Priyadarshika and Nagananda? Choose an option: (1). Ashoka the Great (2). Chandragupta Maurya (3). Harsha Vardhana (4). Samudragupta Correct option : 3 Harsha (c. 590–647 CE), also known as Harshavardhana, was an Indian emperor who ruled North India from 606 to 647 CE. He was a member of the Pushyabhuti dynasty. Harsha is widely believed to be the author of three Sanskrit plays Ratnavali, Nagananda and Priyadarsika.

******ebook converter DEMO Watermarks*******

Q58. By which act was system of ‘Dyarchy’ introduced during British rule in India? Choose an option: (1). Indian Councils Act, 1861 (2). Government of India Act, 1919 (3). Government of India Act, 1935 (4). Government of India Act, 1909 Correct option : 2 Dyarchy, also spelled diarchy, system of double government introduced by the Government of India Act (1919) for the provinces of British India. It marked the first introduction of the democratic principle into the executive branch of the British administration of India.

******ebook converter DEMO Watermarks*******

Q59. Who was the Chairman of the first Union Commission of Backward Classes of India? Choose an option: (1). B.P. Mandal (2). Kaka Kalelkar (3). B.R. Ambedkar (4). Kanshi Ram Correct option : 2 Adhering to Article 340 of the Constitution of India, the First Backward Classes Commission was set up by a presidential order on 29 January 1953 under the chairmanship of Kaka Kalelkar

******ebook converter DEMO Watermarks*******

Q60. According to 2011 Census which of the following districts of the state of the state of Haryana has the lowest sex ratio? Choose an option: (1). Ambala (2). Rewari (3). Mahendergarh (4). Jhajjar Correct option : 4 In the 2011 National Census, it was found that Jhajjar district has the lowest sex ratio in India of the 0-6 group, with just 782 girls to 1,000 boys. Two villages in Jhajjar have extremely low gender-ratios: Bahrana and Dimana have gender ratios of 378 girls to 1,000 boys and 444 girls to 1,000 boys respectively. Note In Jhajjar, parents are able to illegally learn the gender of the fetus through secret early morning ultrasounds at registered clinics and through the use of code-words, Ladoo for boy and Jalebi for girl; these families often go on to abort female fetuses

******ebook converter DEMO Watermarks*******

Q61. Which of the following countries is divided into the largest number of time zones? Choose an option: (1). China (2). Russia (3). Canada (4). United States of America Correct option : 4 A time zone is a region of the globe that observes a uniform standard time for legal, commercial, and social purposes. Time zones tend to follow the boundaries of countries and their subdivisions because it is convenient for areas in close commercial or other communication to keep the same time.

******ebook converter DEMO Watermarks*******

Q62. In which form is phosphorus predominately found in soil? Choose an option: (1). Orthophosphate (2). Phosphorus oxide (3). Phosphorus sulphide (4). Phosphoric acid Correct option : 1 Phosphorus is absorbed by plants in the orthophosphate form, generally as H2PO4- or HPO42-. Phosphorus (P) is an essential element classified as a macronutrient because of the relatively large amounts of P required by plants. Phosphorus is one of the three nutrients generally added to soils in fertilizers. One of the main roles of P in living organisms is in the transfer of energy. Adequate P availability for plants stimulates early plant growth and hastens maturity.

******ebook converter DEMO Watermarks*******

Q63. Which of the following properties is not related to LASER? Choose an option: (1). High intensity (2). High coherence (3). High monochromaticity (4). High optical activity Correct option : 4 Optical rotation or optical activity (sometimes referred to as rotary polarization) is not related to LASER. While all other mentioned properties (high intensity, high coherence & high monochromaticity) are characteristics of LASER.

******ebook converter DEMO Watermarks*******

Q64. Who amongst the following is empowered to declare emergency under the Indian Constitution? Choose an option: (1). The Vice-President (2). The President (3). The Prime Minister (4). The Home Minister Correct option : 2 President of India can declare "State of Emergency in India" under the advice of the cabinet of ministers. This is done on the basis grave threats to the nation from internal and external sources or from financial situations of crisis.

******ebook converter DEMO Watermarks*******

Q65. Under which of the following circumstances can an emergency be declared in India? (1) External aggression (2) Armed rebillion (3) Internal unrest (4) Financial crisis Choose an option: (1). (1), (2) nd (3) (2). (1), (3) nd (4) (3). (2), (3) and (4) (4). (1), (2) and (4) Correct option : 4 The Constitution stipulates three types of emergencies: (1) An emergency due to war, external aggression or armed rebellion (Article 352). This is popularly known as ‘National Emergency’. However, the Constitution employs the expression ‘proclamation of emergency’ to denote an emergency of this type. (2) An Emergency due to the failure of the constitutional machinery in the states (Article 356). This is popularly known as ‘President’s Rule’. It is also known by two other names—‘State Emergency’ or ‘constitutional Emergency’. However, the Constitution does not use the word ‘emergency’ for this situation. (3) Financial Emergency due to a threat to the financial stability or credit of India (Article 360).

******ebook converter DEMO Watermarks*******

Q66. Which of the following is not properly matched? Choose an option: (1). 42nd Constitutional Amendment - Fundamental duties (2). 52nd Constitutional Amendment - Anti -Defection law (3). 73rd Constitutional Amendment - Panchayati Raj (4). 84th Constitutional Amendment - Centre-State Relations Correct option : 4 84th Amendment to the Constitution. The 84th Amendment to the Constitution (which was numbered as the 91st Amendment Bill before it was passed in Parliament) lifted the freeze on the delimitation of constituencies, as stipulated by the 42nd Constitution amendment of 1976, and allowed delimitation within States on the basis of the 1991 Census.

******ebook converter DEMO Watermarks*******

Q67. Consider the following in the context of food inflation in India: (1) Ban on transaction in agriculture products in "Future commodity exchange market" (2) Restrictions on import of agricultural products (3) Unruly weather conditions (4) Hoarding and Trader Cartel Which of the above are the possible reasons for food Inflation? Choose an option: (1). (1) and (2) (2). (2) and (3) (3). (3) and (4) (4). (1) and (4) Correct option : 3

******ebook converter DEMO Watermarks*******

Q68. Who commanded the Mughal forces which subdued Kangra fort in 1620 A.D.? Choose an option: (1). Shah Qulikhan (2). Shaikh Farid (3). Mirza Rustam Quandhari (4). Nawab Ali Khan Correct option : 4 The Kangra Fort was built by the royal Rajput family of Kangra State (the Katoch dynasty), which traces its origins to the ancient Trigarta Kingdom, mentioned in the Mahabharata epic. Nawab Saif Ali Khan, the first Mughal Governor of Kangra had subdued Kangra fort in 1620 A.D.

******ebook converter DEMO Watermarks*******

Q69. With whose assistance is the Himachal Pradesh Government’s ‘organic farming and water harvesting project’ being implemented? Choose an option: (1). Sweden (2). Australia (3). Japan (4). Germany Correct option : 3 Japan International Cooperation Agency (JICA) is assisting Himachal Pradesh for Organic Farming and Water Harvesting projects.

******ebook converter DEMO Watermarks*******

Q70. Asbestos is no longer used for commercial use because: Choose an option: (1). It causes lung cancer (2). Its structures are very weak (3). It is not sound proof (4). It catches fire Correct option : 1 All types of asbestos fibers are known to cause serious health hazards in humans. Exposure to asbestos fibers can cause serious and fatal illnesses including lung cancer, mesothelioma, and asbestosis (a type of pneumoconiosis).

******ebook converter DEMO Watermarks*******

Q71. In which one of the following cases has the Preamble been accepted as a part of the Indian Constitution? Choose an option: (1). A.K. Gopalan case (2). Keshvanand Bharti case (3). Golaknath case (4). Minerva Mills case Correct option : 2 Preamble. Preamble is an introductory statement, stating the aims and objectives of the constitution. Accordingly, the preamble to the Indian constitution spells out the basic philosophy contained in the body of the Indian Constitution. Reading through the Preamble, one can see the purpose that it serves, namely, the declaration of (1) the source of the constitution, (2) a statement of its objectives and (3) the date of its adoption. The Preamble, in brief, explains the objectives of the Constitution in two ways: one, about the structure of the governance and the other, about the ideals to be achieved in independent India. It is because of this, the Preamble is considered to be the key of the Constitution.

******ebook converter DEMO Watermarks*******

HPPSC HPAS General Studies Solved Papers 2018 HPAS Solved Paper Q1. Oil refinery at Bhatinda is being established by: Q2. Tata Iron and Steel Company at Jamshedpur was established in: Q3. The term Operation Flood is associated with : Q4. B.R. Ambedkar was born at : Q5. Montague-Chelmsford Reforms were introduced in India in the year : Q6. The first industrial policy was announced in: Q7. Shingo-La is the name of which pass? Q8. ‘Madho Rai ki Jaleb’ is the main attraction of which popular Himachali fair? Q9. What is the architectural style of the temple built on the edge of the Prashar Lake? Q10. In which district Central Government approved Rs. 80 lakh in 2011 for setting up of Snow Leopard centre? Q11. Who is the author of the book ‘Polyandry in the Himalayas’? Q12. The term 'Outwash plain' is associated with : Q13. The concept of Normal Cycle of Erosion was given by : Q14. The term 'Ria' stands for : Q15. Who became first female President of Singapore? Q16. Why Jagmeet Singh is in news? Q17. Who is appointed as New Chairman of ISRO? Q18. Renewable source of energy is : Q19. Photosynthesis occurs in : Q20. In which part of our body RBC formed? Q21. In which year 'Project Crocodile' was launched in India? Q22. 'Rataundhi' is due to deficiency of which Vitamin? Q23. 'Dohru' is a traditional woollen dress worn by the women of which district? Q24. The town of Rishra is known for the industry of: Q25. Koraput has rich deposits of : Q26. Who won the Women’s Single Wimbledon tournament in 2017? Q27. Name the sportsperson who has been honoured with 'Bharat Ratna'? Q28. 'Appiko Movement' was leaded by : Q29. Which of the following is a non-renewable energy source? Q30. Which famous football tournament was started in 1888 from Annadale? Q31. Which was the old capital of Kullu Rajas from where the twelve generations of Kullu Rajas ruled? Q32. The liquid used in a clinical thermometer is: Q33. Who was awarded Best Actress award in the 64^th National Film awards function? Q34. Match List-I and List-II and select the correct answer from the codes given below : Q35. How many sustainable developments are there in the U.N. Agenda 2030? Sustainable Development Goals (SDGs) Q36. Which one of the following is the largest industry in India? Q37. Which one of the following states have the highest percentage of irrigated area? Q38. The term 'deflation' is associated with : Q39. Which one of the following has the largest number of active volcanoes? Q40. Which one of the following planets has the shortest day? Q41. Which of the following valleys lies between the Dhauladhar and Pir Panjal? Q42. Which of the following Himalayan zones occupies the maximum percentage of the total geographical area of Himachal Pradesh? Q43. Where does river Yamuna, after flowing through Uttrakhand, enter in Sirmaur district of Himachal Pradesh? Q44. The movement started under the aegis of ‘Kisan Sabha’ in which Shri Lakshmi Singh and Vaid Surat Singh played a pivotal role was: Q45. ‘Democratisation of the Administration’ was the main aim of: ******ebook converter DEMO Watermarks*******

Q46. When Himachal Pradesh got right to participate in the election for the first time under the new constitution? Q47. Which of the following Monasteries was built by a Western Tibet ruler ‘Ye-Sashoad’ around the year 996 A.D.? Q48. In 1804 A.D. Rulers of Bilaspur, Mandi, Chamba and other small rulers of Kangra area invited whom to invade Kangra? Q49. In 1848, who suppressed the rebellion of Wazir of Nurpur and then Pramod Chand and Rajas of Jaswan and Datarpur? Q50. What is the approximate capacity of ‘Shontong Karcham’ Hydro Electric Power Project? Q51. To improve the Gross Enrolment Ratio (GER) in Higher Education, the government of H.P. has adopted ‘Rashtriya Uchtar Shikha Abhiyan’ the centrally scheme in the year: Q52. The Mushroom Development Project in Palampur was launched in 1985 with the introduction of the technology of which country? Q53. What was the approximate decadal population growth rate of Himachal Pradesh in census 2011? Q54. What is India’s estimated GDP growth rate for 2018-19 in the Economic Survey? Q55. What was the Index of Industrial Production in December 2017? Q56. With what is 'Operation Green' related? Q57. Who is the first woman advocate whose name has been recommended by the collegium to become the Judge of Supreme Court of India? Q58. What is the bank account available to Dalits and Women under 'Stand-up India? Q59. Immunization of all children by 2020 is related to: Q60. According to the Global Nutrition Report 2017, how many females suffered from anaemia in India? Q61. What is the estimated social sector expenditure in budget 2018-19? Q62. How many poor families will be provided with how much medical insurance per family in 2018-19 under the Ayushmaan Bharat Yojana? Q63. How many hours per year are the people expected to contribute for cleanliness under the Swachh Bharat Abhiyaan? Q64. Match the items given in column I and II: Q65. In context of environment, the term “dirty dozen” refers to: Q66. Vast majority of biodiversity occur in : Q67. Freshwater present on earth is : Q68. Which industry labours are most prone to Cytosilicosis disease? Q69. Which of the following is not a fundamental unit? Q70. The image formed by a convex mirror is : Q71. The global warming has resulted in: Q72. Core of a transformer is made of: Q73. Which of the following is in the deficiency in a body during 'Dengue' fever? Q74. Who won the Nobel Prize for Chemistry in 2017? Q75. Why Hambantota Port has been in news in 2017? Q76. Who said that, “The Constituent Assembly was a one party body in The Assembly was the Congress and the Congress was India”? Q77. Adi Ambedkar Samaj movement is related to which state? Q78. When did Communist Party of India (Marxist) emerge a separate political party? Q79. Who described the critical and creative role of politics in India as 'the Indian model of development'? Q80. The National Forest Policy was launched in : Q81. The rate of decrease of temperature in the troposphere is : Q82. In which case did The Supreme Court give the doctrine of ‘Basic Structure’ of the constitution? Q83. Which states/union territories are involved in the Cauvery river dispute? Q84. Supreme Court’s judgment in Indira Sawhney Vs. Union of India case deals with: Q85. Which of the following is not associated with electoral reforms in India? Q86. Who described Indian Capitalism as 'Dharmshala Capitalism'? Q87. Who said, "Indian federalism is a new kind of federation to meet India’s peculiar need’s"? Q88. Which of the following leaders had worked for tribal upliftment? Q89. Which of the following leaders was never associated with the Indian National Congress? Q90. Find the correct poverty estimates for the given states in 2011-12, as per the Planning Commission? Q91. Which one of the following is not correctly matched? ******ebook converter DEMO Watermarks*******

Q92. Which one of the following igneous rocks has the highest density? Q93. Which of the following Bills was drafted by Dr. B.R. Ambedkar as the first Law Minister of India? Q94. Which of the following was not a socialist leader? Q95. Match List-I and List-II and identify the correct answer from the codes given below: Q96. Regarding the Interim Government of 1946 which of the following is not correct? Q97. Which of the following are statutory bodies? Q98. Match List-I and List-II and select the correct answer from the codes given below: Q99. 1. Match List-I and List-II and select the correct answer from the codes given below : Q100. 18. Match the items given in column I and II : 2017 HPAS Solved Paper Q1. One of the better achievements of Kanha National Park is : Q2. In India, life expectancy is: Q3. United Nations has declared 2017 as the International Year of : Q4. AMRUT was launched on : Q5. Which of the following instruments is used for measuring the depth of the ocean? Q6. Where from is litmus dye extracted? Q7. Which of the following are the primary colors? Q8. Nail polish remover contains: Q9. Which of the following is the most promising biodiesel crop? Q10. Light year is a unit of : Q11. The most abundant rare gas in the atmosphere: Q12. From which place does the Spiti River originate? Spiti River Q13. Nana Saheb, who fought against the English during the Indian Mutiny of 1857, was: Q14. Who presided over the first session of Indian National Congress in Bombay? Q15. Which Socio-Religious Movement of 19th-20th century adopted the practices of occult mysticism? Q16. Where the first session of Muslim League was held? Q17. Of the following, who shot dead Curzon Wyllie in London? Q18. Where was founded the Communist Party of India in 1920? Q19. Which event earned for Vallabhbhai Patel the title of "Sardar"? Q20. In which ocean ‘Mindanao Trench’ exists? Q21. Who is the author of the book "The Unstable Earth"? Q22. In order to prevent oxidation of potato chips during storage in sealed bags, chip manufacturers seal the packet in the presence of : Nitrogen in Food Preservation Q23. Which of the following has a pH of above 10? Milk of Magnesia pH Values Q24. Which of the following processes does not take place during photosynthesis? Photosynthesis Q25. If someone is suffering form the problem of acidity after overeating, what is the remedy? Baking Soda Q26. Which one of the following is a correct sequencing of the features of the Preamble of India Constitution? Q27. Which schedule of the Constitution of India contains provisions for the disqualification of a legislator on grounds of defection? Q28. Who presides over the joint sitting of the Parliament? Q29. Under which provision can the Speaker allow any member of the House to speak in his/her mother language? Q30. Power of Judicial Review, provided in the constitution, is exercised on the basis of which principle? Q31. Which one of the following policies of Government of India is related to the availability of cooking gas to the BPL Households? Ujjawala Yojana Q32. HDI is measured with reference to: The Human Development Index (HDI) Q33. Which one of the following cities is called 'Silicon Valley of India'? Q34. Which country owns the Margarita Island which was the venue of 17th NAM Summit? ******ebook converter DEMO Watermarks*******

Q35. Tehmina Janjua is the first woman in Pakistan’s history to hold the office of : Q36. Who among the following is the Chancellor of Nalanda University (2017)? Q37. To which country does the UN Secretary General, Antonio Guterres, belong? Q38. In which field was Juan Manuel Santos awarded the 2016 Nobel Prize? Q39. Donald Trump is: Q40. Teacher’s Day is celebrated on: Q41. Which Raja of Chamba transferred the capital of his state from Bharmaur to Chamba? Q42. During the reign of which raja of Nurpur princely state the forts of Maukot, Nurpur and Taragarh were captured by the Mughals? Q43. Which raja of Bushahr princely state was conferred the title of Chhatarpati by the Mughal emperor Aurangzed? Q44. By what name did the Greek scholars call the Satluj River? Q45. What is the approximate capacity of Sawra-Kuddu Hydro-Electric Power Project? Q46. At which place in Shimla District of H.P. is Government Sheep Breeding Farm? Q47. At which place in Bilaspur District of H.P. a woman Industrial Training Institute (I.T.I) has been made functional during 2015-16? Bharari Women ITI Q48. The global agreement that deals with the control of transboundary movements of hazardous wastes and their disposal is called: Q49. The biological oxygen demand of water is reduced mainly by: Q50. The use of carbon tax has been very successful to reduce emissions of greenhouse gases in : Q51. Dead zones in the world’s oceans and large lakes are characterized by: Q52. Who among the following were conferred Padma Vibhushan Awards in 2017? 2017 Padma Vibhushan Awards Q53. Which of the following represents the major loss to the human kind due to environmental degradation? Q54. India ratified the Stockholm Convention for protecting human health and the environment from persistent organic pollutants in : Q55. The number of wetlands in India listed under Ramsar Convention is : Q56. In the Budget for 2017-18, the funds allocated for MGNREGA is: Q57. What was the unemployment rate during 2015-16 based on Usual Principal Status in India? Q58. Who conducts the periodical sample survey for estimating the poverty line in India? Q59. For the Budget proposals 2017-18; which of the following is not correct? Q60. Stalactites and Stalagmites are: Q61. Coral reefs are found between: Q62. Savanna region of Brazil is locally called as: Q63. In "Concentric Zone Model" of Burgess, the fifth ring is occupied by: Q64. Brahmaputra, after joining Tista river; is known as: Q65. Match is following and choose the correct answer from the codes given below: Q66. Which of the following States of India has the highest poverty ratio? Q67. The growth rate of Himachal Pradesh during 12th Plan (2012-17) has been estimated at: Q68. As per Census 2011, literacy rate in Himachal Pradesh for male and female respectively is: Q69. Match the following: Q70. Supreme Court’s judgement in Indira Sawhney Vs. Union of India is related to which issue? Q71. Provision for Grant-in-Aid under Article 243 H for the Panchayats has been made out of which source? Q72. Which of the following statements is correct in reference to the Finance Commission of India? Q73. Which one of the following is the correct statement with regard to the Emergency Provisions of Indian Constitution? Q74. Match List1 (Mines) and List2 (Minerals) using the codes given below: Q75. Who of the following, was not the Moderate leader of Indian National Congress? Q76. When was the Montague Declaration made about the introduction of responsible government in stages for India? Q77. Green Revolution made the country self-sufficient in food production but had some fallouts. Consider the following statements: Q78. Kol tribe is found in : Q79. Which of the following rivers have been termed as "living entities" by the High Court? Q80. The NOTA option was first used in the elections in : ******ebook converter DEMO Watermarks*******

Q81. According to the UN Human Development Report (2016), the world’s top three countries in terms of human development are? Q82. Who among the following has been appointed Director of the Central Bureau of Investigation (2017)? Q83. The district to which two Chief Ministers of Northern States (2017) belong, falls in the state of : Q84. The Supreme Court issued a contempt notice to the sitting High Court Judge of: Q85. The Chief Minister and Deputy Chief Minister(s) of the following states were not members of the State Legislative Assembly at the time of their appointments (2017): Q86. Which month of 2017 marks the 100th anniversary of Gandhi’s arrival in Champaran? Q87. Who among the following were awarded Rajiv Gandhi Khel Ratan Award for 2015-16? Q88. Who among the following was conferred Jnanpith Award in 2016? Q89. According to legend which God (Devta) killed Banasur who was the ruler of Sarahan area of upper Shimla? Q90. Who was the first European to draw attention to the history of royal families of Jalandhar and Trigarta? Q91. Around which year did the Gurkhas capture the Sirmaur princely state? Q92. Which Himachali revolutionary edited Sudarshan Chakra magazine from Lahore during the preindependence period? Q93. What is the number of women members in the present H.P. Vidhan Sabha? Q94. How much subsidy is given in H.P. to the women of eligible category for providing Gas connection under Mata Shabri Mahila Sashaktikaran Yojana? Q95. What is the architectural style of Hateshwari Temple at Hatkoti (in Jubbal Tehsil of Shimla District)? Q96. Which of the following subject matter is not found in the Kangra Kalam style Kangra paintings of eighteenth century? Q97. In which two Districts of H.P. will the H.P. Forest Ecosystem Climate Proofing Project be implemented? Q98. According to 2010-11 Agricultural census what is the percentage of small land holding in H.P.? Q99. Which cement company has been given a prospecting license to carry out a detailed study for proving the quantity of lime-stone deposits in Sugrathi-Thangar-Kura Khera-paulikhera-Kandal–dedera of Tehsil Chopal in Shimla District, H.P.? Q100. Who is the author of book Kullu and Lahaul? 2015 HPAS Solved Paper Q1. Which of the following is not a freshwater ecosystem? Q2. Which of these is not an ecosystem service? Q3. Desert ecosystem are characterized by annual rainfall of less than Q4. Which of these is not a factor threatening extinction of several species in India? Q5. Which of these places is reputed to be world’s most biologically diverse? Q6. India is to home to how many (approximately) mammalian species? Q7. According to Paris agreement on climate change, the rise in average temperature of earth’s surface above pre-industrial levels should be limited to: Q8. In which of the following states, ‘Hornbill’ festival has been observed to uphold Unity in Diversity? Q9. Which of the following is not correct about the Harappa’s contacts with its contemporary civilizations? Q10. Who built the Sudarshana lake in the Saurashtra region? Q11. Which of the following Bhakati poets of Maharashtra conveyed his teachings through the numerous Abhangas? Q12. Dara Shikoh was the disciple of which sufi saint? Q13. Who established the East India association in 1866 in London? Q14. How much time did Lord Mountbatten give to Sir C. Radcliff to delineate the international border of India in 1947? Q15. India celebrated its first constitution Day on? Q16. In which year was the Protection of Women from Domestic Violence Act passes? Q17. Ashish Bose is well known for his concept Q18. Bombyx mori is a Q19. Which of the following Indian States will get the first ever solar-stellar observatory? Q20. Which is not an indicator of Human Development Index? Q21. Which soil is best for cotton cultivation? Q22. Bermuda Triangles is located in? Q23. Which is the longest river of the World? Q24. When a body is taken from the earth to moon ******ebook converter DEMO Watermarks*******

Q25. Decibel is used to measure the intensity field of Q26. A pressure cooker works on the principal of Q27. Sweat mainly serves to eliminate Q28. In which basin is Upper Joiner hydro-power project? Q29. The acronym UNFCC represents: Q30. Identify the correct sequence of countries with the highest biodiversityQ31. In the context of climate change, measures undertaken by Government of India have a target of electric power generation by renewable energy (by year 2030) of Q32. In a poultry unit the factor most influencing the cost is the cost of : Q33. Which of the following is not the feature of caste system? Q34. Which state of India was the first to pass Jamindari Abolition Act? Q35. Which of the following vegetable oils does not contain essential fatty acids? Q36. Which of the following statements is correct? Wheat Bran is Rich Source of Fibre Q37. Vegetables are easily perishable because of their high content of: Q38. Of which rivers tributaries are Phojal, Sarwati and Hansa streams Q39. What is the archaeological style of Adi Brahama temple Khokhan? Q40. With which region of H.P. is Bonangchyu dance associated? Q41. Who was the court poet of Raja Dharam Chand of Kangra who wrote Dharam Chand Q42. To which princely state did Zorawar Singh, General of Raja Gulab Singh who invaded Ladakh and Baltistan during 1834-41, belongs ? Q43. When did the Chandrahabhaga Valley which was held as Jagir by the families of Tibetan origin come under sway of Kullu ? Q44. The territory of which of the following princely state was divided between two cousins/scions Vijay Singh and Ram Singh during the nineteenth century? Q45. According to 2011 census which district of H.P. has the highest number of urban households? Q46. Around which year did captain RC Lee set up an apple orchard in Kullu area? Q47. Which Raja of Bushahr princely state received Hang-Rang valley from Tibet as Jagir? Q48. Around which year did Russian painter Nicholas Roerich visit Naggar in Kullu District for the First time? Q49. Which Raja of Bangahal princely state was treacherously killed by Raja Siddha Sen of Mandi? Q50. Which school did Amrita Shergil join in Shimla around 1924-25? Q51. Which Raja of Sirmaur princely state shifted the capital of his state from Rajban to Kalsi in the beginning of the thirteen century? Q52. At which place in Kullu District of Himachal Pradesh is fish farm? Q53. Who founded Himachal Pradesh congress (HVC)? Q54. Which two districts of H.P. are covered under Backward region Grant fund of ministry of Panchayati Raj, Government of India? Q55. Which bank is collaborating with the Himachal Pradesh government in implementing Doodh Ganga Yojna? Q56. Which agency is managing Himachal Pradesh state wide Area networks (HIMSWAN) services? Q57. Who among the following persons has been appointed as UP’s Lokayukta? Q58. Who among the following persons has secured India’s 12th Olympic quota berth for shooting for the Rio Games? Q59. The National Airquality Index (AQI) measures air quality by taking into consideration which of the following pollutants? Q60. Which is incorrect about Himalaya? Q61. Who of the following has been awarded Padma Vibhushan? Q62. Consider the following statements regarding IRNSS-1E: Q63. The Deen Dayal Upadhyaya Gram Jyoti Yojana: Q64. Who among the following was the constituent advisor to the Constituent Assembly? Q65. Which one of the following festivals was most famous in Vijayanagar empire ? Q66. Which of the following states were not annexed to the English East India Company’s dominions under Dalhousies’s Doctrine of Lapse ? Q67. When was the Civil Disobedience Movement finally called off by Mahatma Gandhi? Q68. Consider the following: Q69. With regard to the recently approved "Pradhan Mantri Fasal Bima Yojana" ******ebook converter DEMO Watermarks*******

Q70. What has been the growth rate of GDP of India during 2014-15 at the current prices and constant and constant prices respectively? Q71. Consider the following statements regarding Corporate Tax Avoidance deal : Q72. Pradhan Mantri Fasal Bima Yojna was approved by Central Government on: Q73. Swabhiman is a: Q74. Which of the following is not a function of Reserve Bank of India? Q75. As per HDR, 2014 the Gender Inequality Index of India is: Q76. The 14th Finance Commission has enhanced the states' share in the central divisible pool taxes from: Q77. As per the Census 2011, the average age is above 30 years for the following states: Q78. What is the name of shifting cultivation in Mexico ? Q79. Which one of the following is incorrect? Q80. Which one of the following is incorrect ? Q81. Which lines separates India from Afghanistan ? Q82. Varkala beach is located in : Q83. Which one of the following is not correctly matched ? Q84. Which of the following statements is incorrect regarding Zika virus? Q85. What is the population density (per sq. km) of India and Himachal Pradesh respectively as per Census 2011? Q86. Which of the following human rights come within the ambit of Article 21 of the Indian Constitution? Article 21 Q87. During the term of office, criminal proceedings cannot be instituted against: Q88. Who among the following decides the question of disqualification of a member of Parliament on the grounds of defection? Q89. Who among the following has the power to declare a caste or a tribe as scheduled caste or scheduled tribe? Q90. Who among the following determines the area of a Lok Sabha constituency for the purpose of election? Q91. The Central Information Commission falls under: Q92. Which of the following statements is correct about NITI Aayog? Q93. Which of the following committees recommended the creation of 'Nyaya Panchayats'? Q94. Which of the following are All India Services? Q95. Who is the Chief Economic Advisor to the Government of India? Q96. Consider the following statements regarding International Solar Alliance (ISA) : Q97. Consider the following statements regarding Corruption Perception Index (CPI), 2015 : Q98. Who among the following persons has won the Man Booker Prize, 2015? Q99. First Census was conducted in which year in India? 2014 HPAS Solved Paper Q1. The latitude that passes through Sikkim also passes through Q2. Which of the following does not contain silver? Q3. Aluminum surfaces are often ‘anodized’ this means the deposition of a layer of: Q4. When a CD (compact disc used in audio and video systems) is seen in sunlight, rainbow like colours are seen. This can be explained on the basis of the phenomenon of: Q5. Domestic electrical wiring is basically a: Q6. The only snake that builds a nest is: Q7. The hormone insulin is a: Q8. The normal temperature of human body in Kelvin scale is: Kelvin to Celsius Formula Q9. Which one of the following polymers is widely used for making bullet proof material? Q10. Name the site where Harappan ploughed field has been found: Q11. Name the place where Jaina texts were finally complied. Q12. Which of the following Marathas was given a Manasab of 7000/7000 by Aurangzeb? Q13. Who among the following translated Kalhana’s Rajatarangini into English? Q14. Sidhu and Kanhu were associated with which one of the following? Q15. NITI in NITI Aayog stands for Q16. Which of the following is/are treated as artificial currency? Q17. SEBI was established in the year Q18. The working principal of a washing machine is ******ebook converter DEMO Watermarks*******

Q19. Which of the following drugs reduces fever? Q20. Mars Orbitor Mission is called: Q21. In which district of HP is Yunam-so Lake? Q22. What is Shingni-Mingni? Q23. According to Planning Commission estimates for 2011-12 which of the following states have maximum percentage of population below poverty line Q24. Diego Garcia Island is located in Q25. Great Salt Lake is located in: Q26. Tropic of cancer does not pass through Q27. Which is the second most spoken language in India? Q28. Who among the following is the author of Milind Panho? Q29. Find out among the following, the author of Muasir-i-Alamgiri? Q30. Polworth, Corriedole, Blackpace and Bingi are varieties of: Q31. Who was the head of the boundary commission which demarcated the boundaries of India and Pakistan? Q32. What is the local name of shifting cultivation in Malaysia? Q33. When was ‘Beti Hai Anmol’ Yojna launched in Himachal Pradesh? Q34. Disguised unemployment generally means: Q35. Which of the following amendments to the Indian Constitution has made the right of education a fundamental right? Q36. Which of the following are the exclusive committees of Rajya Sabha? Q37. The elected members of the state legislative assembly participate in the elections of Q38. Which of the Commissions/Committees has recommended that there should be Minister of Panchayati Raj? Q39. Pradhan Mantri Jan Dhan Yojna was launched on Q40. Which of the following has the lowest sex ratio in India? Q41. Which of the following statements is/are correct ? Q42. Which of the following is the largest coal producer? Q43. Which of the following is the correct order of rivers from north to south? Q44. Which Mughal ruler is said to have given the title of ‘Mian’ to the Rajput princes of Punjab Hill states who were held hostages in the royal court? Q45. Ruler of which of the following hill states were from a common ancestor? Q46. Who started the Ice-Skating Rink at Shimla around 1920? Q47. What was the annual growth rate in the economy of Himahcal Pradesh during the 2012-13 fiscal? Q48. Which one of the following is a Pagoda style temple in HP? Q49. Identify the correct order of density of population among the following districts of HP according to 2011 census data (in descending Q50. Which district of HP occupies top position in the production of ginger? Q51. India signed an agreement on transfer of sentenced persons with the Hong-Kong special administrative region in 2014. Q52. Based upon the relations between India and Pakistan, Khurshid M. Kasuri, the Ex-foreign Minister of Pakistan has recently penned down a book. What is the title of the book? Q53. Which of the following statements are correct about the National Human Right Commission? Q54. Which of the following agriculture practices is eco-friendly? Q55. According to the Wealth Health Organization, the desease which causes the death of the largest number of people today is Q56. The Raja of Bushahar princely state was assisted by three hereditary Wazir families. One of them was Kohal. Where did it come from? Q57. When was the Department of Public Relations created in Himachal Pradesh? Q58. In India, which of the following have the highest share in the disbursement of credit to agriculture and allied activities? Q59. The World Investment Report (WIR) is published by Q60. What is the correct sequence of the following four stages in the public policy formulation process? Q61. Annual Summit 2015 of BRICS nations was held in Q62. The most widely used antacid is: Q63. In which district of HP is Saryanj Sarma watershed project? Q64. A simple machine helps a person in doing: ******ebook converter DEMO Watermarks*******

Q65. Which Raja of Spiti invaded Kullu and made it a tributary? Q66. As per the IPCC 4th Assessment Report, the average sea level during the period 1961-2003 increased approximately at the rate of Q67. Who abolished Dual Government in Bengal? Q68. The temples located in the region between the Vindhyas and the Krishna river are known as: Q69. Which one of the following pairs is not correctly matched? Q70. According to Dr. B. R. Ambedkar, which of the following articles of the Indian Constitution was to remain a dead-letter? Q71. The responsibility of law and order rests with the police commissioner in the cities of Q72. Which one of the following functions does not figure in the 11th schedule of the constitution (73rd Amendment)? Q73. Driving force for an eco-system is Q74. Read the following statements regarding the recently released NTCA report on the status of tigers in India, 2014. Q75. At present, the primary energy source in the world is Q76. Which of the following has the maximum bio-diversity? Q77. World Ozone day is celebrated on Q78. In India, the forest cover percentage of the total geographical area of the country is appox. Q79. A fuse is used in main electric supply as safety devices. Which one of the following statements about the fuse is correct? Q80. What is the script of the earliest epigraphical rock inscriptions at Pathyar and Kanhiyara in Kangra? Q81. How many biosphere reserves are there in India? Q82. Earth Summit 2012 for sustainable development was held in Q83. The sensation of fatigue in the muscle after prolonged strenuous physical work is caused by: Q84. The UN Convention on Biodiversity was ratified by the Indian government in the year Q85. National Sample Survey Office was established in the year : Q86. Match the List I with List II Q87. Recently, the government of India has decided Q88. Modi Government re-launched the Kisan Vikas Patra (KVP) investment scheme to top house hold saving for funding infrastructure development in the country. Q89. Shanta Kumar Committee has submitted its report on restructuring of Food Corporation of India (FCI) to Mr. Modi. Q90. As per the population census 2011, what is the percentage of urban population in Himachal and India respectively Q91. Which one of the following does a TV remote control unit to use to operate a TV set? Q92. Which Raja of Mandi gave the Shivratri Fair the form of a cultural festival? Q93. Which of the following peaks are in Himachal Pradesh? Q94. Find the mismatch in the following pairs: Q95. When did the number of members in the HP Vidhan Sabha increase from 36 to 41? Q96. Almost all major oil produces including Saudi Arabian have become dependent on high prices, with break-even level usually over $90 per barrel to meet their fiscal obligations. Consider the following can be termed as explanation for the present decline in oil prices? Q97. The world’s largest ground based telescopic observatory is located in: Q98. Apart from Carbon-dioxide, which of the following gases also have a role in climate change? Q99. At present, which country is the maximum emitter of Carbon-dioxide? Q100. What is the tenure of Chief Election Commissioner of India? 2013 HPAS Solved Paper Q1. What is the name of Pakistani Bus that plies between Lahore and Delhi? Q2. Standard 18-carat gold sold in the market contains Q3. Which of the following is represented by the Harrapan Civilization? Q4. In which of the following year the Asiatic Society of Bengal was established? Q5. Who is Shinzo Abe? Q6. The major component of honey is Q7. With which district of Himachal Pradesh is Kayang folk dance mainly associated? Q8. Which lake is jointly owned by China and India? Q9. "There is enough for everybody’s need but not enough for everybody’s greed." Who said it? ******ebook converter DEMO Watermarks*******

Q10. Which one of the following is not trade block? Q11. Climatology is the science of: Q12. Zulus tribe is found in? Q13. Which one of the following is the largest producer of coffee in the world? Q14. Which is the tallest dam of India? Q15. Which of the following rivers is called ‘Nadittam’ in Rigveda? Q16. The officer who enjoyed authority over the large pasture ground was known as during the Rigvedic period? Q17. Usually fuels on burning causes pollution. which one of the following fuel causes minimum environmental pollution ? Q18. What is the average fat content of "Buffalo milk"? Q19. On the bank of which river is Nadaun town of Hamirpur District? Q20. Which of the following is the author of Tamil Ramayana? Q21. Jhum is a term used for: Q22. When ants bite, they inject: Q23. The 'Stones' formed in human kidney consist mostly of: Q24. Which tehsil of Mahasu district was carved out to form Kinnaur district? Q25. For which work did Mridula Garg get the 2013 Sahitya Academy Award? Q26. According to Transparency Intentional which are the most honest countries in the world in 2012-13? Q27. Who was crowned as Miss World in 2013 beauty contest? Q28. The UN Human Rights Council (HRC) was set up in: Q29. Which day is observed as Anti-Leprosy Day? Q30. Arteries supplying blood to the: Q31. Cryogenic engines find applications in: Q32. Which of the following is not in the vicinity of Kullu? Q33. In which town is Panchvaktra temple? Q34. Ultimate source of energy for living being is: Q35. Soil erosion can be prevented by: Q36. With which game/sport is Deepika Kumari associated? Q37. When was Telegraph/Telegram service closed in India? Q38. Fat present below the skin surface in our body, acts as a barrier against: Q39. In which year publication of Hindi daily Divya Himachal begin? Q40. Who was the first person to vote in the First General Elections in India in 1951-52? Q41. Which of the following has the highest sex ratio as per 2011 census? Q42. Which Indian State has the lowest death rate according to census 2011? Q43. Which one of the following is located in Atlantic Ocean? Q44. Diffusion of light in the atmosphere takes place due to: Q45. The characteristics odour of garlic is due to: Q46. Which lake at Mandi is known for floating island? Q47. Which of the following are the ways of acquiring citizenship of India? Naturalization Q48. The Right to Freedom under Article 19 of the Constitution of India may be suspended by President of India? National emergency under article 352 Q49. Which of the following languages have been declared by the Constitution to be the languages for conducting business in Parliament? Q50. Which is the correct measure of Infant Mortality Rate (IMR)? infant mortality rate Q51. After which American scientist is ‘h-index’ named? h-index Q52. NABARD came into existence in the year: Q53. Who is George Alexander Louis? Q54. Which of the following is believed to be the oldest among Shimla Hill States? Q55. Which one of the following is taken as the Base Year for estimating GDP at constant prices in India during 2012-13? Q56. "What was the average annual growth rate of Agricultural and Allied Sectors in India during 11th five ******ebook converter DEMO Watermarks*******

year plan? Q57. Which of the statements given below is/are correct? Q58. The Targeted Public Distribution System (TDPS) was adopted in the year: Q59. What were the main findings of the N.N. Vohra Committee? Q60. Who among the following increased the Mansab from 5000/- to 7000/-? Q61. Which of the following political parties was called ‘Rajadroha Ka Karkhana’ by the British? Q62. Who among the following established the Central Hindu School at Banaras? Q63. Rani Gidalu was one of the freedom fighter from the north-east. She hailed from one of the following states Q64. Which of the following states did not return Congress in power after the first provincial elections in 1937? Q65. Match names of List I with that of List II and select the correct answer from the codes given below: Q66. Which one of the following is incorrect? Q67. Which is not temperate grassland? Q68. In which of the following rivers ‘Swargadwari’ is located at? Q69. Which one of the following is incorrect? Q70. Which is the correct descending order in terms of length (km) of river? Q71. Which of the following pairs is not correctly matched? Q72. What is India’s Global Position out of 187 countries as per UNDP’s Human Development Report 2011? Q73. Who is the Chairman of Commission for Agricultural Costs and Prices (CACP) at present? Q74. An air bubble in water will act like a: Q75. Which one of the following is not an essential micronutrient for plants? Q76. Identify the correct chronological order in which the following Europeans travelled in Punjab and Shimla Hill states? Q77. Who negotiated the arrangement between the states of Kashmir and Chamba by which the later became independent of the former? Q78. Raja of which state was treated by the wife of Dara Shikoh as her son? Q79. When did the consultative body of ruler of princely states known as Narendra Mandal came into existence? Q80. Who led the famous Mandi conspiracy of 1914-15? Q81. According to Annual Status Education Report (ASER) 2012-13 what percentage of class V students of Himachal Pradesh Government and private schools can read only class II text? Q82. What was the exact constitutional status of the Indian Republic on 26th January 1950? Q83. Disagreement between the two Houses of Parliament on the following bills can be resolved by the Houses in a joint sitting Q84. In a National Park, protection is provided to: Q85. Which of the following union territories do not have any representative in the Rajya Sabha? Q86. Which among the following committees was the first to demand constitutional recognition for Panchayats? Q87. In which of the following states have the Panchayati Raj institutions been set up in conformity with the traditions and customs of the local people? Q88. Who among the following former Presidents contested the election to the office of the President of India without resigning from the office of Vice-President of India? Q89. Which one of the following models/theories views public policy as a continuation of previous government activities with some modifications? Q90. The Khadi and Village Industries Commission was established during which of the Five Year Plans? Q91. In which field did Frederick Sainger win Nobel Prize twice? Frederick Sainger Q92. Which of the following types of glasses can cut-off ultraviolet rays? Q93. Which one of the following is an active component of clove oil? Q94. In eye donation, which one of the following parts of donor’s eye is utilized? Cornea Q95. Which action of Raja Sansar Chand prompted the Hill Chiefs to invite Gurkhas to invade Kangra? Q96. When was a Sanad granted to the Raja of Bilaspur by the British government confirming his possession on the right bank of Satluj? Q97. Court poet of which princely state of Shimla Hills is the author of Shashi Vansh Vinod? Q98. Which mountain pass connects Spiti to Ladakh? ******ebook converter DEMO Watermarks*******

Q99. With which of the following is NGO named Vishakha is associated? 2012 HPAS Solved Paper Q1. At which place in Shimla District is Herbal garden? Q2. Koderma, a place in Jharkhand , is famous for Q3. What is Curiosity Rover? Q4. The continental drift theory was propounded by Q5. What are stalagmites? Q6. With which of the following is Losar festival associated? Q7. According to legend what is the vehicle of Kamdev? Q8. The drugs used to get relief from pain are called: Q9. Metals are good conductors of electricity because Q10. With which game is Ravi Rampaul associated? Q11. Against whom did Aurangzeb fight the battle at Samugarh? Battle of Samugarh Q12. Who was Amrita Shergil? Q13. In which year Shimla was declared as the summer capital of British India? Q14. Which of the following states Nathpa Jhakri Hydel Project under execution Q15. Which country is called the 'Sugar Bowl' of the world? Q16. Who was the first Caliph of Islam ? Q17. What is INS Chakra? Q18. In coming years , skin related disorders will become more common due to : Q19. In the process of galvanization of an iron sheet is coated with: Q20. One liter of water is equivalent to: Q21. A transformer is used for: Q22. Where is KEE monastry? Q23. Which type of pollution is caused by two - wheelers? Q24. Chernobyl disaster was caused by a : Q25. First share market in India was established in : Q26. Who founded the revolutionary organization named 'Abhinava Bharat' ? Q27. At what interval is Bhunda festival of Nirmand held? Q28. Who is the author of 'Abode of God'? Q29. Which of the following is not a tributary of the Beas? Q30. With which region of Himachal Pardesh is Sih ballet associated? Q31. Which Sultan of Delhi helped Raja Megh Chand of Bilaspur to regain his lost kingdom? Q32. Which Raja of Mandi built the Darbar Hall in 1906? Q33. Who won the 2012 Himachal Pardesh Vidhan Sabha election by the highest margin? Q34. Youth of which age group are eligible to avail the skill development allowance announced by the Himachal Pradesh Government in 2013-14 budget? Q35. Match the following proposed rope-ways correctly and choose the correct answer from the codes given below: Q36. According to 2008-09 data, which district of Himachal Pradesh has recorded the highest percentage in the production of pulses? Q37. Which one of the following is not a precondition for implementing national programme for education of girls at the elementary level? Q38. Which is scheme has been introduced in place of Kishori Shakti Yojana in some district of Himachal Pradesh? Q39. Which sector had highest outlay (in % terms ) in the annual plan of 2011-2012? Q40. Beside Kinnaur and Lahaul Spiti, which district of Himachal Pradesh has recorded the lowest percentage of urban population in the 2011 census? Q41. What was the percentage of female literacy in Himachal Pradesh in 1951? Q42. Match the following: Q43. Whom did emperor Ashoka send to Lanka to promote Buddhism? Q44. There are four Upvedas. Three of them are Ayurveda, Dhanaurveda and Sthapatya veda. Which is the fourth? Q45. Which among the following scholar belong to the later Vedic period? Q46. Whom did the Huns defeat to establish their in India? ******ebook converter DEMO Watermarks*******

Q47. The first Rani of Mewar who committed johar was Padmini. Who was the second? Q48. At which place did the battle between Babur and Rana Sanga take place 1527 ? Q49. Who organized the Red Shirts Movement ? Q50. Who is associated with the moderate strand of the Khilafat movement of 1919-20? Q51. The marked of Moshad the spy agency of Israel is that it has as its member a large number of Q52. What is the major religion of Myanmar? Q53. With which field is Malala Yousafzai associated? Q54. Identify the country whose seat of legislature, executive and judiciary is at three different cities? Q55. Which one of the following refused the Nobal Prize awarded to him? Q56. When was the World Anti-Doping Agency(WADA) set up? Q57. Which of the following states records the highest proportion of scheduled castes population ? Q58. The most ideal region for the cultivation of cotton in India is Q59. Which of the following factors is responsible for the rapid growth of sugar production in south India as compared to North India? Q60. Which of the following state groups is the largest producer of iron-ore in india? Q61. Whose government was defeated for the first time on a vote of confidence? Q62. The Question Hour in the House of the people is followed by Q63. CENVAT is assoicated with : Q64. The 73rd and 74th constitutional Amendments were during the primeministership of: Q65. The Indian Constitution refers to minorities based on Q66. Which Indian company has been included for the first time in USA’s index NASDAQ-100? Q67. UNDP has introduced a new poverty index known as: Q68. The excess discharge of fertilizers into water bodies result in: Q69. To which community/organisation maximum goods are exported from India? Q70. The antiknock agent added to unleaded petrol is: Q71. 3-4 benzopyrene causes Q72. Natural silk is a : Q73. Which of the following states have recently (2013) got separate high courts? Q74. Most hazardous metal pollution of automobile exhaust is: Q75. If BOD of a water is found very high, this means water: Q76. Minamata disease affects the : Q77. Chemical which cause bone cancer and degeneration of tissue is : Q78. Who was the Chairman of the committee on pricing and taxation of petroleum products(2013)? Q79. When was World Trade Organization set up by the member countries of the United Nations to promote trade among countries? Q80. Which of the following commissions/committees have recommended legal protection to whistle blowers? Q81. For which of the following posts has the Supreme court directed the central government that not only civil servants but other persons with impeccable integrity should also be considered? Q82. Which of the following occupied the highest place in growth rate during the 11th Plan period 2007-2012? Q83. Which of the following can be remolded time and again without producing any change? Q84. Most commonly used tranquillizers are derivatives of: Q85. At what capacity was P.C. Mahalanobis associated with the formulation of the economic policy of independent India ? Q86. The range of normal human hearing lies between: 2011 HPAS Solved Paper Q1. Who was the first Indian to declare that "Freedom is my birth right"? Q2. At which place was Vardhman Mahavir born? Q3. Zawar mines are important for: Q4. Which Mughal ruler imprisoned his stepmother after the death of his father? Q5. Who founded the All India Depressed Classes Federation? Q6. From which place did Mahatma Gandhi ji started Dandi March to the sea? Q7. Which of the following is not a measures of Human Development Index? Q8. At which place in Mandi District has the Department of Ayurveda set-up herbal garden? Q9. In which district of Himachal Pradesh is Mulgun valley? Q10. Which statement is true about Ellora caves? Q11. Who was the founder of Arya Samaj? ******ebook converter DEMO Watermarks*******

Q12. Under the constitution of India, the power to issue a writ of Habeas Corpus is vested in: Q13. On which river is Chamba Pattan Bridge which being without pillars is first of this kind in Himachal Pradesh? Q14. Participatory Notes (PNs) are associated with which one of the following? Q15. Silicon carbide(SiC) is known as: Q16. Where was the capital of Kullu in the earlier times? Q17. In the coast of which state is Wheeler Island? Q18. Five rings linked together constitute the symbol of Olympic Games. Each ring is of a different color. Two colors are blue and black. Which are the other three colors? Q19. Who translated the collection of South Indian tales into Sanskrit and titled the work as ‘Kathasaritasager’? Kathasaritasager (कथास र ागर) Q20. Arrange the following ruling dynasties of ancient India in correct chronological order: Q21. In which year was the Partition of Bengal annulled? Partition of Bengal Meaning of Annulled Q22. Haem is the important biologically in the myoglobin which is used to store: Haem Q23. Which treaty signaled the departure of the Gurkhas and arrival of the British on the political canvas of Shimla Hill states? Treaty of Sagauli Q24. Mediterranean agriculture is: Mediterranean agriculture Crops in Mediterranean agriculture Q25. Which city in India is called the Garden city? Q26. Koyali is: Gujarat Refinery (Located at Koyali in Gujarat) Q27. Which group of rivers originates from the Himachal mountains? Q28. The Public Accounts Committee presents its report to: Public Accounts Committe Q29. The Governor has the power to impose Governor’s rule in the State of: Q30. Which Indian city has the highest population according to 2011 Census? Q31. ‘Nirmal Bharat Abhiyan Yojna’ is associated with: Nirmal Bharat Abhiyan Q32. Fly ash is environment pollutant produced by: Fly ash Q33. What is the capital of Republic of South Sudan? Juba Q34. What is the name of first Indian Research Station in the Arctic? Himadri Station Q35. Who has been given the 2011 Sahitya Academy Award? Sahitya Academy Award Q36. With which activity is Magsaysay awardees Rajendra Singh associated? Dr. Rajendra Singh Q37. After Subhash Chandra Bose resigned from the President ship of Congress in 1939 who became the party president? Q38. Who were the two prominent leaders of Home Rule League? Home Rule Movement (1916 - 1918) Major Leaders Q39. Environment planning organization is: NEERI Q40. Shallow lake with rich organic products are called: Eutrophic Q41. Which of the following plans is meant for constructing houses for rural people? Q42. In 1943, Muslim League ministers were installed in four provinces. Two of them were Sind and NWFP. Which were the other two? Q43. At which venue did the 1907 A.D. split in Congress Party take place? ******ebook converter DEMO Watermarks*******

Q44. Tarapore Committee was associated with which one of the following? Q45. The Western Ghats and the Eastern Ghats meet at: Q46. Rusting of iron is: Q47. The urine sample of a diabetic patient contains Q48. The quality of diesel oil for use in diesel engines is determined by: Q49. Which of the following is used as moderators in nuclear reactors? Q50. The 40 years war between ancient Aryan king Divodas and Shamber resulted in the defeat of: Q51. Who built the Lakshmi Narayan Temple at Chamba? Q52. Which of the following is not a natural polymer? Q53. When mild steel is heated to a high temprature and suddenly cooled in water, it becomes hard and brittle. The process is called: Q54. Which Raja’s wooing of Nokhu Gaddan is depicted in the "Raja-Gaddan" love lyric? Q55. For distribution of essential commodities all families in Himachal Pradesh have been divided into four categories. Two of them are: Below Poverty Line (BPL), and Above Poverty Line (APL) families. Which are the other two are? Q56. Dr. Piyush Guleri’s award winning work, "Chhaunte" is a ……………………. Q57. According to 2011 census which district of Himachal Pradesh has the lowest female literacy rate? Q58. On which issue did several Panchayats of Nichar Sub-division boycotted the December 2010-January 2011 Panchayat election? Q59. Almost 90% of the world’s annual fish supply comes form: Q60. A polymer which is used for making ropes and carpet fibres is: Q61. The bleaching action of chlorine occurs in the presence of: Q62. The maximum limit of Foreign Direct Investment (FDI) in public sector banking is Q63. The Pennines (Europe), Appalachians (America) and the Aravallis (India) are examples of: Q64. Three important rivers of the Indian Subcontinent have their sources near Mansarovar Lake in the Great Himalayas. These rivers are: Q65. Which one of the following is no longer a source of Income of urban local bodies in Punjab, Haryana and Himachal Pradesh? Q66. As per 13th Finance Commission Recommendations during 2010-15, transfers to the states in the form of ‘grant-in-aid’ are Q67. The Production of most metallic minerals is affected by each of the following, except: 2010 HPAS Solved Paper Q1. The definition of Biosphere is: Q2. Rhododendron is the characteristic vegetation of: Q3. Some reliable indicators of pollutions are: Q4. Gas released during Bhopal gas tragedy was: Q5. Human eye is sensitive to the wavelength in the range of: Q6. Who built the old fort (Purana Qila) in Delhi? Q7. Under the Constitution of India who is the guardian of fundamental rights? Q8. MNREGA programme is related to: Q9. Which one among the following countries is not a member of SAARC? Q10. What is the roughly ratio between the population density of Himachal Pradesh and India as per the provisional figures of 2011 Census of India? Q11. Asbestos is no longer preferred for commercial use because: Q12. Hemorrhagic dengue fever is spread by: Q13. In which district of Himachal Pradesh is Uhal Hydroelectric Project located? Q14. Who was the founder of Satyashodhak Samaj? Q15. Where was the Headquarters of Ghadar Party? Q16. Which part of the world has a high density of organisms? Q17. Carbon Dioxide absorbs strongly in Infrared region and its presence in atmosphere decreases the loss of heat from Earth by radiations. This is called? Q18. During whose viceroyalty was the Vernacular Press Act enacted? Q19. In which district of Himachal Pradesh is Surajtal Lake ? Q20. Freon causes great environmental damage. They are : Q21. What is an archipelago? Q22. Siul stream is a tributary of river? ******ebook converter DEMO Watermarks*******

Q23. Balwant Rai Mehta Committee is related to: Q24. Which term is used for imaginary lines joining the places of equal rainfall? Q25. Freons cause great environmental damage. They are: Q26. What is relevant to the Golden Quadrilateral in India? Q27. Which of the following is not a plant hormone? Q28. What is the silicon valley of United States of America famous for? Q29. What is the density of population at national level in India according to 2011 census? Q30. Which mountain range separates Sirmaur from Shimla? Q31. Who amongst the following can be the chairman of National Human Rights Commission? NHRC (National Human Rights Commission) Chairman of NHRC 2016 Chairperson Q32. Antibodies are found in: B Lymphocytes Q33. Night Blindness is caused by the deficiency of: Night blindness (nyctalopia) Causes of Night Blindless Q34. India is at which state of the following stages of Demographic transition? Q35. Consider the following related to Sarva Shiksha Abhiyan(SSA) being implemented in partnership with the States for addressing the needs of children: Q36. Which one of the following led to the Malogaon Committee to recommend for interest rate cap for mictofinance institutions? Q37. Consider the following in relation with Corporate tax: Q38. Consider the following taxes: Q39. Which of the following is the highest export-oriented handicrafts (in value terms) in India exports? Q40. Which one of the following represents at least in part investment in human capital? Q41. Which one of the following treaties was not covered by Uruguay Round Negotiations, which established WTO? Q42. Who founded the Guler state? Q43. On the basin of which river is Dodra Kawar situated? Q44. In which year was ‘Swarna Jayanti Gram Swarozgar Yojna’ launched in Himachal Pradesh? Q45. 'Mukhya Mantri Bal Udhar Yojna’ of Himachal Pradesh Government is restricted to the children belonging to: Q46. What is the architectural style of Jawalamukhi temple in Kangra District? Q47. Male literacy in the recent Census 2011 has increased to: Q48. Indian agriculture has the following features: Q49. Which one of the following is non-toxic? Q50. Carbohydrates are not used in: Q51. Nanotechnology is where particles of the following size are used: Q52. Carbon dioxide absorbs the infrared radiations and its presence in the atmosphere decreases the loss of heat from earth by radiations. Q53. Which trench is located in the Pacific Ocean? Q54. In which Indian State is the Tawang Buddhist Monastery located? Q55. For which of the following functions was V.K. Shunglu Committee constituted? Q56. Who among the following was the speaker of first Lok Sabha? Q57. Which ancient Indian king wrote the three dramas, i.e. Ratnavali, Priyadarshika and Nagananda? Q58. By which act was system of ‘Dyarchy’ introduced during British rule in India? Q59. Who was the Chairman of the first Union Commission of Backward Classes of India? Q60. According to 2011 Census which of the following districts of the state of the state of Haryana has the lowest sex ratio? Q61. Which of the following countries is divided into the largest number of time zones? Q62. In which form is phosphorus predominately found in soil? Q63. Which of the following properties is not related to LASER? Q64. Who amongst the following is empowered to declare emergency under the Indian Constitution? Q65. Under which of the following circumstances can an emergency be declared in India? Q66. Which of the following is not properly matched? ******ebook converter DEMO Watermarks*******

Q67. Consider the following in the context of food inflation in India: Q68. Who commanded the Mughal forces which subdued Kangra fort in 1620 A.D.? Q69. With whose assistance is the Himachal Pradesh Government’s ‘organic farming and water harvesting project’ being implemented? Q70. Asbestos is no longer used for commercial use because: Q71. In which one of the following cases has the Preamble been accepted as a part of the Indian Constitution?

******ebook converter DEMO Watermarks*******